Вы находитесь на странице: 1из 78

Answers to

even-numbered problems in
Mathematics for
Economic Analysis

Knut Sydster
Peter Hammond
Preface
Mathematics for Economic Analysis, Prentice Hall, 1995 has been out for a long time, and over the years we
have had many request for supplying solutions to the even-numbered problems. (Answers to the odd-numbered
problems are given in the main text.)
This manual provides answers to all the even-numbered problems in the text. These answers are taken
from the old Instructors Manual which is no longer available. (In fact, some of the old answers have been
extended.) For many of the more interesting and/or difcult problems, detailed solutions are provided. For
some of the simpler problems, only the nal answer is presented.

Appendix A in the main text reviews elementary algebra. This manual includes a Test I, designed for the
students themselves to see if they need to review particular sections of Appendix A.

Many students using our text will probably have some background in calculus. The accompanying Test II
is designed to give information to the students about what they actually know about single variable calculus,
and about what needs to be studied more closely, perhaps in Chapters 6 to 9 of the text.
Oslo and Coventry, November 2010

Knut Sydster and Peter Hammond

Contact addresses:
knut.sydsater@econ.uio.no
P.J.Hammond@warwick.ac.uk

Version 1.0
07122010 1113

Knut Sydster and Peter Hammond 2010


CHAPTER 1 INTRODUCTION 1

Chapter 1 Introduction
1.3
2. (a) Put p/100 = x. Then the given expression becomes a + ax (a + ax)x = a(1 x 2 ), as required.
(b) $2000 1.05 0.95 = $1995. (c) The result is precisely the formula in (a). (d) With the notation
used in the answer to (a), a ax + (a ax)x = a(1 x 2 ), which is the same expression as in (a).
4. (a) F = 32 yields C = 0; C = 100 yields F = 212. (b) F = 95 C + 32
(c) F = 40 for C 4.4, F = 80 for C 26.7. The assertion is meaningless.
R(1 p) S(1 q) pS qR
6. x = , y= if (p q)(1 p q)  = 0.
(p q)(1 p q) (p q)(1 p q)
(Use, for example, (A.39) in Section A.9.)

1.4
2. (a) Correct. (b) Incorrect. (c) Incorrect. (d) Correct. (e) Incorrect.
(f) Incorrect. (Usually the sum of two irrationals is irrational, but not always. For example, and
are both irrationals, but + () = 0, which is rational.)
4. (a) y 3 43 x (b) y > 23 z (c) y (m px)/q
6. |2 0 3| = | 3| = 3, |2 1
2 3| = | 2| = 2, |2 7
2 3| = |4| = 4
8. (a) 3 2x = 5 or 3 2x = 5, so 2x = 2 or 8. Hence x = 1 or x = 4.
(b) 2 x 2 (c) 1 x 3 (d) 1/4 x 1 (e) x > 2 or x < 2
(f) 1 x 2 3, and so 1 x 3 or 3 x 1

1.5
2. (a) right, wrong (b) wrong, right (c) right, wrong
(d) and both right (e) wrong (0 5 = 0 4, but 5  = 4), right (f) right, wrong
4. x = 2. (x = 1, 0 or 1 make the equation meaningless. Multiplying each term by the common de-
nominator x(x 1)(x + 1) yields (x + 1)3 + (x 1)3 2x(3x + 1) = 0. Expanding and simplifying,
2x 3 6x 2 + 4x = 0, or 2x(x 2 3x + 2) = 0, or 2x(x 1)(x 2) = 0. Hence, x = 2 is the only
solution.)

6. (a) No solutions. (Squaring each side yields x4 = x+518 x + 5+81, which reduces to x + 5 = 5,
with solution x = 20. But x= 20 does not satisfy the given equation.) (b) Just as in part (a) we nd
that x must be a solution of x + 5 = 5, which has the solution x = 20. Inserting this value of x into
the given equation we nd that x = 20 is the solution.
(1) (2)
8. (a) x + x + 4 = 2 x + 4 = 2x x +4 = 44x +x 2 x 2 5x = 0 x 5 = 0 x = 5.
Here implication (1) is incorrect (x 2 5x = 0 x 5 = 0 or x = 0.) Implication (2) is correct, but it
breaks the chain of implications. (b) x = 0. (By modifying the argument we see that the given equation
implies x = 5 or x = 0. But only x = 0 is a solution.)
10. No. (To study law it seems that you need a course in logic.)

Knut Sydster and Peter Hammond 2010


2 CHAPTER 1 INTRODUCTION

1.6
2. Logically the two statements are equivalent, but the second statement is still an expressive poetic rein-
forcement.

1.7
2. (a) No, not necessarily. (b) Yes (if anybody is both a painter and a poet).

4. F B C is the set of all


 female biology students in the university choir; M F the female mathematics
students; (M B) \ C \ T the students who study both mathematics and biology but neither play tennis
nor belong to the university choir.

6. (b) and (c) are true, the others are wrong. (Counter example for (a), (d), and (f): A = {1, 2}, B = {1},
C = {1, 3}. As for (e), note in particular that A B = A C = A whenever B and C are subsets of A,
even if B  = C.)

8. 50 35 = 15 liked only coffee, 40 35 = 5 liked only tea, 35 liked both, and 10 did not like either. In
all there were 15 + 5 + 35 + 10 = 65 who responded.

10. (a) Consider Fig. 1.7.10(a). Both sets consist of the elements in (1) and in (3).
(b) Consider Fig. 1.7.10(b): A
B consists of (3), (4), (2), and (5). Then (A
B)
C consists of (4),
(5), (6), and (7), which is described verbally in the problem.

A B
(5)
(1)
(4)
(7)
(2)
(3)
A (1) (2) (3) B (6)
(8)
C

Figure 1.7.10(a) Figure 1.7.10(b)

12. (a) Consider Fig. 1.7.10(b), and let ni denote the number of people in the set marked (i), for i = 1, 2, . . . , 8.
The responses reported imply that: n1 +n3 +n4 +n7 = 420; n1 +n2 +n5 +n7 = 316; n2 +n3 +n6 +n7 =
160; n1 + n7 = 116; n3 + n7 = 100; n2 + n7 = 30; n7 = 16. From these equations we easily nd
n1 = 100, n2 = 14, n3 = 84, n4 = 220, n5 = 186, n6 = 46, n7 = 16. (i) n3 + n4 = 304 had read A
but not B; (ii) n6 = 46; (iii) 1000 (n1 + n2 + n3 + n4 + n5 + n6 + n7 ) = 334.
(b) (i) n(A \ B) (ii) n(C \ (A B)) (iii) n( \ (A B C)) = n() n(A B C))
(c) The equality is n1 + n2 + n3 + n4 + n5 + n6 + n7 = (n1 + n3 + n4 + n7 ) + (n1 + n2 + n5 + n7 ) +
(n2 + n3 + n6 + n7 ) (n1 + n7 ) (n3 + n7 ) (n2 + n7 ) + n7 , which is easily veried. The last equality
is a special case of n( \ D) = n() n(D): The number of persons who are in , but not in D, is the
number of persons in all of  minus the number of those who are in D.

Knut Sydster and Peter Hammond 2010


CHAPTER 2 FUNCTIONS OF ONE VARIABLE: INTRODUCTION 3

Chapter 2 Functions of One Variable: Introduction

2.2
2. F (0) = F (3) = 10, F (a + h) F (a) = 10 10 = 0

4. (a) f (1/10) = 10/101, f (0) = 0, f (1/ 2) = 2/3, f ( ) = /(1 + ),
f (2) = 2/5 (b) f (x) = x/(1 + (x)2 ) = x/(1 + x 2 ) = f (x) and moreover, f (1/x) =
(1/x)/[1 + (1/x)2 ] = (1/x) x 2 /[1 + (1/x)2 ] x 2 = x/(1 + x 2 ) = f (x)

6. F (0) = 2, F (3) = 19, F (t + 1) = t 2 + 3

8. (a) b(0) = 0, b(50) = 100/11, b(100) = 200


(b) b(50 + h) b(50) is the additional cost of removing h% more than 50% of the impurities.

10. (a) No. f (2 + 1) = f (3) = 2 32 = 18, whereas f (2) + f (1) = 2 22 + 2 12 = 8 + 2 = 10.


(b)Yes. f (2+1) = f (2)+f (1) = 9 (c) No. f (2+1) = f (3) = 3 1.73, whereas f (2)+f (1) =

2 + 1 2.41.

12. See Figs. 2.2.12(a) and 2.2.12(b).

1 x1 11 1

x x2 1x x

x 1 x

Figure 2.2.12(a). Area = (x + 1)2 = x 2 + 2x + 1 Figure 2.2.12(b) Area = x 2 +1

14. (a) (, 2) (2, ) (b) f (8) = 5


3x + 6
(c) f (x) = = 3 3x + 6 = 3(x 2) 6 = 6, which is absurd.
x2

16. (a) is invalid: |2 + (2)| = 0, whereas |2| + | 2| = 2 + 2 = 4. (b), called the triangle inequality,
|x + y| |x| + |y|, is valid. In fact, if x and y have the same sign, then |x + y| = |x| + |y|. If x and y
have opposite signs, then we see that |x + y| < |x| + |y|. (See Problem 12.4.8 for a generalization.) (c)
is valid: |xy| = |x| |y|. (Look at the 4 different sign combinations of x and y: If x is positive and y is
negative, then xy is negative, |xy| = xy and |x||y| = (x)y = xy, and so on.)  (d) is easily seen to
be valid. (e) is not valid (put x = 1). (f) is easily seen
 to be valid.
 (Note that 4)2 = (2) = 2.)
 
(g) is valid: | 2x| = 2|x|. (h) is valid because |x| |y| |x y| x 2 2|x||y| + y 2
x 2 2xy + y 2 2|x||y| 2xy |x||y| x y, which is true because (c) is valid.

Knut Sydster and Peter Hammond 2010


4 CHAPTER 2 FUNCTIONS OF ONE VARIABLE: INTRODUCTION

2.3
2. See Figs. 2.3.2(a)2.3.2(f).
y y y
4

1 1 1

1 x 1 x 1 2 3 x

Figure 2.3.2(a) Figure 2.3.2(b) Figure 2.3.2(c)

y y y

1 1 1

2 1 1 2 x 1 x 1 x
1

Figure 2.3.2(d) Figure 2.3.2(e) Figure 2.3.2(f)

4. See Figs. 2.3.4(a)(c).


y y y
2 2 2

1 1 1

3 2 1 1 2 3x 1 2 3 4 5 x 3 2 1 1 2 3x
1 1 1
2 2 2

Figure 2.3.4(a) Figure 2.3.4(b) Figure 2.3.4(c)

y
y
y
2 A = (3, 2)
(2, 4)

2 x 1
2
P
B = (5, 4)
2 5 x 1 x

Figure 2.3.6 Figure 2.3.10 Figure 2.3.12

Knut Sydster and Peter Hammond 2010


CHAPTER 2 FUNCTIONS OF ONE VARIABLE: INTRODUCTION 5

6. (5 2)2 + (y 4)2 = 13, or y 2 8y + 12 = 0, with solutions y = 2 and y = 6. Geometric explanation:


The circle with center at (2, 4) and radius 13 intersects the line x = 5 at two points. If the radius were
2, the circle would not intersect the line x = 5. See Fig. 2.3.6.
8. (a) (x 2)2 + (y 3)2 = 16 (b) (x 2)2 + (y 5)2 = 13
10. (x 3)2 + (y 2)2 = (x 5)2 + (y + 4)2 , which reduces to x 3y = 7. See Fig. 2.3.10
12. (x, y) = (1, 0), (0, 1), (1/4, 1/4) are all solutions. See Fig. 2.3.12.
 
14. The condition is: p + r x 2 + y 2 = p + s (x a)2 + y 2 . Cancelling p, then squaring each side and
rearranging yields (r 2 s 2 )(x 2 + y 2 ) = s 2 (a 2 2ax). If r = s, then the markets are separated by the
straight line x = 21 a. Otherwise, completing squares  yields the equation
 [x + as 2 /(r 2 s 2 )]2 + y 2 =
[ars/(r 2 s 2 )]2 . This is a circle with center at as 2 /(r 2 s 2 ), 0 and radius |ars/(r 2 s 2 )|.

2.4
2. (a) f (5) = 0, f (3) = 3, f (2) = 0, f (0) = 2, f (3) = 4, f (4) = 0
(b) Df = [5, 4], Vf = [3, 4]
4. See Figs. 2.4.4(a)(d).

y y y y

1 1 1 1 x

1 1 x 3 1 x 1 x

Figure 2.4.4(a) Figure 2.4.4(b) Figure 2.4.4(c) Figure 2.4.4.4(d)

2.5
2. 0.78 4. See Figs. 2.5.4(a)(c).

y y y
1 4
4
1 2 3 4 5 6 7 8 10 x 3
3 1
2
2 2
1
3
1
4 1 2 3 4 5 x
1 2 3 4 5 1
x

Figure 2.5.4(a) Figure 2.5.4(b) Figure 2.5.4(c)

6. (a) Assume F = aC + b. Then 32 = a 0 + b and 212 = a 100 + b. Therefore a = 180/100 = 9/5


and b = 32, so F = (9/5)C + 32. (b) If X = (9/5)X + 32, then X = 40.

Knut Sydster and Peter Hammond 2010


6 CHAPTER 3 POLYNOMIALS, POWERS, AND EXPONENTIALS

8. L: y = 3x 2; M: y = 43 x + 45 . P : (13/15, 3/5). N : y = 43 x 47 . See Fig. 2.5.8.


10. (a) 75 3P e = 20 + 2P e , and so P e = 11. (b) P e = 90
12. C = 45 y + 100. (The general equation is C = ay + b. Here 900 = a 1000 + b, and a = 80/100 = 4/5,
so b = 100.)
1 1

x0 + h x0
14. The slope is . Multiplying both numerator and denominator by the product (x0 + h)x0 , then
x0 + h x 0
simplifying, yields the given expression.
16. See Figs. 2.5.16(a)(c).

y y y y
L

1 1 1 1

1 x 1 x 1 x 1 x
M
N

Figure 2.5.8 Figure 2.5.16(a) Figure 2.5.16(b) Figure 2.5.16(c)

Chapter 3 Polynomials, Powers, and Exponentials


3.1
2. (a)
x 4 3 2 1 0 1 2

f (x) 2.5 0 1.5 2 1.5 0 2.5

(b) See Fig. 3.1.2(b). (c) f (x) = (1/2)(x + 1)2 + 2. Maximum point (1, 2).
(d) x = 3 and x = 1 (e) f (x) > 0 in the interval (3, 1), while f (x) < 0 for x < 3 and for x > 1.

y
3
2
f (x) = 21 x 2 x + 3
2
1

4 3 21 1 2 3 x
1
2
3
4

Figure 3.1.2(b)

Knut Sydster and Peter Hammond 2010


CHAPTER 3 POLYNOMIALS, POWERS, AND EXPONENTIALS 7

4. (a) x(x + 4). Zeros 0 and 4. (b) Factorization not possible.


No zeros.
(c) 3(x x1 )(x x2 ), where the zeros are x1 = 5 + 15 and x2 = 5 15.
(d) 9(x x1 )(x x2 ), where the zeros are x1 = 1/3 + 5 and x2 = 1/3 5.
(e) (x + 300)(x 100), where the zeros are 300 and 100.
(f) (x + 200)(x 100), where the zeros are 200 and 100.
6. (a) If the other side is y, then 2x + 2y = L, and so y = L/2 x. The area is then A(x) = x(L/2 x) =
Lx/2 x 2 . The square with side L/4 gives the largest area.
(b) Yes; the radius is L/2 so the area is L2 /4 > L2 /16.
8. (a) x = 1, x = 2 (b) (i) x 2 = 9 (x 2 < 0 is impossible), so x = 3
(ii) x 3 = 1 or x 3 = 8, so x = 1 or x = 2
10. y = 2x 2 + x 6. ((1, 3) belongs to the graph if 3 = a + b + c, (0, 6) belongs to the graph if
6 = c, and (3, 15) belongs to the graph if 15 = 9a + 3b + c. It follows that a = 2, b = 1, and c = 6.)
12. (a) (i) 289 290 (ii) 361 377 (b) If B 2 4AC were > 0, then according to formula [3.2] the
equation f (x) = 0 would have two distinct solutions, which is impossible when f (x) 0 for all x. We
nd that A = a12 + a22 + + an2 , B = 2(a1 b1 + a2 b2 + + an bn ), and C = b12 + b22 + + bn2 , so
the conclusion follows.

3.3
2. (a) Integer roots must divide 6. Thus 1, 2, 3, and 6 are the only possible integer solutions. We
nd that 2, 1, 1, 3 all are roots, and since there can be no more than 4 roots in a polynomial equation
of degree 4, we have found them all.
(b) The same possible integer solutions. Only 6 and 1 are integer solutions. (The third root is 1/2.)
(c) Neither 1 nor 1 satises the equation, so there are no integer roots.
(d) First multiply the equation by 4 to have integer coefcients. Then 1, 2, and 4 are seen to be the
only possible integer solutions. In fact, 1, 2, 2 are all solutions.
4. (a) The answer is 2x 2 + 2x + 4 + 3/(x 1), because

(2x 3 + 2x 1) (x 1) = 2x 2 + 2x + 4
2x 2x
3 2

2x 2 + 2x 1
2x 2 2x
4x 1
4x 4
3 remainder
(b) The answer is x2 + 1, because

(x 4 + x 3 + x 2 + x) (x 2 + x) = x 2 + 1
x4 + x3
x2 + x
x2 + x
0 no remainder

Knut Sydster and Peter Hammond 2010


8 CHAPTER 3 POLYNOMIALS, POWERS, AND EXPONENTIALS

(c) The answer is 3x 5 + 6x 3 3x 2 + 12x 12 + (28x 2 36x + 13)/(x 3 2x + 1):

(3x 8 x2 + 1) (x 3 2x + 1) = 3x 5 + 6x 3 3x 2 + 12x 12
3x 8 6x 6 + 3x 5
6x 6 3x 5 + x2 + 1
6x 6 12x 4 + 6x 3

3x 5 + 12x 4 6x 3 + x 2 + 1
3x 5 + 6x 3 3x 2
12x 4 12x 3 + 4x 2 + 1
12x 4 24x 2 + 12x
12x 3 + 28x 2 12x + 1
12x 3 + 24x 12
28x 2 36x + 13 remainder

(d) The answer is x 3 4x 2 + 3x + 1 4x/(x 2 + x + 1), because

(x 5 3x 4 + 1) (x 2 + x + 1) = x 3 4x 2 + 3x + 1
x5 + x4 + x3
4x 4 x 3 +1
4x 4 4x 3 4x 2
3x 3 + 4x 2 +1
3x + 3x + 3x
3 2

x 2 3x + 1
x2 + x + 1
4x remainder

6. (a) p(x) = x(x 2 + x 12) = x(x 3)(x + 4), because x 2 + x 12 = 0 for x = 3 and x = 4. (b)
1, 2, 4, 8 are the only possible integer zeros. By trial and error we nd that q(2) = q(4) =
0, so 2(x 2)(x + 4) = 2x 2 + 4x 16 is a factor for q(x). By polynomial division we nd that
q(x) (2x 2 + 4x 16) = x 1/2, so q(x) = 2(x 2)(x + 4)(x 1/2).

3.4
2. (a) 2.511886 (b) 0.000098 (c) 0.530094
 16
4. 500.16 =
100
50 (Find rst the number that, when raised to the power of 100, yields 50. Then this
number is raised to the power of 16.)

6. (a) 23 = 8, so x = 3/2 (b) 81


1
= 34 , so 3x + 1 = 4, and then x = 5/3.
(c) x 2x + 2 = 2, so x 2x = 0, implying that x = 0 or x = 2.
2 2

8. (a) Multiply by 5K 1/2 to obtain K 1/2 = 15L1/3 . Squaring each side, K = 225L2/3 . (b) abx0b1 = p,
so x0b1 = p/ab. Now raise each side to the power 1/(b 1).
 1/
(c) x = 2a
b
. (Multiply each side by (ax + b)2/3 .) (d) b = 1/ c (1 )a

10. K = 57 315.86 for Y = 100, L = 6, t = 10.

Knut Sydster and Peter Hammond 2010


CHAPTER 3 POLYNOMIALS, POWERS, AND EXPONENTIALS 9

3.5

2. (a) P = 1.22 1.034t million (b) The doubling time t is given by the equation (1.034)t = 2, and we
nd t 20.7 (years).

4. (a) 210 = 1024 (b) f (t) = 2t (c) f (20) = (210 )2 = (1024)2 > 10002 = 1 million.

6. The graph is shown in Fig. 3.5.6.

x 2 1 0 1 2
x2
2 16 2 1 2 16

y
6
5
4
3 y
2 6

y 1
4
2 1 1 2 3 x y = x 2 2x
1
2
2
1 3
1 4 5 4 3 2 1 1 2 x
x
Figure 3.5.6 Figure 3.5.10 Figure 3.5.12


8. If the initial time is t, the doubling time t is given by the equation Aa t+t = 2Aa t , which implies

Aa t a t = 2Aa t , so a t = 2, independent of t.

10. (a) The graph is shown in Fig. 3.5.10.

x 3 2 1 0 1 2 3 4

1 2x 7 3 1 0 1/2 3/4 7/8 15/16

(b) 1 2x gets close to 1 as x becomes very large, and becomes very large negative as x becomes large
negative. (Formally, limx f (x) = 1, limx f (x) = . See Section 6.1.)

12. The graph is drawn in Fig. 3.5.12.

x 10 5 4 3 2 1 0 1 2

x 2 2x 0.1 0.8 1.0 1.1 1.0 0.5 0 2.0 16

14. I (t) = I0 (1/2)t/8 remains after t days.

Knut Sydster and Peter Hammond 2010


10 CHAPTER 4 SINGLE VARIABLE DIFFERENTIATION

3.6
2. The function in (b) is one-to-one and has an inverse: the rule mapping each youngest child alive today to
his/her mother. (Though the youngest child of a mother with several children will have been different at
different dates.) The function in (d) is one-to-one and has an inverse: the rule mapping the surface area to
the volume. The function in (e) is one-to-one and has an inverse: the rule that maps (u, v) to (u 3, v).
The other functions are many-to-one, in general, and so have no inverses.

Chapter 4 Single Variable Differentiation


4.2
2. f  (x) = 6x + 2, f  (0) = 2, f  (2) = 10, f  (3) = 20. The tangent equation is y = 2x 1.
f (x + h) f (x) 1/(x + h) 1/x x (x + h) h 1 1
4. = = = = 2 as h 0,
h h hx(x + h) hx(x + h) x(x + h) x
which proves the implication.
6. (a) f (x + h) f (x) = a(x + h)2 + b(x + h) + c (ax 2 + bx + c) = 2ahx + bh + ah2 , so
[f (x + h) f (x)]/ h = 2ax + b + ah 2ax + b as h 0. Thus f  (x) = 2ax + b. (b) f  (x) = 0
for x = b/2a. The tangent is parallel to the x-axis at the minimum/maximum point.

8. (a) Use [A.12] in Section A.3. (b) and (c) [f (x + h) f (x)]/ h = x + h x)/ h. Multiplying both

numerator and denominator by x + h + x , then using the identity in (a), yields the result. Letting

h 0, the formula follows. (d) x = x 1/2 and 1/ x = x 1/2 .
10. (a) [f (x + h) f (x)]/ h = [(x + h)1/3 x 1/3 ]/ h. Then use the hint.
(b) Follows from (a) by letting h 0.

4.3
2. I is the xed cost, whereas k is the marginal cost, and also the incremental cost of producing one additional
unit.
4. T  (y) = t, so the marginal tax rate is constant.

4.4
2. (a) The following table seems to indicate that the limit is 9.

x 0.9 0.99 0.999 1 1.001 1.01 1.1


x 2 +7x8
x1
8.9 8.99 8.999 9.001 9.01 9.1

*not dened
x 2 + 7x 8
(b) x 2 + 7x 8 = (x 1)(x + 8), so for x = 1, = x + 8 9 as x 1.
x1
4. (a) 22 + 3 2 5 = 5 (b) 1/5 (c) 1 (d) 2 (e) 3x 2 (f) h2
6. (a) 4 (b) 5 (c) 6 (d) 2a + 2 (e) 2a + 2 (f) 4a + 4 8. (a) 1/6 (b) 1/27 (c) n

Knut Sydster and Peter Hammond 2010


CHAPTER 4 SINGLE VARIABLE DIFFERENTIATION 11

4.5
2. (a) 2g  (x) (b) (1/6)g  (x) (c) (1/3)g  (x) 4. (a) 8 r (b) A(b + 1)y b (c) (5/2)A7/2
x a+1
6. Add an arbitrary constant to these answers: (a) (1/3)x 3 (b) x 2 + 3x (c)
a+1

4.6
2. (a) (6/5)x 14x 6 (1/2)x 1/2 (b) 4x(3x 4 x 2 1) (c) 10x 9 + 5x 4 + 4x 3 x 2
3 x 4 + 5x 2 + 18x + 2 4x 2x 2 + 2
4. (a) (b) (c) 2(1+2x)x 3 (d) (e)
2 x( x + 1)2 (x 2 + 2)2 (x + 3)2 (x 2 + 1)2 (x 2 x + 1)2
2(x 3 x 2 + x + 1)
(f)
3x 3 (x + 1)2

6. (a) x = 2 (b) x = 3, x = 0, x = 3 (c) x = 2, x = 2
(d) x = 0, x = 1/2 5/2, x = 1/2 + 5/2
ad bc (2at + b)
8. (a) (b) a(n + 1/2)t n1/2 + nbt n1 (c)
(ct + d)2 (at 2 + bt + c)2
10. Differentiating f (x) f (x) = x gives f  (x) f (x) + f (x) f  (x) = 1, so 2f  (x) f (x) = 1. Hence,
1 1
f  (x) = = .
2f (x) 2 x
12. The Newton quotient of F is

F (x + h) F (x) f (x + h)/g(x + h) f (x)/g(x)


=
h h

Multiplying the numerator and denominator by g(x)g(x+h) yields the expression for the Newton quotient
given in the hint. Letting h 0 yields the desired result.

4.7
2. dy/dx = ax a1 ax a1 and d 2 y/dx 2 = a(a 1)x a2 + a(a + 1)x a2
4. g  (t) = (t 2 2t)/(t 1)2 , so g  (t) = [(2t 2)(t 1)2 (t 2 2t)2(t 1)]/(t 1)4 = 2 for t = 2.
6. For n = 1, the formula is true. Suppose it is true for n = k, so that y = x k y (k) = k! . Then
(d k+1 /dx k+1 )x k+1 = (d k /dx k )(d/dx)(x k+1 ) = (d k /dx k )(k +1)x k = (k +1)(d k /dx k )x k = (k +1)k!,
by the induction hypothesis. This is equal to (k + 1)!, so the given formula is also true for n = k + 1. By
induction, the formula is true for all n.

Knut Sydster and Peter Hammond 2010


12 CHAPTER 5 MORE ON DIFFERENTIATION

Chapter 5 More on Differentiation


5.1
2. (a) 3(2x + 1)2 2 = 6(2x + 1)2 (b) 5(1 x)4 (c) 2(x 2 2x + 2)(2x 2)
(d) (x + 1)4 (4x 1)/x 2 (e) 21(3x 4)8 (f) 2(4x + 3)(2x 2 + 3x 4)3

b(a + 1) at + b a
4. (a) 6at (at 2 + 1)4 (b) na(at + b)n1 (c)
nt 2 nt
6. dx/dt = Ar(Ap + B)r1 (2at + b)
8. Putting y = 1/(g(x))n implies that y  = [0(g(x))n 1n(g(x))n1 g  (x)]/(g(x))2n = n(g(x))n1 g  (x).

k+1
10. For a = 1, the formula is true. Suppose the formula is true for a = k. Consider y = g(x) =

k
g(x) g(x). The product rule for differentiation and the induction hypothesis yield

k1 
k
k
y  = k g(x) g (x) g(x) + g(x) g  (x) = (k + 1) g(x) g  (x), which is the given formula for
a = k + 1. By induction, the formula is true for all natural numbers a.

5.2
2. (a) dY /dt = (dY /dV )(dV /dt) = (3)5(V + 1)4 t 2 = 15(t 3 /3 + 1)4 t 2
(b) dK/dt = (dK/dL)(dL/dt) = AaLa1 b = Aab(bt + c)a1
4. dY /dt = (dY /dK) (dK/dt) = Y  (K(t0 ))K  (t0 )
dx 1 a
6. x = b ap c = b u, with u = ap c. Then = u = . (The restriction
dp 2 u 2 ap c
should be p > c/a, for x to be differentiable.)
 
8. (a) b(t) is the total fuel consumption after t hours. (b) b (t) = B  s(t) s  (t). So the rate of fuel
consumption per hour is equal to the rate per kilometer multiplied by the speed in kph.
10. f (f (x)) = f (3x + 7) = 3(3x + 7) + 7 = 9x + 28. Then 9x + 28 = 100 for x = 8.
12. dC/dx = q(25 21 x)1/2 14. p  (x) = 2(x a)q(x) + (x a)2 q  (x), so p  (a) = 0.
  
16. F  (x) = f  x n g(x) nx n1 g(x) + x n g  (x)

5.3
2x + 3y
2. By implicit differentiation, 4x + 6y + 6xy  + 2yy  = 0. So y  = . In particular, y  = 8/5
3x + y
at the point (1, 2).
4. (a) 2x + 2yy  = 0, and solve for y  to get y  = x/y.

(b) 1/2 x + y  /2 y = 0, and solve for y  to get y  = y/x.
(c) 4x 3 4y 3 y  = 2xy 3 + x 2 3y 2 y  , and so y  = 2x(2x 2 y 3 )/y 2 (3x 2 + 4y).
6. (a) No relationship. (b) f  (x0 ) = g  (x0 )
8. Differentiating y n = x m w.r.t. x yields ny n1 y  = mx m1 , so y  = mx m1 /ny n1 = mx m1 /n(x m/n )n1 =
(m/n)x m1(m/n)(n1) = (m/n)x (m/n)1 .

Knut Sydster and Peter Hammond 2010


CHAPTER 5 MORE ON DIFFERENTIATION 13

5.4
2. f (x) f (0) + f  (0)x = 1
9 10
27 x

4. F (1) = A, F  (K) = AK 1 , so F  (1) = A. Then F (K) F (1) + F  (1)(K 1)=


A + A(K 1) = A(1 + A(K 1)).
6. (a) (i) y = 0.61, dy = 0.6 (ii) y = 0.0601, dy = 0.06
(b) (i) y = 0.011494, dy = 0.011111 (ii) y = 0.001124, dy = 0.001111
(c) (i) y = 0.012461, dy = 0.0125 (ii) y = 0.002498, dy = 0.0025
 
8. g(0) = A 1 and g  () = Aa/(1 + b) (1 + )[a/(1+b)]1 , so g  (0) = Aa/(1 + b). Hence, g()
aA
g(0) + g  (0) = A 1 +
1+b

5.5
2. Follows from formula [5.9] with f = U , a = y, x = y + M s.
4. We nd x(0) = 2[x(0)]2 = 2 1 = 2. Differentiating the expression for x(t) yields x(t) = x(t) + t x(t) +
4[x(t)]x(t), and so x(0) = x(0)+4[x(0)]x(0) = 1+412 = 9. Hence, x(t) x(0)+ x(0)t + 21 x(0)t 2 =
1 + 2t + 29 t 2 .
(px p1 qx q1 )(x p + x q ) (x p x q )(px p1 + qx q1 ) 2(p q)x p+q1
6. h (x) = = , so h (1) =
(x p + x q )2 (x p + x q )2

2 (p q). Since h(1) = 0, h(x) h(1) + h (1)(x 1) = 2 (p q)(x 1).
1 1

5.6
2. ElK T = 1.06. A 1% increase in expenditure on road building leads to an increase in the trafc volume
of approx. 1.06 %.
4. (a) Elx f (x) = 0 (b) Elx f (x) = x/(x + 1) (c) Elx f (x) = 20x 2 /(1 x 2 )
  f (x)Elx f (x)
6. Elx Af (x) = Elx f (x), Elx A + f (x) =
A + f (x)
1 + 2x 30x 3 2x 2
8. (a) 3 (b) (c) 3 (d) Elx 5x 2 = 2, so Elx Elx 5x 2 = 0 (e)
1+x x +1 1 + x2

x1 x Elx x x 5 Elx x 5 x 5x 5
(f) Elx = Elx (x 1) Elx (x 5 + 1) = 5 = 5 (Some answers
x +1
5 x1 x +1 x1 x +1
are easier if the results of Problem 6 are used as well.)
10. (a) 6Elx y = 5, so Elx y = 5/6 (b) Using rules (c) and (b) in Problem 7, we obtain Elx y Elx x =
Elx (x + 1)a + Elx (y 1)b . Here Elx x = 1, whereas

Elx (x + 1)a = [x/(x + 1)a ]a(x + 1)a1 = ax/(x + 1)


Elx (y 1)b = [x/(y 1)b ]b(y 1)b1 y  = [by/(y 1)]Elx y

Thus, Elx y 1 = ax/(x + 1) + [by/(y 1)]Elx y. Solving for Elx y yields


(ax + x + 1)(y 1)
Elx y =
(y by 1)(x + 1)

Knut Sydster and Peter Hammond 2010


14 CHAPTER 6 LIMITS, CONTINUITY, AND SERIES

Chapter 6 Limits, Continuity, and Series


6.1

x3 (1/x) (3/x 2 ) 2 + 3x 3 + 2/x
2. (a) 2 = (b) = 3 (c) a 2
x +1 1 + (1/x 2 ) x x1 1 1/x x

4. limx fi (x) = for i = 1, 2, 3; limx f4 (x) = 0. Then:


(a) (b) 0 (c) (d) 1 (e) 0 (f) (g) 1 (h)

6. y = Ax + A(b c) + d is an asymptote as x .

6.2
2. (a) None of the six functions is continuous at a. (b) Only in (i) does f have a limit: limxa f (x) = A
(c) (i) limxa f (x) = limxa + f (x) = limxa f (x) = A,
(ii) limxa f (x) = f (a), limxa + f (x) = A; (iii) limxa f (x) = limxa + f (x) = ;
(iv) limxa f (x) = , limxa + f (x) = f (a); (v) limxa f (x) = , limxa + f (x) = f (a);
(vi) limxa f (x) = A, limxa + f (x) = B. (d) Only the function in (ii) is left-continuous at x = a.
The functions in (iv), (v), and (vi) are right-continuous at x = a. (e) (v) limx f (x) = A;
(vi) limx f (x) does not exist.

x  = 1. (c) Continuous
4. (a) Continuous for all x. (b) Continuous for all for all x < 2. (d) Continuous
for all x. (e) Continuous for all x where x  = 3 1 and x  = 3 1. (f) Continuous for all x
where x 1 or x > 1. (g) Continuous for all x > 0. (h) Continuous for all x  = 0. (i) Continuous
for all x > 0.

6. See Fig. 6.2.6; y is discontinuous at x = a. In the case where y is the nearest point on the ground, it
would be a continuous function.

a x
Figure 6.2.6

8. Because limx0 f (x) = 2 and limx2+ f (x) = 3, we need f (0) = 2 and f (2) = 3. Because f is
linear on [0, 2], put f (x) = (5/2)x 2 for x [0, 2].

6.3
2. f  (0+ ) = 1, f  (0 ) = 1. At x = 0, f is continuous, but not differentiable.

Knut Sydster and Peter Hammond 2010


CHAPTER 6 LIMITS, CONTINUITY, AND SERIES 15

4. The tax function is


0.15x for x [0, 20 250]
t (x) = 0.28x 2 632.50 for x (20 250, 49 300]

0.31x 4 111.50 for x (49 300, )
In particular, t (22 000) = 3527.50 and t (50 000) = 11388.50.

6.4

3n 3 3 3 2
2. (a) Converges to 5. (b) Diverges (to ). (c) = =
2n2 1 2 1/n2 n 2 2

6.5
2. (a) Geometric series with quotient 1/8. Its sum is 8/(1 1/8) = 64/7. (b) Geometric with quotient
3. It diverges. (c) Geometric, with sum 21/3 /(1 21/3 ). (d) Not geometric. (One can show that
the series is convergent with sum ln 2.)
4. Geometric series with quotient (1 + p/100)1 . Its sum is b/[1 (1 + p/100)1 ] = b(1 + 100/p).
6. The general term does not approach 0 as n in any of these three cases, so each of the series is
divergent.
8. sn = n/(n + 1) = 1/(1 + 1/n) 1 as n . The innite series converges to 1.

6.6
1 (1.06)5
2. Offer (a) is best. The second offer has present value 4600 20 540.
1 (1.06)1
12 000 1.115
4. Schedule (b) has present value [1 (1.115)8 ] 67 644.42.
0.115
(1.115)12 1
The present value of the contract in (c) is 22, 000 + 7000 = 66, 384.08. Thus the contract
0.115(1.115)12
in (c) is the best in any case. When the interest rate becomes 12.5 %, contracts (b) and (c) have present
values equal to 65907.61 and 64374.33, respectively.
6. This is a geometric series with rst term a = D/(1 + r) and quotient k = (1 + g)/(1 + r). It converges
a D/(1 + r) D
iff k < 1, i.e. iff 1 + g < 1 + r, or g < r. The sum is = = .
1k 1 (1 + g)/(1 + r) r g

6.7
2. |(x + 1)3 1| = |x 3 + 3x 2 + 3x + 1 1| |x|3 + 3|x|2 + 3|x| |x| + 3 + 3|x| + 3|x| = 7|x|. Let  > 0.
Choose = /7. Then according to [6.24], limx0 (x + 1)3 = 1. Because f (0) = 1, f is continuous at
x = 0.
4x 2 100
4. (a) 4x 2 100 = 4(x 2 25) = 4(x + 5)(x 5), so for x = 5, = 4(x + 5). Hence, for x = 5,
x5
 4x 2 100 
 
 40 = |4(x + 5) 40| = 4|x 5|. Let  > 0. Choose = /4. According to [6.24], the
x5
conclusion follows. (b) |x 2 2 | = |(x + )(x )|, so for x  = , |(x 2 2 )/(x + ) + 2 | =
|x + 2| = |x + |. Let  > 0. Choose = . According to [6.24], the conclusion follows.

Knut Sydster and Peter Hammond 2010


16 CHAPTER 7 IMPLICATIONS OF CONTINUITY AND DIFFERENTIABILITY

Chapter 7 Implications of Continuity and Differentiability


7.1
2. For x  = 0, we have f (x) = x 3 + ax 2 + bx + c = x 3 (1 + a/x + b/x 2 + c/x 3 ), which shows that f (x)
is (large) positive if x is sufciently large, and that f (x) is (large) negative if x is sufciently large and
negative. Hence Theorem 7.2 implies that f (x) = 0 has a solution. A similar argument applies to the
general case, when the order n of the polynomial is odd. If n is even, there may be no real roots. For
instance, x 2 + 1 = 0 has no real roots.
4. Recall that, arbitrarily close to any given real number, there are rational as well as irrational numbers.
The function f is continuous at a = 0, because |f (x) f (0)| = |f (x) 0| = |f (x)| |x| for any
x, so f (x) f (0) as x 0. If a  = 0 is rational, then |f (x) f (a)| = |f (x)|, which is equal to |x|
when x is irrational. But if a  = 0 is irrational, then |f (x) f (a)| = |f (a)| whenever x is rational. In
either case, f (x) does not approach 0 as x approaches a. It follows that f is discontinuous for all x  = 0.
We prove that g is discontinuous by using the denition of continuity. Let a be an arbitrary number
and choose = 21 . For each positive number , there is a rational number x1 and an irrational number x2 in
the interval (a, a+). Then g(x1 ) = 1 and g(x2 ) = 0. If a is rational, then |g(x2 )g(a)| = |01| = 1,
and if a is irrational, then |g(x1 ) g(a)| = |1 0| = 1. In both cases there is a number x arbitrarily
close to a for which |g(x) g(a)| = 1 > . Hence g is discontinuous at a.

7.2
2. (a) No, because f (x) can get arbitrarily close to 1, by letting x be sufciently close to 1. But there is no
value of x for which f (x) = 1. Similarly, there is no minimum, because f (x) can get arbitrarily close
to 1 by letting x be sufciently close to 1, yet there is no value of x for which f (x) = 1.
(b) No, not at x = 1 and x = 1. y
1
yx1
y y
y  x 2 3 3
y  x  3
1
1
2 1 2 x
1
1

1 1 x 1 2 3 4 5 x
Figure 7.3.2(a) Figure 7.3.2(b) Figure 7.3.2(c)

7.3
2. In (a), f is not differentiable at x = 0; in (b), f is not differentiable at x = 3; in (c), f is not even dened
at x = 1. See Fig. 7.3.2(a)(c).
4. There is at least one point where you must be heading in the direction of the straight line joining A to B
(even if that straight line hits the shore).

7.4

2. (a) 25 = 3(1 2/27)1/3 3(1 13 27
2
9 272 ) 2.924
1 4
3

(b) 33 = 2(1 + 1/32) 2(1 + 532
5 1/5 1
25 322 ) 2.0125
2 1

Knut Sydster and Peter Hammond 2010


CHAPTER 7 IMPLICATIONS OF CONTINUITY AND DIFFERENTIABILITY 17

4. g(0) = g(x) = 0 follows immediately from [1] and [2]. Because f is thrice continuously differentiable,
S(x) and g(x) are differentiable in (0, x). The formula for g  (t) follows easily, and the condition g  (c) = 0
yields S(x) = f  (c). The conclusion follows.

7.5
x 4 4x 3 + 6x 2 8x + 8 0 4x 3 12x 2 + 12x 8 0
2. (a) lim = = lim = =
x2 x 3 3x 2 + 4 0 x2 3x 2 6x 0
12x 2 24x + 12 12
lim = =2
x2 6x 6 6
2(1 + x) 2 x
1/2
0 (1 + x)1/2 1 0
(b) lim = = lim 1/2
= =
x0 2(1 + x + x 2 )1/2 2 x 0 x0 (1 + 2x)(1 + x + x )
2 1 0
21 (1 + x)3/2 1
lim =
x0 2(1 + x + x 2 )1/2 + (1 + 2x)2 ( 21 )(1 + x + x 2 )3/2 3

1 (1 + v ) 0 (1 + v ) 1 v 1
4. G = lim+ = = lim+ . If = 1, then
v0 v 0 v0 1
G = . If > 1, then G = 0, and if < 1, then G = .
f (x) f (1/t) 0 f  (1/t)(1/t 2 ) f  (1/t) f  (x)
6. lim = lim+ = = lim+  2
= lim+  = lim 
x g(x) t0 g(1/t) 0 t0 g (1/t)(1/t ) t0 g (1/t) x g (x)

8. f (x) 1/g(x) 0 1/(g(x))2 g  (x)


L = lim = lim = = lim
xa g(x) xa 1/f (x) 0 xa 1/(f (x))2 f  (x)

(f (x))2 g  (x) g  (x) 1


= lim = L2
lim = L2 lim 
xa (g(x))2 f  (x) xa f  (x) xa f (x)/g  (x)

The conclusion follows. (Here, we have ignored problems with division by 0, when either f  (x) or g  (x)
tends to 0 as x tends to a.)

7.6
2. p = (157.8/D)10/3

4. f (x) = x 2 = (x)2 is not one-to-one on (, ), and therefore has no inverse. On [0, ), f is



strictly increasing and has the inverse g(x) = x.

6. f  (x) = 7x 6 + 25x 4 + 2 > 0 for all x, so f has an inverse g. g  (2) = 1/f  (0) = 1/2.

8. (a) See Fig. 7.6.8(a). (b) Triangles OBA and OBC in Fig. 7.6.8(b) are congruent.

10. See Fig. 7.6.10. (This example shows that the commonly seen statement: if the inverse function exists, the
original and the inverse function must both be monotonic is wrong. This claim is correct for continuous
functions, however.)
   
12. Differentiating f (g(x))
 = x yields () f  (g(x) g  (x) = 1, so g  (x) = 1/f  (g(x) . Differentiating
() yields f  g(x) g  (x)g  (x) + f  g(x) g  (x) = 0. Solving for g  (x) gives
  2       3
g  (x) = f  g(x) g  (x) /f  (g(x) = f  g(x) / f  g(x) . The conclusions follow.

Knut Sydster and Peter Hammond 2010


18 CHAPTER 8 EXPONENTIAL AND LOGARITHMIC FUNCTIONS

y y y
y=x y=x
C = (b, a)
D
(3, 5)
B
(1, 3) (5, 3)

A = (a, b)
(3, 1)

x O E x x

Figure 7.6.8(a) Figure 7.6.8(b) Figure 7.6.10

14. f  (x) = 21 (x+1)1/2 + 21 (x1)



1/2 > 0 for all x > 1. Hence, f is strictly increasing in [1, ), and has an

inverse g. Because f (1) = 2 and f (x) as x , the range of f is [ 2, ). To nd a formula

for g, let (1) y = x + 1 + x 1. Interchanging
x and y, we have (2) x = y + 1 + y 1. The

next step is to solve (2) for y, with x [ 2, ) and y [1, ). We obtain x = y + 1 + y 1

x y + 1 = y 1  (x y + 1 )2 = y 1 x 2 2x y + 1 + y + 1 = y 1

2x y + 1 = x 2 + 2  4x 2 (y + 1) = x 4 + 4x 2 + 4 y = 41 x 2 + 1/x 2 .

Thus, x = y + 1 + y 1 with y 1 implies y = x /4 + 1/x , x 2. We prove the converse by
2 2

substitution:
   
y + 1 + y 1 = x4 + x12 + 1 + x4 + x12 1 = x +4x
2 +4 x 4 4x 2 +4
= x 2x+2 + x 2x2 = x.
2 2 4 2 2
2 + 4x 2
4x
We conclude that g(x) = 41 x 2 + 1/x 2 , x [ 2, ) is the required inverse function.

Chapter 8 Exponential and Logarithmic Functions


8.1
et et
(b) 21 (et/2 et/2 ) (c)
x +x
(d) z2 ez (ez 1)2/3
x 3 3
2. (a) ee ex = ee
(et + et )2

4. p  (x) = kcecx , p  (x) = kc2 ecx . The graph is shown in Fig. 8.1.4.
y

2
y = 21 (ex + ex )
1 y = 21 (ex ex )
p
-3 -2 -1 1 2 3 x
a+k
-1
p(x) = a + k(1 ecx )
-2
a
-3
x

Figure 8.1.4 Figure 8.1.6

Knut Sydster and Peter Hammond 2010


CHAPTER 8 EXPONENTIAL AND LOGARITHMIC FUNCTIONS 19

6. The graphs are in Fig. 8.1.6. Verifying the identities is straightforward. We prove only equality (a):
cosh x cosh y + sinh x sinh y = 21 (ex + ex ) 21 (ey + ey ) + 21 (ex ex ) 21 (ey ey ) =
1 x+y
4 (e + exy + ex+y + exy + ex+y exy ex+y + exy ) = 21 (ex+y + exy ) = cosh(x + y)

8. f (z + x) = a z+x = a z a x = f (z)f (x). Differentiating w.r.t. z while x is xed gives f  (z + x) =


f  (z)f (x). Putting z = 0 yields f  (x) = f  (0)f (x), which is [8.2].

8.2
2. (a) ln 3x = x ln 3 = ln 8, so x = ln 8/ ln 3. (b) x = e3 (c) x 2
4x + 5 = 1, orx 2 4x + 4 = 0,
so (x 2)2 = 0. Thus x = 2. (d) x(x 2) = 1, so x = 1 2 or x = 1 + 2. (e) x = 0 or

ln(x + 3) = 0, so x = 0 or x = 2. (f) x 5 = 1, so x = 36.
 
4. (a) t = (ln x b)/a (b) t = (ln 2)/a (c) t = ln(8/ 2 ) = 25 ln 2 21 ln

6. (a) True ( e 22.5, e 23.1) (b) True because (a) gives ( e )1/e < (e )1/e .

8. (a) Wrong. (Put A = B = C = 1.) (b) Correct by rule [8.7] (b). (c) Correct. (Use [8.7] (b) twice.)
(d) Wrong. (If A = e and p = 2, then the equality becomes 0 = ln 2.) (e) Correct by [8.7](c).
(f) Wrong. (Put A = 2, B = C = 1.)
ln x 1
10. (a) 1/(x + 1) (b) 1/x (c) ln x + 1 (d)
(ln x)2
12. (a) (i) y = x 1 (ii) y = 2x 1 ln 2 (iii) y = x/e
(b) (i) y = x (ii) y = 2ex e (iii) y = e2 x 4e2

14. (a) Let f (x) = ex (1 + x + x 2 /2). Then f (0) = 0 and f  (x) = ex (1 + x), which is positive for
all x > 0, as shown in the problem. Hence f (x) > 0 for all x > 0, and the inequality follows. (b)
Let f (x) = ln(1 + x) 21 x. Then f (0) = 0 and f  (x) = 1/(x + 1) 21 = (1 x)/2(x + 1) > 0
in (0, 1), so ln(1 + x) > 21 x in (0, 1). To prove the other inequality, let g(x) = x ln(1 + x). Then
g(0) = 0 and g  (x) = 1 1/(x + 1) = x/(1 + x) > 0 in (0, 1), so x > ln(1 + x) in (0, 1). (c) Let
f (t) = ln[(1 + t)/(1 t)] 2t. Then f (0) = 0 and f  (t) = 2/[(1 + t)(1 t)] 2 = 2t 2 /(1 t 2 ) > 0
in (0, 1), so the required inequality follows.

16. (a) x x = 0 (b) 4 ln x x (c) x 2 /y 2 18. (a) x (b) 1/ ln x


(c) x ln a

ln x + 2
20. (a) ln y = x ln x implies y  /y = 21 x 1/2 ln x + x 1/2 so y  = x x
2 x
(b) 21 ( x )x (ln x + 1) (c) Differentiate ln ln y = x ln x + ln ln x to get

1
y  /(y ln y) = ln x + 1 + 1/(x ln x) and so y  = x x +x (ln x)2 + ln x +
x

x
22. First take logarithms to obtain the expression:
ln a + (v/)[ ln N + ln K ln(N + bK ) = B (v/) ln(N + bK )
for ln F (), where B does not depend on . Differentiation yields:

 2 N ln N + bK ln K
F () = v F () ln(N + bK )

N + bK
8.3
2. (a) x = 4 (b) x = e (c) x = 1/27 (d) x = 1050 and x = 1050

Knut Sydster and Peter Hammond 2010


20 CHAPTER 8 EXPONENTIAL AND LOGARITHMIC FUNCTIONS

4. (a) ex+14/x = e1 , so x + 1 4/x = 1, hence x = 2. (b) Putting u = ln(x + e), the equation becomes
u3 4u2 u + 4 = 0, so (u 4)(u2 1) = 0, implying that u = 1 or 4. Because x = eu e, one
has x = 0, x = e + 1/e, or x = e4 e.

6. (a) 1 (b) Does not exist. (c) 4/5 (d) 1 (x 1/x = (eln x )1/x = eln x/x , and [8.19].) (e) 0. (x ln x =
ln x/(1/x), and then lHpitals rule.) (f) 1. (x x = (eln x )x = ex ln x , and then use (e).)
1
8. (a) xex x + x 2 + 21 x 3 (b) e2x 1 + 2x + 2x 2 +
4 3
3x (c) x 2 + e 2 x

1 + 2 x + 8 x + 48 x (d) ex + 1 2 + 4 2x + 32 2x 2 + 384 (7 2)x 3
1 9 2 1 3 1 3 1

10. For x  = 0, one has f  (x) = x 3 2e1/x , f  (x) = x 6 (6x 2 + 4)e1/x , and therefore f  (x) =
2 2

x 9 (24x 4 36x 2 + 8)e1/x . Hence, f (k) = x 3k pk (x)e1/x is correct for k = 1, 2, 3. The general
2 2

formula is proved by induction. (Actually, all we need in the following is the fact that pk (x) is a polynomial.
Its degree is of no importance.) Now, f  (0) = limx0 [f (x) f (0)]/x = limx0 e1/x /x = 0
2


because, in general, () e1/x /x p 0 as x 0 for any natural number p. (Substitute x = 1/ t
2

and use [8.18].) The general case is proved by induction: Suppose that f (k) (0) = 0 for an arbitrary
natural number k. Then, by the denition of the derivative, f (k+1) (0) = limx0 [f (k) (x) f (k) (0)]/x =
limx0 x 3k pk (x)e1/x /x = limx0 x 3k1 pk (x)e1/x = 0, using () again. (Note that pk (x), as a
2 2

polynomial, approaches a constant as x 0.)

8.4
2. (a) S(t) = S0 eat (b) t = (ln 2)/a is the time it takes for sales to halve.

4. k = 0.1 ln(705/641) 0.0095. P (15) 739, P (40) 938.

6. (a) In 1950 there were about 276 thousand. In the next 10 years the number increased by 155 thousand.
(b) y 479.36 as t . The graph is sketched in Fig. 8.4.6.

lny
Tractors (in 1000)
8

500
6
400

300 4

200
2
100

0
0 5 10 15 20 t 2 4 6 8 lnx
(1950) (1960) (1970)

Figure 8.4.6 Figure 8.4.10

8. h = f  K/f 2 = rf (1 f/K)K/f 2 = r(K/f 1) = rh. Hence h = Aert for some constant


A. But then 1 + K/f = Aert , and solving for f yields [8.24].

Knut Sydster and Peter Hammond 2010


CHAPTER 9 SINGLE-VARIABLE OPTIMIZATION 21

10. (a) See Fig. 8.4.10 and the following table:

ln x 3.00 3.69 4.09 4.38 4.61 4.79 4.94 5.08 5.19 5.30

ln y 7.44 6.31 5.44 4.79 4.32 4.09 3.81 3.56 3.22 3.00

(b) ln y = ln A+a ln x. We see from the graph in Fig. 8.4.10 that a 2. Assuming that the graph passes
through (say) (ln x, ln y) = (5.19, 3.22), we obtain 3.22 = ln A + (2)5.19, implying that ln A = 13.6,
so A = e13.6 806 129. The required formula is y = 806 129x 2 .

ln y2 ln y1
12. (a) y = 2.81x 0.83 (b) a = , A = y1 x1a 14. Take ln of each side and solve for t0 .
ln x2 ln x1

8.5
2. (a) A = 4, k = 0.25. The price after 5 years is f (5) = 4e1.25 14. (b) Price controls are needed after
6 years. The doubling time before price controls is 2.8 years, after price controls it is 7.3 years.

Chapter 9 Single-Variable Optimization


9.2

8(2 3x)(2 + 3x)
2. h (x)
= . Note that h(x) 0 as x . The function has a maximum at
(3x 2 + 4)2
x = 2 3/3 and a minimum at x = 2 3/3, with h(2 3/3) = 2 3/3 and h(2 3/3) = 2 3/3.
4. (a) f  (x) = [4x(x 4 + 1) 2x 2 4x 3 ]/(x 4 + 1)2 , then simplify and factor.
(b) f has maximum 1 at x = 1, because f (x) increases in [0, 1] and decreases in [1, ). We see that
f (x) = f (x) for all x, so f (x) is symmetric about the y-axis. On (, ), f has maximum 1 at
x = 1 and at x = 1.
6. (a) f  (x) = 48x(3x 2 + 4)2 , so f  (x) > 0 for x < 0 and f  (x) < 0 for x > 0. Hence, f has a
maximum at x = 0. (b) g  (x) = 2(x 2), so g  (x) > 0 for x < 2 and g  (x) < 0 for x > 2. Hence,
g has a maximum at x = 2. (c) h (x) = 20(x + 2)3 , so h (x) < 0 for x < 2 and h (x) > 0 for
x > 2. Hence, h has a minimum at x = 2. (d) F  (x) = 4x/(2 + x 2 )2 , so F  (x) < 0 for x < 0 and

F  (x) > 0 for x > 0. Hence, F has a minimum at x = 0. (e) G (x) = 1/2 1 x > 0 for x < 1, so G
has a maximum at x = 1. (f) H  (x) = 4x 3 /(1 + x 4 )2 , so H  (x) > 0 for x [1, 0), and H  (x) < 0
for x (0, 1]. Hence, H has a maximum at x = 0, and minima at x = 1.
8. Here d  (x) = 2(x a1 ) + 2(x a2 ) + + 2(x an ) = 2[nx (a1 + a2 + + an )]. So d  (x) = 0
for x = x, where x = n1 (a1 + a2 + + an ), the arithmetic mean of a1 , a2 , . . . , an . If x > x, then
d  (x) > 0, and if x < x, then d  (x) < 0. We conclude that x minimizes d(x).

9.3
2. In all cases the maximum and minimum exist by the extreme value theorem. Follow the recipe in [9.5].
(a) f (x) is strictly decreasing. Maximum 1 at x = 0, minimum 7 at x = 3.

Knut Sydster and Peter Hammond 2010


22 CHAPTER 9 SINGLE-VARIABLE OPTIMIZATION

(b) Maximum 10 at x = 1 and x = 2. Minimum 6 at x = 1. (f  (x) = 3x 2 3 = 0 at x = 1.)


(c) Maximum 2.5 at x = 1/2 and x = 2. Minimum 2 at x =1. (f(x) = x+1/x and f  (x) = 11/x 2 = 0
at x = 1.) (d) Maximum 4 at x = 1. Minimum 6 3 at 3. (f (x) = 5x (x 2 3).)
2

(e) Maximum 4.5 109 at x = 3000. Minimum 0 at x = 0.


(f  (x) = 3(x 2 3000x + 2 106 = 3(x 500)(x 2000).)
4. (a) Total costs when there are 61, 70, and 80 passengers are: $ 48,190; $ 49,000; $ 48,000.
(b) C(x) = (60 + x)(800 10x) = 48, 000 + 200x 10x 2 , x [0, 20] (c) Maximum cost is with 70
travelers (x = 10).

6. h (t) = 1/2 t 21 = (1 t)/2 t. We see that h (t) 0 in [0, 1] and h (t) 0 in [1, ), so t = 1
maximizes h(t). The plant is highest after 1 month, when it is 1/2 meter tall.
8. Because A (Q) = [C  (Q)Q C(Q)]/Q2 , the result follows from Example 4.20 in Section 4.6.
 1/b
10. A (Q) = a(b 1)Qb2 c/Q2 , so there is a minimum at Q = c/a(b 1) .

9.4
2. (a) No local extreme points.
(b) Local maximum 10 at x = 1. Local minimum 6 at x = 1.
(c) Local maximum 2 at x = 1.Local minimum 2 at x =
1.
(d) Local maximum 6 3 at x = 3. Local minimum 6 3 at x = 3.
(e) No local maximum point. Local minimum 0.5 at x = 3.
(f) Local maximum 2 at x = 2. Local minimum 2 at x = 0.
4. (a) (a) We nd f  (t) = 0.05(t + 5)(35 t)et . Obviously, f  (t) > 0 for t < 35 and f  (t) < 0 for
t > 35, so t = 35 maximizes f (with f (35) 278). (b) f (t) 0 as t . The graph is shown in
Fig. 9.4.4.
y

10

y
5

200
2
3 2 1 1 2 2 3 x
100

5
30 60 90 t

Figure 9.4.4 Figure 9.4.8

6. (a) a > 0, b = 0 (f  (0) = 0 b = 0. f  (0) 0 a 0. If a = b = 0, then f (x) = x 3 + c


has no loc. min. at x = 0.) (b) a = 6, b = 9. (f  (1) = 0 and f  (3) = 0 give 3 + 2a + b = 0 and
27 + 6a + b = 0.)
2 x2  (x) = 2x 12x 12 ; x =
3
8. (a) f  (x) = ; f 2 is a local maximum point with
(x + 3x +
2
2)
2 (x + 3x + 2)
2 3

f ( 2) 0.17; x = 2 is a local minimum point with f ( 2) 5.83. (b) There are no global
extreme points.
The graph is shown in Fig. 9.4.8.
(c) ex = 2 and so x = 21 ln 2 gives a global maximum, but g has no global minimum.

Knut Sydster and Peter Hammond 2010


CHAPTER 10 INTEGRATION 23

10. f (x) = x 3 + ax + b as x , and f (x) as x . Thus f (x) has at least one real
root. We have f  (x) = 3x 2 + a. Thus, for a 0, f  (x) > 0 for all x  = 0, so f is strictly increasing, and
there is only one real root. Note that for a 0, 4a 3 + 27b2 0. Assume next that a < 0. Then f  (x) = 0

for x = a/3 = p, where p = a/3 > 0. Then f has a local maximum at ( p, b + 2p p)

and a local minimum at ( p, b 2p p). If one of the local extreme values is 0, the equation has a
double root, and this is the case iff 4p 3 = b2 , that is, iff 4a 3 + 27b2 = 0. The equation has three real
roots iff the local maximum value is positive and the local minimum value is negative. This occurs iff

|b| < 2p p or iff b2 < 4p3 or iff 4a 3 + 27b2 < 0.

9.5
2. (a) f  (x) = 3(x 1)(x + 2), f  (x) = 6x + 3 (b) Stationary points: x = 2 and x = 1. f increases
in (, 2) and in (1, ). (c) x = 1/2 is the only inection point.
4. x = 2 and x = 4 are minimum points, whereas x = 2 is a maximum point. Moreover, x = 0, x = 1,
x = 3, and x = 5 are inection points.
6. a = 2/5, b = 3/5 (f (1) = 1 gives a + b = 1. Moreover, f  (x) = 3ax 2 + 2bx and f  (x) =
6ax + 2b, so f  (1/2) = 0 yields 3a + 2b = 0.)
8. (Q) is stationary at Q = (P /ab)1/(b1) . Moreover,  (Q) = ab(b 1)Qb2 < 0 for all Q > 0, so
this is a maximum point.

9.6
2. a 0, b arbitrary.
      
4. (a) Since u(c) is concave, T1 Tt=1 u(ct ) u T1 Tt=1 ct u T1 Tt=1 yt = u(y), where y =
1 T
t=1 yt . Thus y is optimal. (For ct = y, t = 1, 2, . . . , T , this holds with equality.)
T  
(b) Put t = (1 + r)t / Tt=1 (1 + r)t . Then Tt=1 t = 1 and  
   
max Tt=1 (1 + r)t u(ct ) = Tt=1 (1 + r)t max Tt=1 t u(ct ). Also, Tt=1 t u(ct ) u T
t=1 t ct
  T
T
u t=1 t yt . So ct = c = t=1 t yt (t = 1, 2, . . . , T ) solves the problem.

Chapter 10 Integration
10.1
2 2
2. See Figs. 10.1.2(a) to 10.1.2(d). (a) 0 3x 2 dx = 0 x3 = 8 (b) 1/7 (c) e 1/e (d) 9/10
y y y y
2 1 1
2
1

1 2 x 1 x 1 1 x 1 10 x

Figure 10.1.2(a) Figure 10.1.2(b) Figure 10.1.2(c) Figure 10.1.2(d)

Knut Sydster and Peter Hammond 2010


24 CHAPTER 10 INTEGRATION

1
4. A = 1
2 1 (e
x + ex ) dx = 1 1
2 1 (e
x ex ) = e e1

10.2    
2. (a) (t 3 + 2t 3) dt = t 3 dt + 2t dt 3 dt = 41 t 4 + t 2 3t + C
 
d
(b) (x 1)2 dx = (x 2 2x +1) dx = 13 x 3 x 2 +x +C. Alternative: Since (x 1)3 = 3(x 1)2 ,
 dx
we have (x 1)2 dx = 13 (x 1)3 + C1 . This agrees with the rst answer, with C1 = C + 1/3.
 
(c) (x 1)(x + 2) dx = (x 2 + x 2) dx = 13 x 3 + 21 x 2 2x + C

 Either rst evaluate (x + 2) = x + 6x + 12x + 8, to get


(d) 3 3 2

(x + 2)3 dx = 41 x 4 + 2x 3 + 6x 2 + 8x + C, or: (x + 2)3 = 41 (x + 2)4 + C1 .



(e) (e3x e2x + ex ) dx = 31 e3x 21 e2x + ex + C
 3  
x 3x + 4 4
(f) dx = x2 3 + dx = 13 x 3 3x + 4 ln |x| + C
x x
4. (a) Differentiate the right-hand side.

1
(b) (i) 10 (2x + 1)5 + C (ii) 23 (x + 2)3/2 + C (iii) 2 4 x + C

6. (a) F (x) =  ( 21 2x) dx = 21 x x 2 + C. F (0) = 21 implies C = 21 .
(b) F (x) = (x x 3 ) dx = 21 x 2 41 x 4 + C. F (1) = 125
implies C = 16 .
8. The general form for f  is f  (x) = 13 x 3 + A, so that for f is f (x) = 12 x + Ax + B. If we require that
1 4

f (0) = 1 and f (0) = 1, then B = 1 and A = 1, so f (x) = 12 x x + 1.
1 4

10.3
 3 
1 3
5
2. (a) 12/5 (b) 41/2 (c) + t dt = ln(t 1) + 21 t 2 = ln 2 +
2 t 1 2 2
4. (a) 6/5 (b) 26/3 (c) (e 1)/ (d) ln 2 6. (a) 32/15 2.133. See Fig. 10.3.6.
y
y
y= x
2
2 4/3
1
1

x 1 x 3 4 x
1 2
Figure 10.3.6 Figure 10.3.10

  
b + c
8. (a) 59/30 (b) A b 1 + (b c) ln   + d ln |b| (c) 2 2 3/2

1+c
x
10. (a) Given F (x) = a f (t) dt, according to the mean-value theorem, there exists a number x (a, b)
b
such that [F (b) F (a)]/(b a) = F  (x ). Here F (b) = a f (t) dt, F (a) = 0, and F  (x ) = f (x ),
so the conclusion follows. (b) f (x ) = 4/3 at x = 16/9. See Fig. 10.3.10, in which the two shaded
areas are equal.

Knut Sydster and Peter Hammond 2010


CHAPTER 11 FURTHER TOPICS IN INTEGRATION 25

10.4
2. (a) Let n be the total
 2b number of individuals. The number of individuals with income in the
 2binterval [b, 2b]
2b
2 1 nB
is then N = n Br dr = n Br = . Their total income is M = n Br 2 r dr =
2b
 2b b
2b
b b

n Br 1 dr = n B ln r = nB ln 2. Hence the mean income is m = M/N = 2b ln 2.


b b  2b  2b  2b
(b) Total demand is x(p) = nD(p, r)f (r) dr = nAp r Br 2 dr = nABp r 2 dr =
b b b
2b
r 1 21 1
nABp
= nABp b1 .
b 1 1
 15 15 1 0.06t 500

4. PDV = 500e0.06t dt = 500 0 e = 1 e0.9 4945.25.
0 0.06 0.06
FDV = e0.0615 PDV = e0.9 PDV 2.4596 4945.25 12163.3.

Chapter 11 Further Topics in Integration


11.1
1 1 1
2. (a) 1 x ln(x + 2) dx = 1 21 x 2 ln(x + 2) 1 21 x 2 x+2 1
dx
 1  
= 2 ln 3 2 1 x 2 + x+2 dx = 2 2 ln 3
1 1 4 3

(b) Recall that dx d x


2 = 2x ln 2, and therefore 2x / ln 2 is the indenite integral of 2x . If follows that
 2 2  2 x 2
2x 2 8 2x 8 3
x2x dx = x dx = 2
=
0 0 ln 2 0 ln 2 ln 2 0 (ln 2) ln 2 (ln 2)2 
(c) First use integration by parts on the indenite integral: with f (x) = x 2 and g(x) = ex , () x 2 ex dx =
x 2 ex 2xex dx.  To evaluate the lastintegral we must use integration by parts once more: with f (x) = 2x
and g(x) = ex , 2xex dx = 2xex 2ex dx = 2xex (2ex +C). Inserted into () this gives x 2 ex dx =
1 1
x 2 ex 2xex +2ex +C, and hence, 0 x 2 ex dx = 0 (x 2 ex 2xex +2ex ) = (e2e+2e)(00+2) = e2.
Alternatively, more compactly using formula [11.2]:
 1  1 1  1 
1 1
x e dx = x e 2
2 x 2 x
xe dx = e 2 xe
x x
ex dx = e 2[e 0 ex ] = e 2
0 0 0 0 0
4. Using integration by parts, we have
 t1  t1  t1

F (t)(t) dt =  F (t)(t) F (t)(t) dt
t0 t0 t0
 t1  t1
= F (t1 )(t1 ) F (t0 )(t0 ) F (t)(t) dt = F (t)(t) dt
t0 t0

6. Using [11.2], we have


 T T  T
rt  rt
U (C(t))e dt =  U (C(t))(1/r)e + (d/dt)U (C(t))(1/r)ert dt
0 0 0

which becomes the required expression when U (C(0)) = 0.

Knut Sydster and Peter Hammond 2010


26 CHAPTER 11 FURTHER TOPICS IN INTEGRATION

11.2
2. (a) 19 (x 2 + 1)9 + C. (Substitute u = x 2 + 1, du = 2x dx.) (b) With u = x 3 + 2, du = 3x 2 dx and
 2 x 3 +2  1 u 1 x 3 +2
x e dx = 3 e du = 3 e + C = 3 e
1 u
+ C. (c) First attempt: u = x + 2, which gives
ln(x + 2) ln u
du = dx and dx = du. Not promising. A better idea: Substitute u = ln(x + 2).
2x + 4 2u 
dx ln(x + 2)
Then du = and dx = 2 u du = 4 (u) + C = 4 (ln(x + 2)) + C.
1 1 2 1 2
x+2 2x + 4
  
(d) Attempt: u = 1 + x. Then, du = dx, and x 1 + x dx = (u 1) u du = (u3/2 u1/2 ) du =

2 5/2
5u 23 u3/2 + C = 25 (1 + x)5/2 23(1 + x)3/2 + C. Second
 attempt: u = 1+ x. Then u2 = 1 + x

and 2udu = dx. Then the integral is x 1 + x dx = (u2 1)u2u du = (2u4 2u3 ) du e.t.c.
Check that you get the same answer. Actually, even integration by parts works in this case. Put f (x) = x

and g  (x) = 1 + x, and choose g(x) = 23 (1 + x)3/2 . (The answer looks different, but is not.)
 
x3 x2 x
(e) With u = 1 + x , x = u 1, and du = 2xdx, so
2 2 dx = dx =
  (1 + x 2 )3 (1 + x 2 )3
u1 1 1
1
du = 21 (u2 u3 ) du = 21 u1 + 41 u2 + C = + + C.
2 u3  2(1 + x 2) 4(1 +
 x)
2 2

(f) With u = 4 x 3 , u2 = 4x 3 , and 2udu = 3x 2 dx, so x 5 4 x 3 dx = x 3 4 x 3 x 2 dx =
 
(4u2 ) u ( 23 )u du = ( 83 u2 + 23 u4 ) du = 89 u3 + 15
2 5
u +C = 89 (4x 3 )3/2 + 15
2
(4x 3 )5/2 +C
 x 2x
2
x
2t 2
4. Substituting u = t 2 2t and assuming x > 2 gives dt = ln u = ln 13 (x 2 2x). Thus,
3 t 2t
2
3
the equation implies that 13 (x 2 2x) = 23 x 1. Hence, x 2 4x + 3 = 0, with solutions x = 1 and x = 3.
Here x = 1 is impossible, because the integral is not dened when x = 1. So the solution is x = 3.
6. Substitute z = x(t). Then dz = x  (t)dt, and the result follows.
 b+
8. (a) Introduce z = x as a new variable in the right-hand side integral. Then dz = dx, and a+ f (x
b
) dx = a f (z) dz. Then replace the dummy variable z by x in the last integral. (b) Introduce z = x/
as a new variable in the right-hand side integral.

u8
10. Substitute u = x . Then I = 6
1/6 du. Here u8 : (u2 +1) = u6 u4 u2 1+1/(u2 +1).
1 u2
It follows that
I = 67 x 7/6 65 x 5/6 2x 1/2 6x 1/6 3 ln |1 x 1/6 | + 3 ln |1 + x 1/6 | + C

11.3
 +  b b
1 1 1
2. (a) f (x) dx = dx = x= (b a) = 1
a ba ba a ba

+  b b
1 1 1 1
(b) xf (x) dx = x dx = x2 = (b2 a 2 ) = (a + b)
b a a 2(b a) a 2(b a) 2
1 b
1 b3 a 3 1 2
(c) x =
3
= (a + ab + b ) 2
3(b a) a 3 ba 3
b b
4. Divergence because 0 x/(1 + x 2 ) dx = 0 21 ln(1 + x 2 ) = 21 ln(1 + b2 ) as b . But
b b 1
b x/(1 + x ) dx = b 2 ln(1 + x ) = 0 for all b, so the limit is 0.
2 2

Knut Sydster and Peter Hammond 2010


CHAPTER 12 LINEAR ALGEBRA: VECTORS AND MATRICES 27
b b
6. 1
1+x 2
1
x2
for x 1, and 1
1 x2 dx = 1 (1/x) = 1 1/b 1 as b , so by
Theorem 11.1 the given integral converges.
 
8. (a) z = r1 (1 er ) (b) z = r2 1 r1 (1 er )

10. Integrating by parts, ln x/ x dx = 2 x ln x 4 x + C. Hence,
 1
1
ln x/ x dx = (2 x ln x 4 x) = 4 (2 h ln h 4 h) 4 as h 0+ , so the given
h h
integral converges to 4. ( h ln h 0 by lHpitals rule.)
  
12. See Fig. 11.3.12. If p > 1, then n=1 (1/n ) = 1 +
p p
n=2 (1/n ) is nite because
p
n=2 (1/n ) is the
sum of the shaded rectangles, and this sum is certainly less than the area under the curve y = 1/x p over

[1, ), which is equal to 1/(p 1). If p 1, the sum p
n=1 (1/n ) is the sum of the larger rectangles in
the gure, and this sum is larger than the area under the curve y = 1/x p over [1, ), which is unbounded

when p 1. Hence, p
n=1 (1/n ) diverges in this case.

y y = 1/x p

1 2 3 4 x

Figure 11.3.12

11.4
2. The Gini coefcients are approximately: 0.34 (US 1980), 0.37 (US 1990), 0.35 (Netherlands 1959), 0.27
(Netherlands 1985), 0.69 (World 1989). (These numbers are obtained by approximating the Lorenz curve
with a broken line through the points corresponding to the observations, then computing the area between
this broken line and the diagonal of the square. Note that this underestimates the Gini coefcients.)

Chapter 12 Linear Algebra: Vectors and Matrices


12.1
2. (a) No sector delivers to itself. (b) The total amount of good i needed to produce one unit of each good.
(c) This collection gives the number of units of each good which are needed to produce one unit of good j .
(d) No economic interpretation. (The goods are usually measured in different units, so it is meaningless
to add them together.)

4. The Leontief model for this three sector model is as follows:


0.9x1 0.2x2 0.1x3 = 85
0.3x1 + 0.8x2 0.2x3 = 95
0.2x1 0.2x2 + 0.9x3 = 20
which does have the claimed solution.

Knut Sydster and Peter Hammond 2010


28 CHAPTER 12 LINEAR ALGEBRA: VECTORS AND MATRICES

12.2
2. a + b + c = (1, 6, 4), a 2b + 2c = (3, 10, 2), 3a + 2b 3c = (9, 6, 9), a b c =
(a + b + c) = (1, 6, 4)
4. 3(x, y, z) + 5(1, 2, 3) = (3x 5, 3y + 10, 3z + 15) = (4, 1, 3) provided that 3x 5 = 4, 3y + 10 = 1,
and 3z + 15 = 3. So x = 3, y = 3, z = 4.
3x 2y = 1 2x + 4y = 1
6. (a) , x = 1, y = 2 (b) , which have no solution.
4x + 3y = 2 3x 6y = 0
8. We need to nd numbers t and s such that t (2, 1)+s(1, 4) = (4, 11). This vector equation is equivalent
to (2t + s, t + 4s) = (4, 11), which in turn is equivalent to the equation system (i) 2t + s = 4
(ii) t + 4s = 11. This system has the solution t = 3, s = 2, so (4, 11) = 3(2, 1) 2(1, 4).

12.3
2. (a) = 0 gives x = (3, 1), = 1/4 gives x = (2, 5/4), = 1/2 gives x = (1, 3/2),
= 3/4 gives x = (0, 7/4), = 1 gives x = (1, 2). See Fig. 12.3.2. (b) When runs through
[0, 1], then x will run through all points on the line segment between a and b.
4. See Fig. 12.3.4. (The point R should be one unit further down.)
z

S  (3,2, 4)
y
 1
 3/4
 1/2
 1/4 Q
1  0 y
b P
a
x
R
1 1 2 3 x
Figure 12.3.2 Figure 12.3.4

12.4
2. (a) a b = b a = 6, (a + b)
c = a c + b c = 9, a (3b) = 3a b = 18.
(b) a = 3, b = 3, c = 29 , (c) |ab| = 6 ab = 9
4. The scalar product of the two vectors is x 2 +(x 1)x +33x = x 2 +x 2 x +9x = 2x 2 +8x = 2x(x +4),
which is 0 for x = 0 and x = 4.
6. x = (5, 7, 12), u = (20, 18, 25), x u = 526
8. ||a + b||2 = (a + b) (a + b) = a a + 2a b + b b ||a||2 + 2||a||||b|| + ||b||2 = (||a|| + ||b||)2 .
The conclusion follows because ||a + b|| and ||a|| + ||b|| are both nonnegative.

x1 8 2 6 + 2
10. (a) Total output is x2 = 4 + (1 ) 6 = 2 + 6 .
x3 4 10 6 + 10

Knut Sydster and Peter Hammond 2010


CHAPTER 12 LINEAR ALGEBRA: VECTORS AND MATRICES 29

To produce the output in (i) we can put = 1/2. To produce the output in (ii) would require a value of
such that 6+2 = 7, 2+6 = 5, and 6+10 = 5, and the second of these equations gives a different
value for from the other two. (b) In (a) we saw that (i) can be produced even without throwing away
outputs. For (ii) to be possible if we were allowed to throw away output, there must exist a [0, 1]
such that 6 + 2 7, 2 + 6 5, and 6 + 10 5. These inequalities reduce to 5/6, 1/2,
5/6, which are incompatible.
(c) Revenue = R() = p1 x1 + p2 x2 + p3 x3 = p1 (6 + 2) + p2 (2 + 6) + p3 (6 + 10) =
(6p1 2p2 6p3 ) + 2p1 + 6p2 + 10p3 . If the constant slope 6p1 2p2 6p3 is > 0, then R() is
maximized at = 1; if 6p1 2p2 6p3 is < 0, then R() is maximized at = 0. Only in the special
case where 6p1 2p2 6p3 = 0 can the two plants both remain in use.

12.5
2. (a) Direct verication. (To show that a lies on L, put t = 0.) (b) The direction of L is given by (1, 2, 1).
(c) The equation of plane is (1)(x1 2) + 2(x2 (1)) + 1 (x3 3) = 0, or x1 + 2x2 + x3 = 1.
(d) We must have 3(t + 2) + 5(2t 1) (t + 3) = 6, and so t = 4/3. Thus P = (2/3, 5/3, 13/3).
4. (a) Direct verication. (b) x1 = 2 t, x2 = 1 + 2t, x3 = 1 + 3t.

12.6
   
2 3 4 1 1 1 1 0 0 3
2. (a) A = (b) A = 4. A + B = , 3A =
3 4 5 1 1 1 7 5 6 9

12.7

4 1 1 2 3 5 5 3 3
2. A + B = 9 2 7 , A B = 1 2 3 , AB = 19 5 16 ,
3 1 4 1 1 2 1 3 0

0 4 9 23 8 25
BA = 19 3 3 , (AB)C = A(BC) = 92 28 76
5 1 3 4 8 4

0.2875

4. T(Ts) = 0.2250
0.4875

12.8

a d e x ax + dy + ez
2. We start by performing the multiplication d b f y = dx + by + f z . Next,
e f c z ex + fy + cz

ax + dy + ez
(x, y, z) dx + by + f z = (ax 2 + by 2 + cz2 + 2dxy + 2exz + 2fyz)
ex + fy + cz
which is a 1 1 matrix.

1
4. Equality in both [1] and [2] AB = BA. 6. (a) x0 = (1/ 3) 1 (b) An x0 = x0 for all n.
1

Knut Sydster and Peter Hammond 2010


30 CHAPTER 13 DETERMINANTS AND MATRIX INVERSION


a 2 + bc ab + bd
8. (a) Direct verication yields (1) = (a + d)A (ad bc)I2 =
A2
ac + cd bc + d 2
(b) Multiplying (1) by A and using A = 0 yields (2) (a + d)A2 = (ad bc)A, and further,
3

(3) 0 = (a + d)A3 = (ad bc)A2 . If ad bc  = 0, (3) yields A2 = 0. If ad bc = 0,


(2) yields (a + d)A2 = 0, and if a +d = 0, again
A = 0. Finally, if ad bc = a + d = 0,
2

1 1
then (1) implies A2 = 0. (c) Let A = . Then A2 = 0, and also (of course) A3 = 0.
1 1

12.9
   
3 1 0 2 3 1 6 2
2. A = , B = , (A + B) = , (A) = ,
2 5 2 2 4 7 4 10
  
4 10 4 10 2 4
AB = , (AB) = = B A , A B =
10 8 10 8 10 14
4. Symmetry if a 2 1 = a + 1 and a 2 + 4 = 4a, which reduces to a = 2.
6. (a) (A1 A2 A3 ) = (A1 (A2 A3 )) = (A2 A3 ) A1 = (A3 A2 )A1 = A3 A2 A1 . (b) Easy induction proof.
  
8. (a) tr(A + B) = ni=1 (aii + bii ) = ni=1 aii + ni=1 bii = tr(A) + tr(B)
   
(b) tr(cA) = ni=1 caii = c ni=1 aii = c tr(A) (c) tr(AB) = ni=1 jn=1 aij bj i , whereas tr(BA) =
n n n n
i=1 j =1 bij aj i = j =1 i=1 bj i aij , because the indices i and j can be interchanged. The two are
equal because the order of multiplication and summation in each double sum is irrelevant. (If this is hard
to understand, write out the sums in full for the cases when n = 2 and/or n = 3.)

(d) tr(A ) = ni=1 aii = tr(A)

Chapter 13 Determinants and Matrix Inversion


13.1
 
3 0 
2. See Fig. 13.1.2. The shaded area is 18 =  .
2 6

(2, 6)

(3, 0)
Figure 13.1.2


a11 b11 + a12 b21 a11 b12 + a12 b22
4. The matrix product is AB = , implying that
a21 b11 + a22 b21 a21 b12 + a22 b22
|AB| = (a11 b11 + a12 b21 )(a21 b12 + a22 b22 ) (a11 b12 + a12 b22 )(a21 b11 + a22 b21 ). On the other hand,
|A||B| = (a11 a22 a12 a21 )(b11 b22 b12 b21 ). A tedious computation shows that the two expressions
are equal.

Knut Sydster and Peter Hammond 2010


CHAPTER 13 DETERMINANTS AND MATRIX INVERSION 31

Y C = I0 + G0
6. We write the system as . Then Cramers rule yields
bY + C = a

   
 I0 + G0 1   1 I 0 + G0 
   
 a 1  a + I 0 + G0  b a  a + b(I0 + G0 )
Y =   = , C=   =
 1 1  1b  1 1  1b
   
 b 1  b 1

The expression for Y is most easily found by substituting the second equation into the rst, and then
solving for Y . Then use C = a + bY to nd C.

8. |A(t)| = 2t (1 t), which is 0 for t = 0 and t = 1.

13.2

1 1 1
2. AB = 7 13 13 , |A| = 2, |B| = 3, |AB| = |A| |B| = 6
5 9 10

4. By Sarrus rule the determinant is (1 + a)(1 + b)(1 + c) + 1 + 1 (1 + b) (1 + a) (1 + c), which


reduces to the given expression.

6. Direct verication using [13.7] or Sarrus rule.

13.3

2. +a12 a23 a35 a41 a54 (Four lines between pairs of elements rise as one goes to the right.)

13.4

2 1 1

2. A = 1 0 2 , |A| = |A | = 2
3 1 5

4. (a) The rst and the second columns are proportional. (b) Add the second column to the third. Then the
rst and the third columns are proportional. (c) The term x y is a common factor for each entry in
the rst row. If x = y, all elements in the rst row are 0. If x y = 0, we divide this row by x y, and
the resulting determinant has rows 1 and 2 identical, so it is 0.

6. |AB| = |A||B| = 12, 3|A| = 9, | 2B| = (2)3 (4) = 32, |A| + |B| = 1, whereas |A + B| is not
determined.

8. (a) Because A2 = In it follows from part 8 of Theorem 13.1 that |A|2 = 1, and so |A| = 1.
(b) Direct verication. (c) (In A)(In + A) = In2 + In A AIn A2 = In A2 = 0 A2 = In

Knut Sydster and Peter Hammond 2010


32 CHAPTER 13 DETERMINANTS AND MATRIX INVERSION

10. Start by adding the sum of the last n 1 rows to the rst row. Then na + b is a common factor in the
rst row, which we take out. Next, add the rst row multiplied by a to all the other n 1 rows. The
upper triangular matrix resulting from these operations obviously has the determinant bn1 :
   
a + b a a  na + b na + b na + b 
   
 a a+b a 1  a a + b a 
Dn =  .. .. .  =  . .. .. 
. ..  
.. .. . ..
 . . .  . . . . 
   
a a a 1 a a a+b
  a a  
1 1 1  1 1 1
   
a a + b a  0 b 0
= (na + b)  .. .. .. .
.

 .. = (na + b) .
 .. .. .. .. 
. . . .  .  . . .
   
a a a + b 0 0 b

According to [13.14], the last determinant is bn1 . Thus Dn = (na + b)bn1 .

13.5
2. In each of these cases we keep expanding by the last (remaining) column. The answers are: (a) abc
(b) abcd (c) 6 4 3 5 1 = 360

13.6
2. Multiply the two matrices to get I3 .
 1 
2 3 4 3
4. Using = , we get: (a) x = 3, y = 1 (b) x = 1, y = 2 (c) x = y = 0
3 4 3 2

0 1 1 1 1 2
6. (a) |A| = 1, A2 = 1 1 2 , A3 = 2 2 3 , then direct verication of the equality.
1 1 1 1 2 2
0 0 1
(b) By (a), (A I)2 A = I. (c) P = (A I)1 = 1 0 1 does the job. So does P.
0 1 0
8. (a) A2 = (PDP1 )(PDP1 ) = PD(P1 P)DP1 = PDIDP1 = P(D)2 P1 .
(b) Suppose the formula is valid for m = k. Then Ak+1 = AAk = PDP1 (PDk P1 ) = PD(P1 P)Dk P1
= PDIDk P1 = PDDk P1 = PDk+1 P1 .
10. (a) If C2 + C = I, then C(C + I) = I, and so C1 = C + I = I + C. (b) Because C2 = I C, so
C3 = C2 C = (I C)C = C C2 = C (I C) = I + 2C. Moreover, C4 = C3 C = (I + 2C)C =
C + 2C2 = C + 2(I C) = 2I 3C.

s + 17 4t 16 0
1
12. (a) AT = 12 2s + 10 5t 8 0 . We see that AT = I3 and so T = A1 for s = 5, t = 4.
3s + 15 4t 16 12
(b) BX = 2X + C BX 2X = C (B 2I)X = C. But it iseasy to see that B 2I =A,
1/2 0 0 1/6
so BX = 2X + C AX = C X = A1 C = TC. Hence, X = 1/2 3 3 1/6 .
1/2 1 2 1/6

Knut Sydster and Peter Hammond 2010


CHAPTER 14 FURTHER TOPICS IN LINEAR ALGEBRA 33

13.7

C C21 C31 3 5 9 18 16 10
1 11 1
2. C12 C22 C32 = 18 6 18 . 3. (I A)1 5
= 62 2 19 8
|A| 72
C13 C23 C33 6 14 18 4 7 16
4. Let B denote the n p matrix whose ith column has the elements bi1 , bi2 , . . . , bin . The p systems of n
equations in n unknowns can be expressed as AX = B, where A is n n and X is n p. Following the
method in Example 13.20, exactly the same row operations that transforms the n 2n matrix (A : I) into
(I : A1 ) will also transform the n (n + p) matrix (A : B) into (I : B ), where B is the matrix with
elements bij . Because these row operations are together equivalent to premultiplcation by A1 , it must

be true that B = A1 B. When i = k, the solution to the system is x1 = bk1 , x2 = bk2 , . . . , xn = bkn .

13.8
2. The determinant of the system is equal to 10, so the solution is unique. The determinants in (2) are
     
 b1 1 0  3 b1 0  3 1 b1 
    
D1 =  b2 1 2 ,
 D2 =  1 b2 2 ,
 D3 =  1 1 b2 
b 3 1  2 b3 1  2 3 b3 
3

Expanding each of these determinants by the column (b1 , b2 , b3 ), we nd that D1 = 5b1 + b2 + 2b3 ,
D2 = 5b1 3b2 6b3 , D3 = 5b1 7b2 4b3 . Hence, x1 = 21 b1 101
b2 15 b3 , x2 = 21 b1 + 10
3
b2 + 53 b3 ,
x3 = 2 b1 + 10 b2 + 5 b3 .
1 7 2

Chapter 14 Further Topics in Linear Algebra


14.1
2. Only the vectors in (b) are linearly independent.
4. If x(1, 1, 1) + y(2, 1, 0) + z(3, 1, 4) + w(1, 2, 2) = (0, 0, 0), then x, y, z, and w satisfy the equations
x + 2y + 3z + w = 0, x + y + z + 2w = 0, x + 4z 2w = 0. One solution is x = 2, y = 1, z = 1,
w = 1, so the vectors are linearly dependent.
6. (a) Suppose a + b + c = 0. Taking the scalar product of each side with a yields a (a + b + c) =
a 0 = 0. But a b = a c = 0, so (a a) = 0. Because a  = 0, = 0. In a similar way we can prove
that = = 0, so a, b, and c are linearly independent. (b) Generalize the proof in (a).

14.2
2. (a) By cofactor expansion along the rst row, the determinant of the matrix A is |A| = x (1) 0 1 +
(x 2 2) 1 = x 2 x 2 = (x + 1)(x 2). If x = 1 and x = 2, then |A|  = 0, so the rank of A
equals 3. If x = 1 or x = 2, then |A| = 0 and r(A)  2. On
 the other hand, the minor we get if we
 0 1
strike out the rst row and the third column in A is   = 1  = 0 for all x, so r(A) can never be
1 x 
less than 2. Thus, r(A) = 2 if x = 1 or x = 2, r(A) = 3 otherwise.
(b) A little calculation shows that the determinant is t 3 + 4t 2 4t 16, and if we note that this expression
has t + 4 as a factor, it follows that the determinant is t 3 + 4t 2 4t 16 = t 2 (t + 4) 4(t + 4) =

Knut Sydster and Peter Hammond 2010


34 CHAPTER 14 FURTHER TOPICS IN LINEAR ALGEBRA

(t 2 4)(t + 4) = (t + 2)(t 2)(t + 4). Thus, if t is not any of the numbers 2, 2, and 4, the rank
of
 the matrix is 3. If we strike out the second row and the rst column of the matrix, we get the minor
5 6 
 
 1 t + 4  = 5t + 14, which is different from 0 for all the three special values of t that we found above,
and thus the rank of the matrix is 2 if t = 4, 2, or 2, and 3 otherwise.
(c) The rst and third rows are identical, as are the second and fourth. But the rst two rows are always
linearly independent. So the rank is 2 for all values of x, y, z, and w.

14.3
2. Unique solution for c  = 3/2: x = (3 kc)/(3 2c), y = (k 2)/(3 2c). For c = 3/2, k = 2 the
solutions are x = (3/2)a + 1, y = a, a arbitrary. For c = 3/2, k  = 2 there are no solutions.

4. A((1 )x1 + x2 ) = (1 )Ax1 + Ax2 = (1 )b + b = b. This shows that if x1 and x2 are


different solutions, then so are all points on the straight line through x1 and x2 .

6. (a) |At | = (t 2)(t + 3), so At has an inverse for all t except


t = 2 and t = 3.
1 3
(b) r(At ) = 3 for t  = 2 and for t = 3. Because is always nonsingular, r(A2 ) = 2, r(A3 ) = 2.
2 5
(c) x1 = 46 + 19t, x2 = 19 7t, x3 = t, with t arbitrary.
(d) Any z  = 0 satisfying zA2 = 0 has this property. In particular, z = (10a, 7a, a) for a  = 0.

14.4
2. |A I| = 0 iff |(A I) | = 0 iff |A I| = 0 using [12.44] in Section 12.9 and Thm. 13.1 [2] in
Section 13.4. The conclusion follows.
2 3
2a 2a 2 0 4a 4a 3 0
4. (a) X AX = (ax 2 + ay 2 + bz2 + 2axy), A2 = 2a 2 2a 2 0 , A3 = 4a 3 4a 3 0
0 0 b2 0 0 b3
(b) = 0, = 2a, = b (c) p() = (b )(2a ) = 3 + (2a + b)2 2ab. Using A2 and
A3 from part (a), it is easy to show that p(A) = A3 + (2a + b)A2 2abA = 0.

6. Since A1 exists, |A|  = 0. So is an eigenvalue for AB |AB I| = 0 |A(B A1 )| =


0 |B A1 | = 0 |(B A1 )A| = 0 |BA I| = 0 is an eigenvalue for
BA. (Using [14.11] in the next section, here is a simpler argument: AB and A1 (AB)A = BA have the
same eigenvalues.)

14.5
2. (a) No, because the eigenvectors are not linearly independent. (b) Yes, because the eigenvectors are
linearly independent.

14.6
 
1 2 
2. (a) |A I| =  = 2 + 6 = 0, with the real eigenvalues 1 = 3 and 2 = 2. One
2 2   
1 2
set of linearly independent eigenvectors is and . These two eigenvectors are orthogonal.
2 1

Knut Sydster and Peter Hammond 2010


CHAPTER 15 FUNCTIONS OF SEVERAL VARIABLES 35

Thus, Theorem 14.7 (not 14.6!) is conrmed. Further, let


 
1 2 1 1/5 2/5
P= for which P =
2 1 2/5 1/5

It is easy to verify that P1 AP is the diagonal matrix diag (3, 2), which conrms Theorem 14.8 (not
14.6!).
 
 0 1 

(b) Here |A I| =  0 1  = ( 1)( + 2) = 0, with the real eigenvalues 1 = 2,
 1 1 1 
1 1 1
2 = 0 and 3 = 1. One set of linearly independent eigenvectors is 1 , 1 , and 1 .

2 0 1
Obviously, these eigenvectors are mutually orthogonal. Thus Theorem 14.7 is conrmed. Further, let

1 1 1 1/6 1/6 1/3
P= 1 1 1 for which P1 = 1/2 1/2 0
2 0 1 1/3 1/3 1/3

It is easy to verify that P1 AP is the diagonal matrix diag (2, 0, 2), which conrms Theorem 14.8.

Chapter 15 Functions of Several Variables


15.1
2. f (1, 1) = 2, f (2, 3) = 51, f (1/x, 1/y) = 3/x 2 2/xy + 1/y 3 , [f (x + h, y) f (x, y)]/ h =
6x 2y + 3h, [f (x, y + k) f (x, y)]/k = 2x + 3y 2 + 3yk + k 2

4. (a) F (1, 1) = 10, F (4, 27) = 60, F (9, 1/27) = 10, F (3, 2 ) = 10 3 21/6 ,
F (100, 1000) = 1000, F (2K, 2L) = 25/6 F (K, L) (b) a = 5/6
6. (a) y gets 21.053 2.07 times larger.
(b) ln y = ln 2.9 + 0.015 ln x1 + 0.25 ln x2 + 0.35 ln x3 + 0.408 ln x4 + 0.03 ln x5
8. (a) x + y > 0 (b) Must require 2 (x 2 + y 2 ) 0, i.e. x 2 + y 2 2.
(c) Put a = x 2 + y 2 . We must require (4 a)(a 1) 0, i.e. 1 a 4. (Use a sign diagram.)
 
10. Total output is ni=1 (T /ti ) = T ni=1 (1/ti ). If all n of the machines were equally efcient, and their
production per unit were tH , then each machine would produce T /tH units. Total output is the same as
 
before if T ni=1 (1/ti ) = nT /tH . So tH = n/ ni=1 (1/ti ), the harmonic mean of t1 , . . . , tn .

15.2

2. x 2 + y 2 =
6 is a level curve of f at height c = 6 4, because when x +
2 y 2 = 6, then f (x, y) = 6
6 + 2 = 6 4. In the general case, a level curve is dened by f (x, y) = x 2 + y 2 (x 2 + y 2 ) + 2 = c
for some constant c. Let x 2 + y 2 = u2 , where u > 0. Then u u2 + 2 = c. Solving for u yields

u = 21 21 9 4c (provided that c 9/4) and thus x 2 + y 2 is a constant, which is the equation for a

circle centered at (0, 0). If 9 4c > 0 and 1 9 4c > 0, i.e. 2 < c < 9/4, there are two circles. If
c 2 or c = 9/4, there is only one positive value of u, so the level curve is one circle.

Knut Sydster and Peter Hammond 2010


36 CHAPTER 15 FUNCTIONS OF SEVERAL VARIABLES

4. Generally, the graph of g(x, y) = f (x) in 3-space consists of a surface traced out by moving the graph
of z = f (x) parallel to the y-axis in both directions. The graph of g(x, y) = x is the plane through the
y-axis at a 45 angle with the xy-plane. The graph of g(x, y) = x 3 is shown in Fig. 15.2.4. (Only a
portion of the unbounded graph is indicated, of course.)
z

Figure 15.2.4

15.3
2. (a) f1 (x, y) = 7x 6 , f2 (x, y) = 7y 6 , and f12

(x, y) = 0
  
(b) f1 (x, y) = 5x ln y, f2 (x, y) = x /y, and f12
4 5 (x, y) = 5x 4 /y
(c) f1 (x, y) = 5(x 2 2y 2 )4 2x = 10x(x 2 2y 2 )4 , f2 (x, y) = 5(x 2 2y 2 )4 (4y) = 20y(x 2 2y 2 )4 ,

and f12 (x, y) = 40x(x 2 2y 2 )3 (4y) = 160xy(x 2 2y 2 )3

4. (a) FS = 2.260.44S 0.56 E 0.48 = 0.9944S 0.56 E 0.48 , FE = 2.260.48S 0.44 E 0.52 = 1.0848S 0.44 E 0.52
(b) SFS + EFE = S 2.26 0.44S 0.56 E 0.48 + E 2.26 0.48S 0.44 E 0.52 = 0.44 F + 0.48 F = 0.92 F ,
so k = 0.92.

6. (a) zx = 2x, zy = 2e2y , zxx


 = 2, z = 4e2y , z = z = 0 (b) z = y/x, z = ln x, z = y/x 2 ,
yy xy yx x y xx
zyy = 0, zxy = zyx = 1/x (c) zx = y 2 yexy , zy = 2xyxexy , zxx
    = y 2 exy , z = 2xx 2 exy ,z =
yy xy
 = 2y xyexy exy
zyx

8. Here z/x = x/(x 2 + y 2 ), z/y = y/(x 2 + y 2 ), 2 z/x 2 = (y 2 x 2 )/(x 2 + y 2 )2 , and 2 z/y 2 =


(x 2 y 2 )/(x 2 + y 2 )2 . Thus, 2 z/x 2 + 2 z/y 2 = 0.

10. ux = au/x and uy = bu/y, so uxy = auy /x = abu/xy. Hence, uxy /ux uy = 1/u (u  = 0). Then,
   
1 uxy 1 ux 1 1 uxy
=  2 = 2 = 
ux x ux uy ux u u uy y ux uy

15.4
1 2
2. F (1, 0) = F (0, 0) + 0 F1 (x, 0) dx 2, F (2, 0) = F (1, 0) + 1 F1 (x, 0) dx F (1, 0) + 2
1  1 
F (0, 1) = F (0, 0) + 0 F2 (0, y) dy 1, F (1, 1) = F (0, 1) + 0 F1 (x, 1) dx F (0, 1) + 2
1
F (1, 1) = F (1, 0) + 0 F2 (1, y) dy F (1, 0) + 1

4. With u = y/x, zx = f (u) uf  (u) and zy = f  (u). The equation for the tangent plane at (x, y, z)
is Z xf (u) = [f (u) uf  (u)](X x) + f  (u)(Y y). This goes through the origin (0, 0, 0) iff

Knut Sydster and Peter Hammond 2010


CHAPTER 15 FUNCTIONS OF SEVERAL VARIABLES 37

xf (u) = [f (u) uf  (u)](x) + f  (u)(y) iff 0 = xuf  (u) + f  (u)(y), which is satised because
xu = y.

15.5
2. F1 (x, y, z) = 2xexz + x 2 zexz + y 4 exy , so F1 (1, 1, 1) = 4e; F2 (x, y, z) = 3y 2 exy + xy 3 exy , so
F2 (1, 1, 1) = 4e; F3 (x, y, z) = x 3 exz , so F3 (1, 1, 1) = e.

4. First-order partials: w1 = 3yz + 2xy z3 , w2 = 3xz + x 2 , w3 = 3xy 3xz2 . Second-order partials:
        
w11 = 2y, w12 = w21 = 3z + 2x, w13 = w31 = 3y 3z2 , w22 = 0, w23 = w32 = 3x, w33 = 6xz.

6. For (x, y)  = (0, 0), f1 = y(x 4 + 4x 2 y 2 y 4 )(x 2 + y 2 )2 . Thus, for y  = 0, f1 (0, y) = y. This is also
correct for y = 0, because f1 (0, 0) = limh0 [f (h, 0) f (0, 0)]/ h = 0. Similarly, for (x, y)  = (0, 0),
f2 = x(x 4 4x 2 y 2 y 4 )(x 2 + y 2 )2 , and f2 (x, 0) = x for all x. It follows that f12 
(0, 0) = 1 and
  
f21 (0, 0) = 1. Also, for (x, y)  = (0, 0), f12 = f21 = (x + 9x y 9x y y )(x 2 + y 2 )3 . Thus,
6 4 2 2 4 6
   
f12 (0, y) = 1, f12 (x, 0) = 1, and f12 (x, y) has no limit as (x, y) (0, 0). Similarly for f21 .

15.6
2. (a) YK = aAK a1 and YK = aBLa1 , so KYK + LYL = aAK a + aBLa = a(AK a + BLa ) = aY
(b) KYK + LYL = KaAK a1 Lb + LAK a bLb1 = aAK a Lb + bAK a La = (a + b)AK a Lb = (a + b)Y
2aKL5 bK 4 L2 2bK 5 L aK 2 L4
(c) YK = and Y 
= , so KYK + LYL =
(aL3 + bK 3 )2 L
(aL3 + bK 3 )2
2aK 2 L5 bK 5 L2 + 2bK 5 L2 aK 2 L5 K 2 L2 (aL3 + bK 3 ) K 2 L2
= = = Y . (According to
(aL + bK )
3 3 2 (aL + bK )
3 3 2 aL3 + bK 3
Section 16.5 these functions are homogeneous of degrees a, a + b, and 1, respectively, so the results are
immediate consequences of Eulers Theorem.)

4. D/p and E/q are normally negative, because the demand for a commodity goes down when the
price of that commodity increases. D/q and E/p are (usually) positive, because the demand for a
commodity increases when the price of a substitute increases.

(m/)1 
(m/)1
6. YK = maK 1 Aet aK + bL , YL = mbL1 Aet aK + bL .
 


(m/)1
Thus, KYK + LYL = m(aK + bL )Ae aK + bL t = mY . (This function is homo-
geneous of degree m, so the result is an immediate consequences of Eulers Theorem.)

15.7
2. If the monopolist is not allowed to discriminate, he must charge the same price P in both markets. Thus,
from [1], P = a1 b1 Q1 = a2 b2 Q2 . Substituting Q1 = (a1 P )/b1 and Q2 = (a2 P )/b2 in the
expression for prot gives the quadratic function = (a1 b2 +a2 b1 ) [a1 b2 +a2 b1 +(b1 +b2 )]P +
(b1 + b2 )P 2 /b1 b2 . This is maximized when P = 21 + [a1 b2 + a2 b1 )/2(b1 + b2 )], giving maximum
prot of [a1 b2 + a2 b1 (b1 + b2 )]2 /4b1 b2 (b1 + b2 ). If this expression is subtracted from the (higher)
maximum prot in the example, we eventually nd that the loss of prot is (a1 a2 )2 /4(b1 + b2 ). Note
that the loss is 0 if a1 = a2 .

4. L = yy xy
2
/xx

Knut Sydster and Peter Hammond 2010


38 CHAPTER 16 TOOLS FOR COMPARATIVE STATICS

15.8
2. (a) = 5/4, = 1/2, d = 7/8 (b) = 3/5, = 1/5, d = 26/5
    
a b x ax + by a b x
4. = , so (x, y) = x(ax + by) + y(bx + cy), which reduces
b c y bx + cy b c y
to ax 2 + 2bxy + cy 2 .

15.9

  3 1 3/2 x1
1 1 x
2. (a) (x, y) (b) (x1 , x2 , x3 ) 1 1 2 x2
1 1 y
3/2 2 3 x3
4. Let 1 , . . . , n be the eigenvalues of A. If A is positive semidenite and symmetric, then according to
Theorem 15.2(b), all the eigenvalues are nonnegative. Because |A| is not 0, 0 is not an eigenvalue, so all
the eigenvalues must be positive. Then according to Theorem 15.2(a), A is positive denite.

Chapter 16 Tools for Comparative Statics


16.1
dz dy z z dy z z
2. = F1 (t, y) + F2 (t, y) = + . If z = t 2 + yey and y = t 2 , then = 2t, = ey + yey ,
dt dt t y dt t y
dy dz 2 2
and = 2t. Hence we get = 2t (1 + et + t 2 et ).
dt dt
4. The usual rules for differentiating (a) a sum; (b) a difference; (c) a product; (d) a quotient; (e) a composite
function of one variable.
6. (a) F is dened for x > 0, y > 0. F (x, y) = 0 y = 1/x 2 . See Fig. 16.1.6.
dz 2 + e3t
(b) = 2 as t .
dt 1 + e3t
y

2
F (x, y) > 0

y = 1/x 2

1 2 x

Figure 16.1.6

8. From [16.1], dz/dt = F1 (x, y)dx/dt + F2 (x, y)dy/dt. Differentiating w.r.t. t yields d 2 z/dt 2 =
(d/dt)[F1 (x, y)dx/dt] + (d/dt)[F2 (x, y)dy/dt]. Here (d/dt)[F1 (x, y)dx/dt] = [F11

(x, y)dx/dt +
    
F12 (x, y)dy/dt]dx/dt+F1 (x, y)d x/dt . Also, (d/dt)[F2 (x, y)dy/dt] = [F21 (x, y)dx/dt+F22
2 2 (x, y)dy/dt]dy/dt+
  
F2 (x, y)d y/dt . Assuming F12 = F21 , the conclusion follows.
2 2

Knut Sydster and Peter Hammond 2010


CHAPTER 16 TOOLS FOR COMPARATIVE STATICS 39

16.2
2. z/t1 = F  (x)x/t1 , z/t2 = F  (x)x/t2
4. (a) Let u = ln v, where v = x 3 + y 3 + z3 3xyz. Then u/x = (1/v)(v/x) = (3x 2 3yz)/v.
Similarly, u/y = (3y 2 3xz)/v, and u/z = (3z2 3xy)/v. Hence,

u u u x y z 3v
x +y +z = (3x 2 3yz) + (3y 2 3xz) + (3z2 3xy) = =3
x y z v v v v

which proves (i). Equation (ii) is then proved by elementary algebra. (b) Here z/x = 2xyf  (x 2 y)
and z/y = x 2 f  (x 2 y), so xz/x = 2x 2 yf  (x 2 y) = 2yz/y.
1 1
6. F  () = 0 (/)(xex ) dx = 0 x 3 ex dx. Introduce u = x 2 as a new variable. Then the integral
2 2

  1 2
is (1/2 2 ) 0 ueu du = (1/2 2 )0 (ueu eu ) = (e e + 1)/2 2 . Also, F () = (1/2)0 ex =

(e 1)/2, so F  () = (e e + 1)/2 2 is correct.


 t
8. With T = T (t), N (t) = n(t)e(tT ) n(t T )e(tT ) (1 T ) + n( )()(1 T )e(tT ) d =
tT
[n(t) (1 T )n(t T )]e(tT ) (1 T )N (t).
 2t 
10. We have z(t) = t F (, t) d , where F (, t) = x( )e t r(s) ds . Leibnizs formula gives z(t) =
 2t  2t  2t 
2F (2t, t) F (t, t) + t (F (, t)/t) d = 2x(2t)e t r(s) ds x(t) + t x( )e t r(s) ds r(t) d =
 2t
2x(2t)p(t) x(t) + t F (, t)r(t) d = 2p(t)x(2t) x(t) + r(t)z(t), and therefore z(t) r(t)z(t) =
2p(t)x(2t) x(t).
Q a
12. (a) g  (Q) = c + h 0 f (D) dD p Q f (D) dD and g  (Q) = (h + p)f (Q) 0 for all Q, so g
 Q a a
is convex. (b) g  (Q ) = 0 yields c + h 0 f (D) dD p Q f (D) dD = 0. Here Q f (D) dD =
a  Q
0 f (D) dD 0 f (D) dD = 1 F (Q ). It follows that F (Q ) = (p c)/(h + p).

16.3
2. (a) With F (x, y) = xy, F1 = y, F2 = x, F11
 
= F22 
= 0, and F12 = 1. Hence, [16.9] and [16.12] yield
y 1 2yx 2
y  = , y  = 3 (2 1 yx) = 3 = 3
x x x x
(b) y  = (1 + 3y)/(3x 1), y  = 6(1 + 3y)/(3x 1)2
(c) y  = 5x 4 /6y 5 , y  = (10/3)x 3 y 5 (125/36)x 8 y 11
4. (a) With F (x, y) = 2x 2 + xy + y 2 , y  = F1 /F2 = (4x + y)/(x + 2y) = 4 at (2, 0). Moreover,
y  = (28x 2 + 14y 2 + 14xy)/(x + 2y)3 = 14 at (2, 0). The tangent has the equation y = 4x + 8.
(b) y  = 0 requires y = 4x. Inserting this into the original equation gives the two points.
6. For x = 1 and y = 2 both LHS and RHS are 1, so the curve passes through (1, 2). Implicit
differentiation yields 2yy  + 5 = ex(y2) + xex(y2) (y 2 + xy  ). At (1, 2) we nd y  = 4/3, so the
equation for the tangent is y = 13 (4x + 2).

16.4
2. Let z = ug with u = a1 x1d + a2 x2d + a3 x3d . Then, for i = 1, 2, 3, Eli z = Elu ug Eli u = g(xi /u)ai dxid1 =
dgai xid /u, so El1 z + El2 z + El3 z = dga1 x1d /u + dga2 x2d /u + dga3 x3d /u = dg. (This result follows

Knut Sydster and Peter Hammond 2010


40 CHAPTER 16 TOOLS FOR COMPARATIVE STATICS

easily from the fact that the function is homogeneous of degree dg (see Problem 2(b) in Section 16.6)
and from the elasticity form [16.28] of the Euler equation.)
4. (a) Elx z = 20, Ely z = 30, Elt x = t/(t + 1), Elt y = 2t/(t + 1), so, according to [16.15], Elt z =
Elx z Elt x + Ely z Elt y = 20t/(t + 1) + 30 2t/(t + 1) = 80t/(t + 1).
2x 2 1 2y 2 2[ln t + t 4 e2t (2 t)]
(b) Elt z = 2 + (t + 2) =
x + y 2 ln t x2 + y2 (ln t)2 + t 4 e2t
6. Taking logarithms yields ln z = ax + by and a ln x + b ln y + c ln z = 0. Differentiating w.r.t. x yields
zx /z = a +byx and a/x +byx /y +czx /z = 0. Solving for yx and zx , and then multiplying the expressions
a(1 + cx) a(x y)
obtained by x/y and x/z respectively, yields nally Elx y = , Elx z = .
b(1 + cy) 1 + cy
8. The easiest way to solve this problem is to note that
KL
= KL(aL2/3 + bK 2/3 )3/2 = (aK 2/3 + bL2/3 )3/2
(aL + bK 2/3 )3/2
2/3

Comparing this with Example 16.20, we see that the production function is actually a CES-function with
A = A0 e0.021t and = 2/3. According to [2] in that example, KL = 1/(2/3 + 1) = 3/5.
10. With F (K, L) = AK a Lb , FK = aF /K, FL = bF /L, FKK  
= a(a 1)F /K 2 , FKL = abF /KL, and
    
FLL = b(b1)F /L 2 .

 2 Also, F F
K L (KF K +LF L ) = (aF
/K)(bF /L)(a+b)F = ab(a+b)F 3 /KL.

Moreover, KL (FL ) FKK 2FK F2 FKL 


+ (FK )2 FLL

= ab(a + b)F 3 /KL, so KL = 1.

16.5
2. x(tp, tr) = A(tp)1.5 (tr)2.08 = At 1.5 p 1.5 t 2.08 r 2.08 = t 1.5 t 2.08 Ap 1.5 r 2.08 = t 0.58 x(p, r), so the
function is homogeneous of degree 0.58 .
4. By using [16.18] we nd that f is homogeneous of degree 0, and using the results in Example 15.13(b)
in Section 15.3 we conrm that xf1 (x, y) + yf2 (x, y) = 0.
6. f (tx, ty) = a ln[g(tx, ty)/tx] = a ln[tg(x, y)/tx] = a ln[g(x, y)/x] = f (x, y), so f is homogeneous
of degree 0.
8. Suppose F is homogeneous of degree k  = 0. Then F 1 is homogeneous of degree 1/k iff F 1 (tx) =
t 1/k F 1 (x) for all t > 0. But this equation is equivalent to F (F 1 (tx)) = F (t 1/k F 1 (x)). Here the
LHS is tx, and because F is homogeneous of degree 1/k, the RHS is (t 1/k )k F (F 1 (x)) = tx. The

function F (x) = x dened for x 0 with range [0, ) has an inverse F 1 = x 2 , dened for x 0.
F is homogeneous of degree 1/2, while F 1 is homogeneous of degree 2.

16.6

(tx1 tx2 tx3 )2 1 1 1
2. (a) F (tx1 , tx2 , tx3 ) = + +
(tx1 )4 + (tx2 )4 + (tx3 )4 tx1 tx2 tx3

t 6 (x1 x2 x3 )2 1 1 1 1
= 4 4 + + = tF (x1 , x2 , x3 )
t (x1 + x24 + x34 ) t x1 x2 x3

The function is homogeneous of degree 1.



g
g
(b) G(tx1 , tx2 , tx3 ) = a(tx1 )d + b(tx2 )d + c(tx3 )d = t d (ax1d + bx2d + cx3d ) =
(t d )g G(x1 , x2 , x3 ) = t dg G(x1 , x2 , x3 ), so G is homogeneous of degree dg.

Knut Sydster and Peter Hammond 2010


CHAPTER 16 TOOLS FOR COMPARATIVE STATICS 41

4. All are homogeneous of degree 1, as is easily checked by using [16.23].


6. When F (x, y) = a ln x + b ln y, xF1 + yF2 = x(a/x) + y(b/y) = a + b, which is not equal to kF (x, y)
for any choice of k, so F is not homogeneous of any degree.
8. (a) f ((tx1 )m , . . . , (txn )m ) = f (t m x1m , . . . , t m xnm ) = (t m )r f (x1m , . . . , xnm ) = t mr h(x1 , . . . , xn ), so h is
homo. of degree mr. (b) Homo. of degree sp. (c) Homo. of degree r for r = s, not homo. for r  = s.
(d) Homo of degree r + s. (e) Homo. of degree r s.

16.7
2. Differentiating partially w.r.t. x yields () 3x 2 + 3z2 zx 3zx = 0, so zx = x 2 /(1 z2 ). Similarly, or by
symmetry, zy = y 2 /(1 z2 ). To nd zxy  , differentiate () partially w.r.t. y to obtain 6zz z + 3z2 z
y x xy
 
3zxy = 0, so zxy = 2zx y /(1 z ) .
2 2 2 3

4. Implicit differentiation gives fP (r, P )Pw = gw (w, P ) + gP (w, P )Pw . Hence
gw (w, P )
Pw =  < 0, because gw > 0, gP > 0, and fP < 0.
gP (w, P ) fP (r, P )
6. Differentiating the equation w.r.t. x gives 1 azx = f  (y bz)(bzx ). Differentiating w.r.t. y gives
azy = f  (y bz)(1 bzy ). Simple algebra yields azx + bzy = 1.

16.8
2. Write the function on the form g (, ) = (1 + )a (1 + )a 1, where a = 1/(1 + ). Then

g (, )/ = a(1 + )a1 (1 + )a , g (, )/ = (1 + )a a(1 + )a1

Hence, g (0, 0) = 0, g (0, 0)/ = a, g (0, 0)/ = a, and g (, ) a + a.


4. f (0.98, 1.01) 4.97. The exact value is 4.970614, so the error is 0.000614.
6. (a) and (b): dz = (y 2 + 3x 2 )dx + 2xy dy
8. (a) dU = 2a1 u1 du1 + + 2an un dun
1 1
(b) dU = A(1 u1 + + n un )11/ (1 u1 du1 + + n un dun )
10. Let T (x, y, z) = [x 2 + y 2 + z2 ]1/2 = u1/2 , where u = x 2 + y 2 + z2 . Then dT = 21 u1/2 du =
1 1/2
2u (2xdx + 2ydy + 2zdz). For x = 2, y = 3, and z = 6, we have u = 49, T = 7 and dT =
1
7 (xdx +ydy +zdz) = 17 (2dx +3dy +6dz). Accordingly, T (2+0.01, 30.01, 6+0.02) T (2, 3, 6)+
7 [2 0.01 + 3(0.01) + 6 0.02] = 7 + 7 0.11 7.016. (The exact value is 49.2206 7.015739.)
1 1

12. (a) dX = AN 1 et dN + AN et dt
(b) dX1 = BEX E1 N 1E dX + B(1 E)X E N E dN
14. g(0) = f (x10 , . . . , xn0 ). Using the result in part (a) of Problem 11 in Section 16.2 gives g  (0) =
f1 (x10 , . . . , xn0 ) dx1 + + fn (x10 , . . . , xn0 ) dxn . The approximation g(1) g(0) + g  (0) now yields,
using vector notation, f (x0 + dx) f (x0 ) + f1 (x0 ) dx1 + + fn (x0 ) dxn .

16.9
2. By differentiation, 3 dx1 + 2x2 dx2 dy1 9y22 dy2 = 0, 3x12 dx1 2dx2 + 6y12 dy1 dy2 = 0. Letting
dx2 = 0 and solving for dy1 and dy2 leads to dy1 = Adx1 and dy2 = Bdx1 , where A = y1 /x1 =
J 1 (3 27x12 y22 ) and B = y2 /x1 = J 1 (3x12 + 18y12 ) with J = 1 + 54y12 y22 .

Knut Sydster and Peter Hammond 2010


42 CHAPTER 17 MULTIVARIABLE OPTIMIZATION

4. Differentiation w.r.t. x yields y + ux v + uvx = 0, u + xux + yvx = 0. Solving this system for ux and
vx yields
u2 y 2 u2 y 2 xy uv 2xy 1
ux = = , vx = =
yv xu 2yv yv xu 2yv
where we substituted xu = yv and uv = 1 xy. Now,

2u  2uux 2yv (u2 y 2 )2yvx


= u =
x 2 x x 4y 2 v 2
2u(u2 y 2 ) (u2 y 2 )(2xy 1)/v (u2 y 2 )(4uv 1)
= 2 2
=
4y v 4y 2 v 3
(The answer to this problem can be expressed in many different ways.)
w z w w
6. From w = G(x, y, z) we have () ( )y = Gx + Gz ( )y . Here, ( )y,z = Gx and ( )x,y = Gz .
x x x z
Substituting these expressions into () yields the result.

16.10
2. Denote the left hand sides of the three equations by f1 , f2 , and f3 , respectively. Then the Jacobian
determinant of these functions w.r.t. u, v, and w is
   
 f1 /u f1 /v f1 /w   1 1 3w2 
  
 f2 /u f2 /v f2 /w  =  1 3v 2 1  = 2(3w2 1)
  
 f /u f /v f /w   1 1 3w 2 
3 3 3

At P the value of this determinant is 4  = 0, and the functions f1 , f2 , and f3 are C 1 functions, so according
to the implicit function theorem the system denes u, v, and w as differentiable functions of x, y, and z
in a neighborhood of P . Differentiating the system w.r.t. x (holding y and z constant) yields

ux vx 3w 2 wx = 0


ux + 3v 2 vx wx = 2
ux vx + 3w 2 wx = 2x

Substituting (x, y, z, u, v, w) = (1, 1, 0, 1, 0, 1) and solving the system yields ux = 5/2, vx = 1, and
wx = 1/2.

Chapter 17 Multivariable Optimization


17.1
2. First-order conditions: f1 (x, y) = 4x 2y + 36 = 0, f2 (x, y) = 2x 4y + 42 = 0, with the solution
x = 5, y = 8.
4. (a) P (10, 8) = P (12, 10) = 98 (b) First-order conditions: Px = 2x + 22 = 0, Py = 2y + 18 = 0.
It follows that x = 11 and y = 9, with P (11, 9) = 100.
6. Solving the constraint for z yields z = 4x + 2y 5, so we have to minimize P (x, y) = x 2 + y 2 +
(4x + 2y 5)2 w.r.t. x and y. We nd P (x, y) = 17x 2 + 5y 2 + 16xy 40x 20y + 25. The rst-order
conditions are P1 = 34x + 16y 40 = 0, P2 = 16x + 10y 20 = 0, with the solution x = 20/21,
y = 10/21. The minimum value is 525/441.

Knut Sydster and Peter Hammond 2010


CHAPTER 17 MULTIVARIABLE OPTIMIZATION 43

8. FK = 2(K 3)(L6) and FL = 4(L6)(K 3), so here [17.3] yields 2(K 3)(L6) =
0.65, 4(L 6) (K 3) = 1.2. The solution is K = 2.8, L = 5.75.

10. x = am/(a + b + c)p, y = bm/(a + b + c)q, z = cm/(a + b + c)r. (The constraint yields
z = (m px qy)/r, so we maximize P = Ax a y b [(m px qy)/r]c w.r.t. x and y. Keeping in
mind that z depends on x and y, the rst-order conditions are Px = Ax a1 y b zc1 [az c(px/r)] = 0
and Py = Ax a y b1 zc1 [bz c(qy/r)] = 0. These give px = (a/c)rz and qy = (b/c)rz. Inserted into
the constraint and solved for z, these yield the given value for z, and the other two follow easily.)

17.2
2. Use Theorem 9.2 with F (u) = ln u.

4. By the chain rule, g1 (x, y) = F  (f (x, y))f1 (x, y) and g2 (x, y) = F  (f (x, y))f2 (x, y). Because F  > 0,
the conclusion follows.

6. (a) S = {(x, y) : g(x, y) = c}, where g(x, y) = x 2 + xy + y 2 , c = 3, with g continuous.


(b) The given equation is symmetric in x and y, so 2 x 2. (c) The set is closed because
g(x, y) = x 2 + xy y 2 is continuous. The pair given in the hint does satisfy the equation, so the
conclusion follows.

17.3
2. (a) f1 (x, y) = 3x 2 9y and f2 (x, y) = 3y 2 9x. The only stationary point in the interior of S is (3, 3).
((0, 0) is a stationary point on the boundary of S.) Along the four parts of the boundary we nd the
following candidates for extreme points: (0, 0), (4, 0), (4, 12), (4, 4), ( 12, 4), and (0, 4). Comparing
the values of f at all these points, we nd that f has mimimum 91 at (0, 4) and at (4, 0), minimum 0 at
(3, 3). (A maximum and a minimum exist by the extreme-value
theorem.)
(b) Maximum 9/4 at (1/2, 3/2) and at (1/2, 3/2). Minimum 1/4 at (1/2, 0).
(c) Maximum 3 at (0, 0) and at (1, 0). Minimum 2 at (0, 1) and at (0, 1).
(d) Maximum 2e4 at (0, 0). Minimum 0 for all (x, 1/2) where x [0, 2], and for all (2, y) where
y [0, 1/2].
2  
4. (a) The domain D is shown in Fig. 17.3.4. The rst order partials are f1 (x, y) = e 4 (x+y ) 1 41 (x + y) ,
1

2  
f2 (x, y) = e 4 (x+y ) 1 21 y(x + y) . (b) The point (7/2, 1/2) gives the maximum value 4e15/16 .
1

6. (a) f1 = 9x(x 2 + y 2 )1/2 4x(x 2 + y 2 )1/2 , f2 = 9y(x 2 + y 2 )1/2 4y(x 2 + y 2 )1/2 + 1. At a stationary
point, f1 = 0 implies that x = 0 or x 2 + y 2 = 4/9. But x 2 + y 2 = 4/9 implies f2 = 1, so x = 0 at the
stationarypoint. Then f2= 9y 2 3 if y > 0, whereas f2 = 9y 2 + 5 if y < 0. The stationary points
are (0, 13 3) and (0, 13 5).
(b) S is closed and bounded and f is continuous, so extreme points exist according to the extreme-value
theorem. There are no stationary points in the interior of S, so the extreme points must be on the boundary.
Along x = 0, y [1, 1], f (0, y) = 3y 3 3y if y 0, and f (0, y) = 3y 3 + 5y if y 0. Along this
part of the boundary f has its largest value 0 at (0, 1) and at (0, 0), and its smallest value 10 5/9 at
(0, 13 5). Along x 2 + y 2 = 1, y [1, 1], f (x, y) = y 1, which has its largest value 0 at (0, 1) and
its smallest value 2 at (0, 1).
We conclude that the maximum value is 0 at (0, 1) and at (0, 0), and
that its smallest value is 10 5/9 at (0, 13 5).

Knut Sydster and Peter Hammond 2010


44 CHAPTER 17 MULTIVARIABLE OPTIMIZATION

z y y = 2x 2
z= ln(1 + x 2 y)
y 2

y
x y = x2
1 y = kx
D
y =1x
x x
1 1
Figure 17.3.4 Figure 17.4.2(f) Figure 17.4.8

17.4
2. (a) (0, 0) saddle point, (1, 1) local minimum. (b) (4/3, 1/3) local minimum.
(c) (2, 3) local maximum, (a, 0) and (0, b) no decision (a and b arbitrary
numbers).
(d) (0, 0) saddle point, (a, 0) and (a, 0) local maxima where a = 3/2.
(e) (0, 0) saddle point, (1/2, 1/4) and (1/2, 1/4) local mininma.
(f) (0, b) no decision (b an arbitrary number). (The graph of this function is shown in Fig. 17.4.2(f). f
is dened for x = 0 and for y > 1/x 2 . f1 (x, y) = 2xy/(1 + x 2 y) and f2 (x, y) = x 2 /(1 + x 2 y).
Here f1 = f2 = 0 at all points (0, b) with b . Because AC B 2 = 0 when (x, y) = (0, b), the
second-derivative test fails. But by studying the function directly, we see that (0, b) is a local maximum
point if b < 0; a saddle point if b = 0; and a local minimum point if b > 0. See Fig. 17.4.2(f).)
4. (a) The level curves are x = 1 and y = 0. g(x, y) is positive if y > 0 and x  = 1.
(b) g1 = y(x 2 1)ex+3y , g2 = (x 1)2 (1 + 3y)ex+3y . (1, 1/3) is a local minimum. (1, y0 ) is a local
minimum for y0 > 0, a local maximum for y0 < 0, and a saddle point for y0 = 0. (Use the results in (a).)
(c) g(x, 1) = (x 1)2 ex+3 as x , so g has no maximum; g(x, 1) = (x 1)2 ex3
as x , so g has no minimum either.
6. We nd easily in all three cases that (0, 0) is a stationary point where z = 0 and AC B 2 = 0. In case
(a), z 0 for all (x, y), so the origin is a maximum point. In case (b), z 0 for all (x, y), so the origin
is a minimum point. In (c), z takes positive and negative values at points arbitrarily close to the origin,
so it is a saddle point.
8. (a) Fig. 17.4.8. The domain where f (x, y) is negative is shaded. (b) (0, 0) is seen to be a stationary
point at which z = 0. As Fig. 17.4.8 shows, this function takes positive and negative values for points
arbitrary close to (0, 0), so the origin is a saddle point. (c) g(t) = f (th, tk) = (tkt 2 h2 )(tk2t 2 h2 ) =
2h4 t 4 3h2 kt 3 + k 2 t 2 , so g  (t) = 8h4 t 3 9h2 kt 2 + 2k 2 t and g  (t) = 24h4 t 2 18h2 kt + 2k 2 . Then
g  (0) = 0 and g  (0) = 2k 2 , so t = 0 is a minimum point for k  = 0. For k = 0, g(t) = 2t 4 h4 , which has
a minimum at t = 0.

17.5
2. (a) The set is convex; see Fig. 17.5.2(a). (b) The set is convex; see Fig. 17.5.2(b).
(c) The set is not convex; see Fig. 17.5.2(c). (d) The set is convex; see Fig. 17.5.2(d).
(e) The set is not convex; see Fig. 17.5.2(e). (f) The set is not convex; see Fig. 17.5.2(f). ((4, 0) and
(0, 4) belong to the set, but not 21 (4, 0) + 21 (0, 4) = (2, 2).)

Knut Sydster and Peter Hammond 2010


CHAPTER 17 MULTIVARIABLE OPTIMIZATION 45
y y y
4
3
3
2 2
1 1
x x x
-3 -2 -1 1 2 3 -4 -3 -2 -1 1 2 3 4
-1 -1
-2 -2
-3
-3
-4

Figure 17.5.2(a) Convex. Figure 17.5.2(b) Convex. Figure 17.5.2(c) Not convex.

y y y
3
3 4
2
2 1 3
x 2
1 -3 -2 -1 1 2 3
-1
x 1
-1 1 2 3 -2
x
-1 -3 1 2 3 4

Figure 17.5.2(d) Convex. Figure 17.5.2(e) Not convex. Figure 17.5.2(f) Not convex.

4. To prove that aS + bT is convex, take x aS + bT , y aS + bT , and a number [0, 1]. Because


x aS + bT , there are points x1 S and y1 T such that x = ax1 + by1 . Similarly, there are points
x2 S and y2 T such that y = ax2 + by2 . Then (1 )x + y = (1 )(ax1 + by1 ) + (ax2 + by2 ) =
a[(1 )x1 + x2 ] + b[(1 )y1 + y2 ]. This belongs to aS + bT because S and T are convex, so
(1 )x1 + x2 S and (1 )y1 + y2 T . Therefore aS + bT is convex.

17.6
2. If f is linear, then f is both concave and convex according to Example 17.12. On the other hand, if f is
both concave and convex on the convex set S, then [17.15] is satised with equality for any (0, 1)
and for all x, x0 in S. Thus the graph of f contains any line segment between two points on its graph. So
the graph of f is a hyperplane, and f is linear.

4. We prove the rst statement in [17.16]. The second statement is proved in the same way. Suppose rst
that f is concave. Take any two points (x1 , y1 ) and (x2 , y2 ) in Mf . Then y1 f (x1 ) and y2 f (x2 ). Let
[0, 1]. Then (1)(x1 , y1 )+(x2 , y2 ) = ((1)x1 +x2 , (1)y1 +y2 ). Here (1)x1 +x2 S,
because S is convex. Moreover, by the concavity of f , f ((1 )x1 + x2 ) (1 )f (x1 ) + f (x2 )
(1 )y1 + y2 . This inequality implies that (1 )(x1 , y1 ) + (x2 , y2 ) belongs to Mf , so Mf is convex.
Suppose on the other hand that Mf is convex. Take any x1 , x2 in S and let [0, 1]. Of course,
f (x1 ) f (x1 ) and f (x2 ) f (x2 ), so (x1 , f (x1 )) and (x2 , f (x2 )) both belong to Mf . Because Mf is
convex, (1 )(x1 , f (x1 )) + (x2 , f (x2 )) = ((1 )x1 + x2 , (1 )f (x1 ) + f (x2 )) Mf . This
implies that (1 )f (x1 ) + f (x2 ) f ((1 )x1 + x2 ), and so f is concave.

Knut Sydster and Peter Hammond 2010


46 CHAPTER 17 MULTIVARIABLE OPTIMIZATION

17.7
2. Part (a) remains valid iff either a or b is strictly positive. This follows immediately from the arguments
in the proof of part (a) of Theorem 17.6. (Of course, if a = b = 0, af + bg is certainly not strictly
concave.)
 It follows from the
 proof of part (c) thatif F is strictly concave, then the inequality
F f ((1 )x + x) F (1 )f (x ) + f (x) becomes strict for x0  = x, so U is strictly concave.
0 0

There is no need to assume that f is strictly concave. For part (e), suppose f and g are strictly concave
over the convex set S. Let (0, 1) and let x  = y. Then, because f and g are strictly concave,
f ((1 )x + y) > (1 )f (x) + f (y), and g((1 )x + y) > (1 )g(x) + g(y). It follows that

   
h((1 )x + y) = min{f (1 )x + (y) , g (1 )x + y }
> min{(1 )f (x) + f (y), (1 )g(x) + g(y)}

But
(1 )f (x) + f (y) (1 ) min{f (x), g(x)} + min{f (y), g(y)}
= (1 )h(x) + h(y)

Similarly, (1)g(x)+g(y) (1)h(x)+h(y). Therefore, h((1)x+y) > (1)h(x)+h(y).

17.8
      2
2. (a) f11 = 2a, f12 = 2b, f22 = 2c, and f11 f22 (f12 ) = 2a2c (2b)2 = 4(ac b2 ). The result follows
from Theorem 17.10. (b) According to Theorem 17.9, f is concave iff a 0, b 0, and ac b2 0;
f is convex iff a 0, b 0, and ac b2 0.
   
4. (a) f11 (x, y) = ex ex+y , f12 (x, y) = ex+y , and f22 (x, y) = ex+y , so f11 (x, y) < 0 and
   2
f11 f22 (f12 ) = e 2x+y > 0. Hence, f is strictly concave.
    2
(b) g is strictly convex. (g11 = ex+y + exy > 0 and g11 g22 (g12 ) = 4e2x > 0.)
     2
6. f is concave where f11 = 2 6x 0, f22 = 2 0, and f11 f22 (f12 ) = 2(2 6x) 1 0.
Here 2 6x 0 iff x 1/3 and 2(2 6x) 1 0 iff x 5/12. We conclude that the largest convex
domain on which f is concave is S = {(x, y) : x 5/12}.

8. f1 = 4x + 4, f2 = 2y + 4, f11



= 4, f12 
= 0, f22 = 2. Clearly (1, 2) is a stationary point for f
and the conditions in Theorem 17.11(a) are satised, so (1, 2) is the maximum point.

10. (a) The rms choose p and q to maximize px+qy(5+x)(3+2y) = 26p+24q5p 2 6q 2 +8pq69.
This is a strictly concave function. The maximum is given by p = 9, q = 8, x = 16, y = 4. As prot is
123, whereas Bs is 21. (b) Firm As prot is now A (p) = px 5 x = p(29 5p + 4q) 5 29 +
5p 4q = 34p 5p 2 + 4pq 4q 34, with q xed. This quadratic polynomial is maximized at p =
pA (q) = 15 (2q +17). Likewise, rm Bs prot is now B (q) = qy 32y = 28q 6q 2 +4pq 8p35,
with p xed. This quadratic polynomial is maximized at q = qB (p) = 13 (p + 7).
(c) Equilibrium occurs where p = pA (q) and q = qB (p). So 5p = 2q + 17 and 3q = p + 7. These give
p = 5, q = 4, x = 20, y = 12. A gets 75, B gets 21.
(d) The process starts at (9, 8) and converges to (5, 4). See Fig. 17.8.10.

Knut Sydster and Peter Hammond 2010


CHAPTER 17 MULTIVARIABLE OPTIMIZATION 47
q
p = pA (q)
10
8 (9, 8)
6
q = qB (p)
4
(5, 4)
2
p
2 4 6 8 10 12 14
Figure 17.8.10

17.9
2. (a) Stationary points must satisfy the three equations: f1 = 2x + 2y + 2z = 0, f2 = 4y + 2x = 0, and
f3 = 6z + 2x = 0. The only solution is (0, 0, 0). The Hessian matrix is
  
f11 f12 f13 2 2 2
   =
f21 f22 f23 2 4 0
 

f31 f33f32 2 0 6
 
  2 2 2
2 2  
The leading principal minors are 2,   = 4,  2 4 0  = 8. So (0, 0, 0) is a local minimum point.
  
2 4 2 0 6
(b) f has two stationary points, (3, 3, 3) and ( 13 y02 , y0 , y0 ), where y0 1.63 is the other real root (apart
from y = 3) of the equation y 4 54y + 81 = 0. (There are two complex roots.) The Hessian matrix is

6x 9 9

H = 9 6y 0
9
6z 0
 
 6x 9 
The leading principal minors areD1 = 6x, D2 =   = 36xy 81, D3 = |H| =
9 6y 
54(4xyz 9y 9z). At (3, 3, 3) the leading principal minors are all positive, so this is a local min-
imum point. At ( 13 y02 , y0 , y0 ), D1 > 0, D2 < 0, and D3 < 0. According to [17.27], this is a saddle point.
(c) The stationary points are the solutions of the equation system
f1 = 8x2 8x1 = 0, f2 = 20 + 8x1 12x22 = 0, f3 = 48 24x3 = 0, f4 = 6 2x4

The rst equation gives x2 = x1 , and then the second equation gives 12x12 8x1 20 = 0 with the two
solutions x1 = 5/3 and x1 = 1. The last two equations determine x3 and x4 . There are two stationary
points, (5/3, 5/3, 2, 3) and (1, 1, 2, 3). The Hessian matrix is
8 8 0 0
8 24x2 0 0
f  (x1 , x2 , x3 , x4 ) =
0

0 24 0
0 0 0 2
and the leading principal minors of the Hessian are D1 = 8, D2 = 192x2 64 = 64(3x2 1), D3 = 24D2 ,
and D4 = 48D2 . At (1, 1, 2, 3) we get D2 < 0, so this point is a saddle point. The other stationary point,
(5/3, 5/3, 2, 3), we get D1 < 0, D2 > 0, D3 < 0, and D4 > 0, so this point is a local maximum point.

Knut Sydster and Peter Hammond 2010


48 CHAPTER 17 MULTIVARIABLE OPTIMIZATION

17.10
2. If f is increasing, or decreasing, then the upper level sets {x : f (x) a} are all intervals, which are
convex sets, so f is quasi-concave.
4. Let x, x0 be any pair of vectors in S, and [0, 1]. Let z = (1 )x + x0 . Now, if f (z)
min{f (x), f (x0 )}, then f (x) f (x0 ) implies f (z) f (x0 ), as in [17.31]. Hence, Theorem 17.15
implies that f is quasi-concave.
Conversely, suppose f is quasi-concave. If f (x) f (x0 ), then [17.31] implies f (z) f (x0 ). On
the other hand, if f (x) f (x0 ), then interchanging x and x0 in [17.31] and replacing by 1 implies
that f (z) f (x). In either case, f (z) min{f (x), f (x0 )}.
6. (a) This is mainly an exercise in manipulating determinants. If you feel that the calculations below look

frightening, try to write them out in full for the case k = 3 (or k = 2). Note that zij = ai aj z/xi xj for

i  = j , and zii = ai (ai 1)z/xi . Rule 3 in Theorem 13.1 tells us that a common factor in any column
2

(or row) in a determinant can be moved outside. Therefore,


 a1 (a1 1) a 1 a2 a 1 ak 
 z 
   
 z z
  
 2 x1 x2 x1 xk
 z11 z12 . . . z1k x1
   
 z z . . . z   a2 a1 a2 (a2 1) a 2 ak 
 21 22 2k  z z z 
 
Dk =  . = 2
. . 
x2 x1 x 2 x 2 xk 
 .. .. . .. ..   .. .. .. 
    . . 
  . . . . 
  

zk1 zk2 . . . zkk  ak a1 a k a2 ak (ak 1) 
 z z z
xk x1 xk x2 xk2
 a1 1 a1 a1 

 
 x1 x1 x1  a 1 a1 
   1 a1
 a2 a2 1 a2   
(1) a1 a2 . . . ak k   (2) a1 a2 . . . ak k  a2 a2 1 a2 
= z x2 x2 x2  =  z  .. 
x1 x2 . . . xk  . (x1 x2 . . . xk )2  ..
. .. ..
. . .  . . . 
 .. .. .. ..   
  ak ak ak 1
 ak ak ak 1 
 
xk xk xk
where equality (1) holds because aj z/xj is a common factor in column j for each j and equality (2) holds
because 1/xi is a common factor in row i for each i.
 a1 a2 . . . ak k
(b) Let sk = ki=1 ai = a1 + + ak , and Pk = z . We use the expression for Dk that
(x1 x2 . . . xk )2
we found in part (a), and add rows 2, 3, . . . , k to the rst row. Then each entry in the rst row becomes
equal to sk 1. Afterwards we take the common factor sk 1 in row 1 and move it outside.
s 1 s 1 s 1  1 1 
 k k k   1
   
 a2 a2 1 a2   a2 a2 1 a2 
Dk = Pk  ..  .. .. 
..  = (sk 1)Pk  ..  .. .. .. 
 . . . .   . . . . 
   
ak ak ak 1 ak ak ak 1
Now subtract column 1 from all the other columns. Rule 7 in Theorem 13.1 says that this does not change
1 0 0 

 
 a2 1 0 

the value of the determinant, so Dk = (sk 1)Pk  .. .. .. ..  = (1)k1 (sk 1)Pk .
 . . . . 
 
ak 0 1

Knut Sydster and Peter Hammond 2010


CHAPTER 18 CONSTRAINED OPTIMIZATION 49

 
(c) By assumption, ai > 0 for all i, so if ni=1 ai < 1, then sk = ki=1 ai < 1 for all k. Therefore Dk has
the same sign as (1)k . Combining Theorem 15.3 and Theorem 17.14, it follows that f is strictly concave.

Chapter 18 Constrained Optimization


18.1
2. (a) When f (x, y) = 3xy, g(x, y) = x 2 + y 2 , and c = 8, equation [18.3] reduces to 3y/3x = 2x/2y,
and so x 2 = y 2 . Inserted into the constraint this yields x 2 = 4, and so x = 2. The solution candidates
are therefore: (2, 2), (2, 2), (2, 2), (2, 2). Here f (2, 2) = f (2, 2) = 12 and f (2, 2) =
f (2, 2) = 12. So (2, 2) and (2, 2) are the only possible solutions of the maximization problem,
and (2, 2) and (2, 2) are the only possible solutions of the minimization problem. (The extreme-value
theorem ensures that we have found the solutions, because f is continuous and the constraint curve is a
closed bounded set (a circle).)
(b) With L = x + y (x 2 + 3xy + 3y 2 3), the rst-order conditions are 1 2x 3y = 0
and 1 3x 6y = 0. From these equations we get 2x + 3y = 3x + 6y, or (3y + x) = 0.
Here = 0 is impossible, so x = 3y. Inserted into the constraint we have (3, 1) and (3, 1) as the
only possible solutions of the maximization and minimization problems, respectively. The extreme value
theorem ensures that solutions exist. (The objective function is continuous and the constraint curve is a
closed bounded set (an ellipse).)

18.2
2. (a) x = a/5, y = 2a/5 solve the problem. Solving x + 2y = a for y yields y = 21 a 21 x, and then
x 2 + y 2 = x 2 + ( 21 a 21 x)2 = 45 x 2 21 ax + 41 a 2 . This quadratic function certainly has a minimum at
x = a/5. (b) L(x, y) = x 2 + y 2 (x + 2y a). The necessary conditions are L1 = 2x = 0,
L2 = 2y 2 = 0, implying that 2x = y. From the constraint, x = a/5 and then y = 2a/5, = 2a/5.
(c) Find the point on the line x+2y = a which has the minimal distance from the origin. The corresponding
maximization problem has no solution. (See Fig. 18.2.2.)
(d) f (a) = (a/5)2 + (2a/5)2 = a 2 /5, so df (a)/da = 2a/5, which is also the value of the Lagrangean
multiplier. Equation [18.7] is conrmed.
y
(a/5, 2a/5)
a/2

a x

Figure 18.2.2

4. L(2, 2) = 2 > L(1, 1) = 1. (In fact, L(x, y) has a saddle point at (1, 1).)

Knut Sydster and Peter Hammond 2010


50 CHAPTER 18 CONSTRAINED OPTIMIZATION

6. (a) L(x, y) = 100ex ey (px +qy m), so the rst-order conditions are: (1) Lx = ex p = 0
and (2) Ly = ey q = 0. These imply that x = ln(p) = ln + ln p, y = ln + ln q. Inserting
the expressions for x and y into the constraint, and solving for ln , yields
m + q ln q q ln p
ln = (m + p ln p + q ln q)/(p + q). Therefore x(p, q, m) = and
p+q
m + p ln p p ln q
y(p, q, m) = .
p+q
tm + tq ln(tq) tq ln(tp) tm + tq(ln t + ln q) tq(ln t + ln p)
(b) x(tp, tq, tm) = =
tp + tq t (p + q)
= x(p, q, m), so x is homogeneous of degree 0. The constraint px + qy = m remains unchanged when
x, y, m are all multiplied by t, so the maximization of U is over the same constraint set in both cases.
Hence, the optimal values of x and y have to be unchanged.
y y y
8. (a) g(x, y) = 0 xt (y t) dt = 0 (xyt xt 2 ) dt = 0 ( 21 xyt 2 13 xt 3 ) dt = 16 xy 3 . Similar computations
yield f (x, y) = 16 xy 5 + 121
xy 4 + 16 xy 3 . The solution of [] is x = 384 3 M, y = 1/4. (Note that,
because 6 xy = M, the problem reduces to that of maximizing M + 21 My My 2 for y 0.)
1 3

(b) As tends to 0, the cost of extraction tends to innity. Because the sales price increases as t increases,
it is reasonable to expect that the terminal time y tends to innity. (c) The value function is f (M) =
M + M/16. The Lagrange multiplier must satisfy the equation f1 g1 = 0, or 16 y 5 + 12 y + 16 y 3
1 4

6 y = 0. Inserting the optimal values of x and y and solving for yields = 1 + 1/16. Clearly,
1 3

f (M)/M = .

18.3
2. (a) (i) The constraint implies that x = y 2 /8 0, so the problem reduces to that of minimizing h(x) =
(x 1)2 + 8x for x 0. Because h (x) = 2x + 6 6 for all x 0, the minimum value is 1 at x = 0.
(ii) The problem reduces to minimizing k(y) = ( 18 y 2 1)2 + y 2 . Here k  (y) = 2( 18 y 2 1) 41 y + 2y =
y + 23 ). Thus k  (y) < 0 for y < 0, and k  (y) > 0 for y > 0. We conclude that k(y) is minimized
1 2
y( 16
at y = 0, which gives the same solution as in (i).
(b) For the Lagrangean L = (x1)2 +y 2 (y 2 8x), the rst-order conditions are: (1) 2(x1)+8 = 0;
(2) 2y 2y = 0. From (2), either = 1 or y = 0. But inserting = 1 into (1) yields x = 3, and then
no value of y satises the constraint. So y = 0, implying the solution x = y = 0.
(c) The problem is to nd which point on the parabola y 2 = 8x is closest to (1, 0).

18.4
2. In this case, f (x, y) = x 2 + y 2 , g(x, y) = x + 2y, so
   
0 g1 g2  0 1 2 
  
   =  
D(x, y) =  g1 
f11 
g11 
f12 g12   1 2 0  = 10
   
 g 
f21 
g21 f  g    2 0 2
2 22 22

By [18.14], there is a local minimum at the stationary point (a/5, 2a/5).

18.5
2. Here L = x + 4y + 3z (x 2 + 2y 2 + 13 z2 b). So necessary conditions are:
(1) L1 = 1 2x = 0; (2) L2 = 4 4y = 0; (3) L3 = 3 23 z = 0. It follows that  = 0,

Knut Sydster and Peter Hammond 2010


CHAPTER 18 CONSTRAINED OPTIMIZATION 51

and so x = 1/2, y = 1/, z = 9/2. Inserting these values into the constraint yields
= 9/b, so
2

= 3/ b. The value of the criterion function is x + 4y + 3z = 18/,


so = 3/ b minimizes the
criterion function. The minimum point is (a, 2a, 9a), where a = b/6.

15 + h, y = 3 + k, and z = 15 + , constraints [1] and [2] reduce to h + 2k + l = 0 and


4. With x = 16 1 11

2h k 3 = 0, so k = h and  = h. But then easy algebra shows that f (x, y, z) = (16/15)2 +


(1/3)2 + (11/15)2 + 3h2 . Because 3h2 0 for all h, (16/15)2 + (1/3)2 + (11/15)2 must then be the
maximum value of f (x, y, z) subject to the constraints. The maximum value is obviously attained when
h = k =  = 0, or when (x, y, z) = (16/15, 1/3, 11/15).

6. (a) Here L = x + y + z (x 2 + y 2 + z2 1) (x y z 1). Necessary conditions are:


(1) L1 = 1 2x = 0; (2) L2 = 1 2y + = 0; (3) L3 = 1 2z + = 0. Adding (1) and
(2) yields (x + y) = 1. Adding (1) and (3) yields (x + z) = 1. It follows that  = 0 and also that
z = y. Substituting z = y in the two constraints yields: (4) x 2 + 2y 2 = 1; (5) x 2y = 1. From (5),
x = 2y + 1, so (4) gives 6y 2 + 4y = 0, with solutions y = 0 and y = 2/3. The only possible solutions
to the problem are therefore (1, 0, 0) and (1/3, 2/3, 2/3). In fact, (1, 0, 0) solves the maximization
problem, while (1/3, 2/3, 2/3) solves the minimization problem.
(b) The constraints are represented geometrically by a closed bounded curve which is the intersection
between a sphere (x 2 + y 2 + z2 = 1) and a plane (x y z = 1). The continuous function f (x, y, z) =
x + y + z must therefore attain maximum and minimum values subject to the constraints, and in (a) we
have found the extreme points.

8. Here L = a12 x12 + + an2 xn2 (x1 + + xn 1). Necessary conditions are that Lj = 2aj2 xj = 0,
j = 1, . . . , n, and so xj = /2aj2 . Inserted into the constraint, this implies that 1 = 21 (1/a12 + +1/an2 ).
Thus, for j = 1, . . . , n, we have
1 1
xj = = 2 n
aj2 (1/a12 + + 1/an2 ) aj i=1 (1/ai2 )

(Implicitly, this solution assumes that each ai  = 0. If at least one ai is 0, the minimum value is 0, which
is attained by letting a corresponding xi be 1, with the other xj all equal to 0.)

10. (a) The Lagrangean is here L = Ax1a1 xnan (p1 x1 + + pn xn m), and so
Li = ai U/xi pi = 0, i = 1, . . . , n. In particular, p1 x1 = a1 U/, and so = a1 U/p1 x1 . It
follows that pi xi = (ai /a1 )p1 x1 . Inserted into the budget constraint, we have p1 x1 + (a2 /a1 )p1 x1 +
a1 m
+ (an /a1 )p1 x1 = m, which implies that x1 = . The demand functions are given
p1 (a1 + + an )
ai m
by xi = for i = 1, . . . , n.
pi (a1 + + an )
(b) The Lagrangean is L = x1a + + xna (p1 x1 + + pn xn m), and so Li = axia1 pi = 0.
a/(1a) 1/(1a)
In particular, = ax1a1 /p1 . Algebraic manipulations yield pi xi = pi p1 x1 , i = 1, . . . , n.
Using the budget constraint equation gives an expression for x1 , and subsequently we nd the other
1/(1a)
mp
demand quantities xi . The consumers demand functions are xi = n i , for i = 1, . . . , n.
" a/(1a)
pj
j =1

Knut Sydster and Peter Hammond 2010


52 CHAPTER 18 CONSTRAINED OPTIMIZATION

12. For n = 2, the Lagrangean is L = a11 x12 + 2a12 x1 x2 + a22 x22 (x12 + x22 1). The rst-order conditions
for x1 = x10 , x2 = x20 , with = 0 to solve the problem are:

2a11 x10 + 2a12 x20 20 x10 = 0


()
2a12 x10 + 2a22 x20 20 x20 = 0

Cancelling the common factor 2 and writing () as a matrix equation yields


  
a11 a12 x10 x10
= 0
a21 a22 x20 x20

or, in obvious matrix notation, () Ax0 = 0 x0 . Thus, if x0 solves the problem, then x0 is an eigenvector
of A, with 0 as an eigenvalue. Multiplying () from the left by x0 = (x10 , x20 ) then implies that
x0 Ax0 = x0 (0 x0 ) = 0 x0 x0 = 0 , because x0 x0 = 1. We conclude that the maximum value of x Ax
subject to x x = 1 must be the largest eigenvalue of A, whereas the minimum value must be the smallest
eigenvalue. Both a maximum and a minimum value do exist because of the extreme-value theorem.
The argument in the general case of n variables is entirely similar. In fact, equating the partials of
the Lagrangean to 0 leads to a system of equations like (), which is then easily seen to reduce to ()

18.6
2. (a) The Lagrangean is L = 4z x 2 y 2 z2 (z xy). The rst-order conditions are 2x + y = 0,
2y + x = 0, 4 2z = 0, and z = xy. The only triples (x, y, z) that satisfy these conditions are
(0, 0, 0) with = 4, (1, 1, 1) with = 2, and (1, 1, 1) with = 2. (b) f 0.1 = 20.1 = 0.2

18.7
m
2. If L(x, c) = f (x) j =1 j (gj (x) cj ), then L/cj = j , so [18.24] follows from [18.31].

18.8
2. (a) Writing the constraints as g1 (x, y) = x + ex y 0 and g2 (x, y) = x 0, the Lagrangean is
L = 21 x y 1 (x + ex y) 2 (x). The KuhnTucker conditions are then:

L 1
(1) = 1 (1 ex ) + 2 = 0
x 2
L
(2) = 1 + 1 = 0
y
(3) 1 0 (= 0 if x + ex < y)
(4) 2 0 (= 0 if x > 0)

From (2) and (3), we conclude that 1 = 1 and so x + ex = y.


Either x = 0 or x > 0. In the latter case, (4) implies 2 = 0. Then (1) implies 1
2 (1 ex ) = 0,
orex = 21 . Hence x = ln 2, and so y = x + ex = ln 2 + 21 .
If x = 0, then (1) implies 2 = 21 , which contradicts 2 0.

Knut Sydster and Peter Hammond 2010


CHAPTER 18 CONSTRAINED OPTIMIZATION 53

We conclude that (x, y) = (ln 2, ln 2 + 21 ) is the only point satisfying the KuhnTucker conditions.
(By sketching the constraint set and studying the level curves 21 x y = c, it is easy to see that the point
we found solves the problem.)
(b) With the Lagrangean L = x 2 + 2y 1 (x 2 + y 2 ) 2 (y), the KuhnTucker conditions are:

L
(1) = 2x 21 x = 0
x
L
(2) = 2 21 y + 2 = 0
y
(3) 1 0 (= 0 if x 2 + y 2 < 5)
(4) 2 0 (= 0 if y > 0)

Note that the second constraint cannot be active, because if y = 0, then (2) implies 2 = 2, which
contradicts (4). So y > 0, and from (4), 2 = 0. Then from (2), 1 y = 1. Hence 1 > 0, and so from
(3), x 2 + y 2 = 5. Also, (1) evidently implies that x = 0 or 1 = 1.

If x = 0, then x 2+ y 2 = 5 implies y = 5.With y > 0, only y = 5 is possible. Then (2)
implies that 1 = 1/ 5. Thus (x, y, 1 , 2 ) = (0, 5, 1/ 5, 0) satises (1) to (4) and is a solution
candidate.
If 1 = 1, then 1 y = 1 implies y = 1 and from x 2 +y 2 = 5 it follows that x = 2. Thus (2, 1, 1, 0)
and (2, 1, 1, 0) are both solution candidates.

Now, f (0, 5) = 2 5, while f (2, 1) = f (2, 1) = 6. The constraint set is closed and bounded
(as a half-disk), so the extreme-value theorem applies. Thus, (2, 1) and (2, 1) both solve the problem,
with 1 = 1 and 2 = 0.

4. (a) The Lagrangean is L = x + ay (x 2 + y 2 1) + (x + y). There must exist numbers and


such that: (1) 1 2x + = 0; (2) a 2y + = 0; (3) 0 (= 0 if x 2 + y 2 < 1);
(4) 0 (= 0 if x + y > 0).

(b) For a 1, the point(1/ 1 +a 2 , a/ 1 + a 2 ) solves the problem, with = 21 1 + a 2 , = 0.
For a < 1, the point (1/ 2, 1/ 2) solves the problem, with = (1 a)/2 2, = 21 (1 + a).

6. The Lagrangean is L = ln(x 2 + 2y) 21 x 2 y 1 (2 xy) + 2 (x 1) + 3 (y 1). The KuhnTucker


conditions are that there must exist numbers 1 0, 2 0, and 3 0 such that

2x
(1) L1 (x, y) = x + 1 y + 2 = 0
x 2 + 2y
2
(2) L2 (x, y) = 2 1 + 1 x + 3 = 0
x + 2y
(3) 1 (2 xy) = 0
(4) 2 (1 x) = 0
(5) 3 (1 y) = 0

(We have deliberately formulated the conditions in a slightly different way than was done for Problems
2 and 4. See [18.37] in the main text.)

Knut Sydster and Peter Hammond 2010


54 CHAPTER 18 CONSTRAINED OPTIMIZATION

Suppose rst that 1 = 0. Then (1) and (2) imply


2x 2
x + 2 = 0, 1 + 3 = 0
x 2 + 2y x 2 + 2y

Because x and y are both 1, we get 3 = 1 2/(x 2 + 2y) 1 23 > 0 and 2 = x 2x/(x 2 + 2y) =
x3 > 0. It follows from (4) and (5) that x = y = 1, which contradicts xy 2. Thus 1 > 0 and so
xy = 2.
Suppose next that 2 > 0. Then x = 1 (because of (4)) and y = 2/x = 2, so 3 = 0. From (2) it
follows that 1 = 3/5. Then (1) implies that 2 = 3/5, contradicting 2 0. Hence, 2 = 0.
Suppose now that 3 > 0. Then (5) implies y = 1, and so x = 2/y = 2. From (1) it follows that
1 = 4/3. Then, from (2), 3 = 1/3 + 1 8/3 < 0, which is impossible. Thus 3 = 0.
So far we have proved that 2 = 3 = 0, 1 > 0, and xy = 2. Equations (1) and (2) now yield
2x 2
= x 1 y, = 1 1 x
x2 + 2y x2 + 2y

It follows that x 1 y = x(1 1 x). But 1 > 0 and so y = x 2 . Because xy = 2, we must have x 3 = 2.
Hence, x = 21/3 and y = 2x 1 = 22/3 is the only point which satises the KuhnTucker conditions.
(The corresponding nonnegative multipliers are 1 = 21/3 13 , 2 = 3 = 0.)
Why is this a maximum? In the feasible set, x 2 +2y 3, and f1 (x, y) and f2 (x, y) are both negative.
For a given value of x, f (x, y) decreases as y increases, and for a given value of y, f (x, y) decreases as
x increases. It follows that the constrained maximum lies on the part of the curve xy = 2 between the
two points (1, 2) and (2, 1). (See Fig. 18.8.4.) This is a closed and bounded set, so the extreme-value
theorem implies that f (x, y) has a maximum on this part of the curve. This point must be a maximum
point for f over the whole feasible set.
y

1
(1, 1)

x
1 2 3 4

Figure 18.8.4

18.9
2. The Lagrangean is L = 9x + 8y 6(x + y)2 1 (x 5) 2 (y 3) 3 (x + 2y 2) and the
KuhnTucker conditions are:

(1) L1 = 9 12(x + y) 1 + 3 0 (= 0 if x > 0)

Knut Sydster and Peter Hammond 2010


CHAPTER 18 CONSTRAINED OPTIMIZATION 55

(2) L2 = 8 12(x + y) 2 23 0 (= 0 if y > 0)


(3) 1 0 (= 0 if x < 5)
(4) 2 0 (= 0 if y < 3)
(5) 3 0 (= 0 if x + 2y < 2)

We go through the systematic procedure explained at the end of Section 18.8. (Shortcuts are possible.
For instance, one could start by proving that x = 5 is impossible, then that y cannot be positive, etc.)
x < 5, y < 3, x + 2y < 2. Then 1 = 2 = 3 = 0. If x = 0, then from (1), y 3/4 > 0, so from
(2), 8 12y = 0. Thus y = 2/3, which contradicts y 3/4. So x > 0. Then (1) implies 9 = 12(x + y).
If y > 0, then (2) implies 8 = 12(x + y), a contradiction. Thus y = 0. But then 9 = 12x, so x = 3/4.
So a solution candidate is (x, y) = (3/4, 0), with 1 = 2 = 3 = 0.
x = 5, y < 3, x + 2y < 2. Then 2 = 3 = 0 and (1) implies 1 = 9 12(5 + y) < 0, a contradiction.
x < 5, y = 3, x + 2y < 2. Then 1 = 3 = 0 and (2) implies 2 = 8 12(x + 3) < 0, a contradiction.
x < 5, y < 3, x + 2y = 2. Then 1 = 2 = 0 and 2y = 2 + x > 0, so y > 0 and (2) imply
8 12(x + y) = 23 . It follows that 3 = 4 6[x + 21 (2 + x)] = 2 9x < 0, a contradiction.
x < 5, y = 3, x + 2y = 2. Then x = 4 and 1 = 0. Now (1) gives 3 = 75, and then (2) gives
2 = 226, contradicting 2 0.
x = 5, y < 3, x + 2y = 2. Then the rst and second equalities yield y = 3.5, which contradicts y < 3.
x = 5, y = 3, x + 2y < 2. Then 3 = 0. Now (1) gives 1 = 87, contradicting 1 0.
x = 5, y = 3, x + 2y = 2. Obviously impossible.
We conclude that the only admissible pair satisfying (1) to (5) is (x, y) = (3/4, 0). The extreme-value
theorem implies that we have found the solution. (Theorem 18.4 of the next section also shows optimality.)

m 
m
4. Using (a) in Theorem 18.3 with x = x implies that f (x0 ) j gj (x0 ) f (x) j gj (x). But,
j =1 j =1
because x also solves [18.41], f (x) = f (x0 ). Thus, because j 0 and gj (x) cj , j = 1, . . . , m,
m m 
m 
m
and also because of [18.45], we have () j cj j gj (x) j gj (x0 ) = j cj . Here the
j =1 j =1 j =1 j =1
two middle terms, being squeezed between two equal numbers, must themselves be equal. Therefore

m 
m 
m
f (x) j gj (x) = f (x0 ) j gj (x0 ) f (x) j gj (x) for all x 0. Also, if k > 0 and
j =1 j =1 j =1

m
gk (x) < ck for any k, then j (gj (x) cj ) < 0, which contradicts (). Thus x satises (a) and (b) in
j =1
Theorem 18.3.
6. (a) As suggested by the hint, L(x , ) L(x , ) for all 0 implies that

"
m "
m
(1) j (gj (x ) cj ) j (gj (x ) cj )
j =1 j =1


for all 0. If gk (x ) > ck for any k, then jm=1 j (gj (x ) cj ) can be made arbitrarily large by
choosing k large and j = 0 for all j  = k. Hence, gj (x ) cj , j = 1, . . . , m. Now, however, because

Knut Sydster and Peter Hammond 2010


56 CHAPTER 18 CONSTRAINED OPTIMIZATION


one can have j = 0, j = 1, . . . , m, (1) implies that mj =1 j (gj (x ) cj ) 0. Yet j 0 and

 m
gj (x ) cj , j = 1, . . . , m, so j =1 j (gj (x ) cj ) = 0.
Finally, whenever gj (x) cj , j = 1, . . . , m, the inequality L(x, ) L(x , ) implies that
"
m "
m
f (x) f (x ) j [gj (x) gj (x )] = j [gj (x) cj ] 0
j =1 j =1

Therefore f (x) f (x ), so x solves [18.41].


(b) First, part (b) of the question should be corrected to read as follows:
Suppose that there exist x 0 and 0 satisfying both gj (x ) cj and gj (x ) = cj whenever
j> 0 for j = 1, . . . , m, as well as L(x, ) L(x , ) whenever x 0. Show that L(x, ) has a
saddle point at (x , ) in this case.
Then an answer is as follows: The rst inequality in [] is satised by assumption. As for the second,
under the indicated hypotheses, one has
"
m "
m
L(x , ) L(x , ) = (j j )[gj (x ) cj ] j [gj (x ) cj ] = 0
j =1 j =1

whenever 0. Thus, [] has been veried.


y

P
x

2x  y  c 2
2x  y  c 1

Figure 18.10.2

18.10
2. (a) As shown in Fig. 18.10.2, the feasible set is the intersection of the two circular disks with the non-
negative orthant. The solution is at the point P , where the circles intersect. Thus, (x + 1)2 + y 2 = 4 =

x 2 + (y + 1)2 . It follows that x = y and so 2x 2 + 2x + 1 = 4. The (relevant) positive root is 2 ( 7 1),


1
 
so the solution is 21 ( 7 1), 21 ( 7 1) . (b) The KuhnTucker conditions are:

(1) 2 1 2(x + 1) 2 2x 0 (= 0 if x > 0)


(2) 1 1 2y 2 2(y + 1) 0 (= 0 if y > 0)
(3) 1 0 (= 0 if (x + 1)2 + y 2 < 4)
(4) 2 0 (= 0 if x 2 + (y + 1)2 < 4)

The objective function is linear and thus concave. The constraint functions are both sums of convex
functions, so they are convex. The conditions (1) to (4) are therefore sufcient for an admissible pair to

Knut Sydster and Peter Hammond 2010


CHAPTER 18 CONSTRAINED OPTIMIZATION 57

solve the problem.
With x = y = 21 ( 7 1), equations (1) to (4) are satised with 1 = (7 + 3 7)/28
and 2 = (3 7 7)/28.
(c) The change in the optimal value of 2x + y is approximately 2 0.1 0.003.
4. (a) If x = a, f  (x ) 0; if x (a, b), f  (x ) = 0; if x = b, f  (x ) 0.
(b) See Figs. 18.10.4.

y y y

x* a b x a x* b x a x* b x
(a) (b) (c)
Figure 18.10.4

6. (a) See Fig. 18.10.6. The optimal solution is obviously at (x, y) = (1, 0). At this point the two constraints
g(x, y) = y (1 x)3 0 and h(x, y) = y 0 are both binding. The gradient vector of g is
(g1 , g2 ) = (3(1 x)2 , 1) = (0, 1) at (1, 0), and the gradient of h is (h1 , h2 ) = (0, 1). Hence, the
two gradients are linearly dependent at (1, 0) (in fact, (0, 1) = (1)(0, 1)), and so the constraint
qualication is violated at this point.
y

y  (1  x)3

1 x
3x  y  c
Figure 18.10.6

(b) The Lagrangean is L = 3x + y 1 (y (1 x)3 ) + 2 x + 3 y, so the KuhnTucker conditions are:

(1) L1 = 3 31 (1 x)2 + 2 = 0


(2) L2 = 1 1 + 3 = 0
(3) 1 0 (= 0 if y < (1 x)3 )
(4) 2 0 (= 0 if x > 0)
(5) 3 0 (= 0 if y > 0)

Suppose x > 0. Then from (4), 2 = 0, and (1) implies 1 (1 x)2 = 1. Thus 1  = 0, so 1 > 0 and
then from (3), y = (1 x)3 . From 1 (1 x)2 = 1 it follows also that x  = 1, so y = (1 x)3  = 0, and

Knut Sydster and Peter Hammond 2010


58 CHAPTER 19 LINEAR PROGRAMMING

hence y > 0. So from (5), 3 = 0. Then (2) yields 1 = 1. Therefore (1) implies that (1 x)2 = 1. So
x = 2, because x > 0. But for x = 2, one has y = (1 x)3 = 1, a contradiction.
Suppose x = 0. Then (1) implies 3 31 + 2 = 0, so 31 = 3 + 2 > 0. Thus, from (3),
y = (1 x)3 = 1, and so by (5), 3 = 0. Then from (2), 1 = 1. So all the conditions (1)(5) are
satised by x = 0, y = 1, with 1 = 1 and 2 = 3 = 0.
So the KuhnTucker conditions have a unique solution, which is not optimal. Finally, the two
constraints g(x, y) = y (1 x)3 0 and k(x, y) = x 0 are both binding at (0, 1). But (g1 , g2 ) =
(3(1 x)2 , 1) = (3, 1) and (k1 , k2 ) = (1, 0) are linearly independent. So the constraint qualication
is satised at (0, 1), where the KuhnTucker conditions are met. The KuhnTucker conditions can be
violated at the optimum (1, 0), because the constraint qualication is not satised at that point.

Chapter 19 Linear Programming


19.1
2. (a) No maximum exists. Consider Fig. 19.1.2. By increasing c, the dashed level curve x1 + x2 = c for the
criterion function moves to the north-east and so this function can obviously take arbitrary large values.
(b) Yes, the maximum is at P = (1, 0). The level curves are the same as in (a), but the direction of
increase is reversed.

x2 y
x1  x2  1
4
x1  x2  c x1  3x2  3
3 1000 700x  1000y  c

1
P

1 2 3 4 5 6 x1 1000 2000 x

Figure 19.1.2 Figure 19.1.4

4. The LP problem is:


3x + 5y 3900
() max 700x + 1000y subject to x + 3y 2100 x 0, y 0

2x + 2y 2200

The problem is represented geometrically in Fig. 19.1.4. The solution is obviously at P , where the two lines
3x + 5y = 3900 and 2x + 2y = 2200 intersect. Solving these equations yields x = 800 and y = 300. So
the rm should produce 800 TV sets of type A and 300 sets of type B.

19.2
#
3u1 + u2 3
2. (a) min 6u1 + 4u2 subject to u1 0, u2 0
2u1 + 4u2 4
#
x1 + 2x2 10
(b) max 11x1 + 20x2 subject to x1 0, x2 0
3x1 + 5x2 27

Knut Sydster and Peter Hammond 2010


CHAPTER 19 LINEAR PROGRAMMING 59

19.3
#
10x1 + 25x2 10 000
2. max 300x1 + 500x2 subject to x1 0 , x2 0
20x1 + 25x2 8 000
The solution can be found graphically. It is x1 = 0, x2 = 320, and the value of the criterion function is
160, 000, the same value found in Example 19.2 for the optimal value of the primal criterion function.

19.5
2. (a) Figure 19.5.2 shows a portion of the feasible set and two dashed level curves for the criterion function
Z = y1 + 2y2 . We see that the minimum is attained at the point (y1 , y2 ) = (3, 2).
(b) The dual is: x +x x + x 1 1 2 3 4
max 15x1 + 5x2 5x3 20x4 subject to 6x1 + x2 + x3 2x4 2
x1 0, x2 0, x3 0, x4 0

Because y1 and y2 are both positive, the rst two constraints are satised with equality. Furthermore,
the two last constraints in the primal problem are satised with strict inequality at the optimum. Hence
x3 = x4 = 0. Thus we have x1 + x2 = 1, 6x1 + x2 = 2, so x1 = 1/5 and x2 = 4/5. The maximum is
thus at (x1 , x2 , x3 , x4 ) = (1/5, 4/5, 0, 0).
(c) If the constraint is changed to y1 + 6y2 15.1, the solution to the primal (P) is still at the intersection
of the lines (1) and (2) in Fig. 19.5.2, but with (1) shifted up slightly. The solution to the dual is completely
unchanged. The optimal values in (P ) and (D) both increase by (15.1 15) x1 = 0.02.

y2
(4)

10

(3)

5
(2) y1 + 2y2 = Z0
(3, 2)
(1)
y1
5 10 15

Figure 19.5.2

4. The dual to problem [1] is


1

1 q1 + 21 q2 + r 1 1

2 q1 + 2 q2
1 2 +r 2 1
[2] min q1 V1 + q2 V2 + r 1 x 1 + + r N x N s.t. . ..

.. .

N
1 q1 + 2N q2 +r N 1

where the variables are all 0. The implications [i] to [vi] are all special cases of the complementary slackness
conditions [19.13] and [19.14] in Theorem 19.4.

Knut Sydster and Peter Hammond 2010


60 CHAPTER 20 DIFFERENCE EQUATIONS

Chapter 20 Difference Equations


20.1
2. (a) Monotone convergence to x from below. (b) Damped oscillations around x .
(c) Monotonically increasing towards . (d) Explosive oscillations around x .
(e) xt = x for all t. (f) Oscillations around x with constant amplitude.
(g) Monotonically (linearly) increasing towards . (h) Monotonically (linearly) decreasing towards .
(i) xt = x0 for all t.
4. (a) Because the parameters are positive, yk+1 is positive provided that yk > 0. Because y0 is positive, by
induction, yt is positive for all t. (b) Substituting yt = 1/xt gives the equation xt+1 = (a/c)xt + b/c.
When a = 2, b = 3, and c = 4, this equation reduces
to xt+1 = (1/2)x
1 t + 3/4. When x0 = 1/y0 = 2,
the solution is xt = (1/2)t+1 + 3/2, and so yt = (1/2)t+1 + 3/2 . Clearly, yt 2/3 as t .
Wt+2 Pt+1 + Wt+1
6. (a) From [1], = = , and [2] follows immediately.(b) According to [2], the
Wt+1 Pt + Wt
fraction Wt+1 /( + Wt ) is the same for all t. For t = 0 it is equal to W1 /P0 = c, and thus [3] follows.
From [20.4], the solution is
 c  c
Wt = W0 (c)t + (1  = c)
1 c 1 c

c
(c) The equation is stable iff |c| < 1. In this case, Wt as t .
1 c

20.2
2. According to [4] in Example 20.5, the yearly repayment is

0.07 100, 000


z= 8058.64
1 (1.07)30

In the rst year the interest payment is 0.07B = 7000, and so the principal repayment is 8058.64
7000 = 1058.64. In the last year, the interest payment is 0.07b29 8058.64[1 (1.07)1 ] 527.20,
and so the principal repayment is 8058.64 527.20 = 7531.44.

20.4
2. xt = A + B t is a solution: xt+2 2xt+1 + xt = A + B (t + 2) 2[A + B (t + 1)] + A + B t =
A + Bt + 2B 2A 2Bt 2B + A + B t = 0. Substituting t = 0 and t = 1 in xt = A + B t yields
A = x0 and A + B = x1 , with solution A = x0 and B = x1 x0 . So xt = A + B t is the general solution
of the given equation.
4. xt = (A + B t)2t + 1 is a solution:
xt+2 4xt+1 + 4xt = [A + B (t + 2)]2t+2 + 1 4{[A + B (t + 1)]2t+1 + 1} + 4[(A + B t)2t + 1] =
4A2t + 4Bt2t + 8B2t + 1 8A2t 8Bt2t 8B2t 4 + 4A2t + 4Bt2t + 4 = 1. Substituting t = 0
and t = 1 in xt = A 2t + B t2t + 1 yields A + 1 = x0 and 2A + 2B + 1 = x1 , with solution A = x0 1
and B = 21 x1 x0 + 21 . So xt = A 2t + B t2t + 1 is the general solution of the given equation.

Knut Sydster and Peter Hammond 2010


CHAPTER 20 DIFFERENCE EQUATIONS 61

20.5
2. (a) The characteristic equation m2 + 2m + 1 = (m + 1)2 = 0 has the double root m = 1, so the
general solution of the homogeneous equation is xt = (C1 + C2 t)(1)t . We nd a particular solution by
inserting ut = A 2t . This yields A = 1, and so the general solution of the inhomogeneous equation is
xt = (C1 + C2 t)(1)t + 2t .
(b) By using the method of undetermined coefcients to determine the constants A, B, and C in the
particular solution ut = A5t + B cos 2 t + C sin 2 t, we obtain A = 41 , B = 10
3
, and C = 10
1
. So the
general solution to the given equation is xt = C1 + C2 2 + 4 5 + 10 cos 2 t + 10 sin 2 t.
t 1 t 3 1


4. The characteristic equation is m2 4(ab+1)m+4a 2 b2 = 0, with solutions m1,2 = 2(ab+1 1 + 2ab ).
The general solution is therefore Dn = C1 mn1 + C2 mn2 .

6. Inserting xt = ut (a/2)t into [20.21], assuming that b = 41 a 2 , we obtain the equation


xt+2 + axt+1 + 41 a 2 xt = ut+2 (a/2)t+2 + aut+1 (a/2)t+1 + 41 a 2 ut (a/2)t
= 41 a 2 (a/2)t (ut+2 2ut+1 + ut ), which is 0 if ut+2 2ut+1 + ut = 0. The general solution of this
equation is ut = A + B t, so xt = ut (a/2)t = (A + B t)(a/2)t , which is the result claimed for case
2 in the frame on page 751.

8. (a) The rst two equations state that consumption and capital are proportional to the net national product
in the previous period. The third equation states that net national product, Yt , is divided between con-
sumption, Ct , and net investment, Kt Kt1 .
(b) To derive a second-order difference equation, rst replace t by t + 2 in the last displayed equation in
the problem to obtain Yt+2 = Ct+2 + Kt+2 Kt+1 . But Ct+2 = cYt+1 , Kt+2 = Yt+1 , and Kt+1 = Yt ,
so we obtain Yt+2 (c + )Yt+1 + Yt = 0. Explosive oscillations occur when (c + )2 < 4 and
> 1.

10. From the quadratic formula, m2 + am + b has real zeros iff b 41 a 2 . Combining [20.27] and [20.28],
we must prove that f (m) = am2 + bm + c = 0 has both roots in the interval (1, 1) iff |a| < 1 + b and
b < 1.
For these roots to be in the interval (1, 1), it is necessary that f (1) > 0, f (1) > 0, f  (1) < 0,
and f  (1) > 0. Thus 1 a + b > 0, 1 + a + b > 0, 2 + a < 0, and a + 2 > 0, which is equivalent to
|a| < 1 + b and |a| < 2. But then, because the roots are real, b 41 a 2 < 1.
Conversely, if |a| < 1 + b and b < 1, then f (1) > 0, f (1) > 0, and |a| < 2. So f  (1) =
2 + a < 0 and f  (1) = 2 + a > 0.

12. (a) Because u(1) (2)


t and ut are two linearly independent solutions of the homogeneous (!) equation, Aut +
(1)

Bu(2)
t is the general solution of the homogeneous equation. So, to prove that [] is the general solution of
 ci u(2)
t1
(2)  ci ui+1
t1 (1)
[], it sufces to prove that Ut = u(1)
t Di+1 + ut
i+1
Di+1 is a particular solution of []. Insertion
i=0 i=0
in [], followed by tedious algebraic computation, eventually yields the result.
"t1 ti1
r2 r1ti1
(b) pt = Ar1t + Br2t + ui+2 , t = 0, 1, . . .
r2 r 1
i=0
(First write the equation in the form pt+2 + 1 pt+1 + 2 pt = ut+2 , t = 0, 1, 2, . . .. Assuming that r1 , r2
are real(!) and different roots of the characteristic polynomial, two linearly independent solutions of the
homogeneous equation are u(1) t (2)
t = r1 and ut = r2 . So Di+1 = r1 r2 r1 r2
t i+1 i+2 i+2 i+1
= r1i+1 r2i+1 (r2 r1 ).

Knut Sydster and Peter Hammond 2010


62 CHAPTER 21 DIFFERENTIAL EQUATIONS

Then
"
t1
ui+2 r2i+1 "
t1
ui+2 r1i+1 "
t1 ti1
r r ti1
r1t + r2t = 2 1
ui+2
r1i+1 r2i+1 (r2 r1 ) r1i+1 r2i+1 (r2 r1 ) r2 r 1
i=0 i=0 i=0

is the particular solution in [] to the nonhomogeneous equation.)

Chapter 21 Differential Equations


21.1
2. (a) If x = Ct 2 , then x = 2Ct, and so t x = 2Ct 2 = 2x. (b) x = Ct 2 passes through (1, 2) if 2 = C 12 ,
so C = 2. Hence, x = 2t 2 is the desired solution.
 t
4. (a) Replace the problem by: If x = x(t) satises x 2 = 2 1 t 2 , then x = .
x 1 t2
(The original problem
 makes no sense. The differential equation only makes sense if t (1, 1),

and then x + 2 1 t =0 is only satised for x =
2 2
 0.) Differentiating each side of x = 2 1 t
2 2

w.r.t. t yields 2x x = 2t/ 1 t 2 , and so x = t/x 1 t 2 .


(b) Differentiating each side yields 21 et 2t ex x(x + 1) + ex x = 0, and the result follows.
2

(c) Differentiating each side yields x 2 + 2(1 t)x x = 3t 2 . Simplifying and using
(1 t)x 2 = t 3 yields the given equation.

6. The point of this problem is that you do not have to solve the differential equation (which would involve
the inverse tangent function, anyway). From x = (1+x 2 )t, we see at once that x < 0 for t < 0 and x > 0
for t > 0. Thus t = 0 is a global minimum point for x(t), and because x(0) = 0, one has x(t) 0 for all
t. Differentiating the given equation w.r.t. t yields x = 2x xt + (1 + x 2 ) = 2xt (1 + x 2 )t + (1 + x 2 ) =
(1 + x 2 )(2xt 2 + 1). Clearly x > 0 for all t, so x = x(t) is convex. (In fact, the equation does not have a
solution on all of , so x(t) only has a minimum on an open interval around 0.)

21.2
2. See Fig. 21.2.2. (The solutions are the circles t 2 + x 2 = C, t = 0, with C an arbitrary nonnegative
constant. This can be shown by direct differentiation, or by solving the separable equation x = t/x
using the method set out in the next section.)

t 2 + x2 = 4
1
t
1

Figure 21.2.2

Knut Sydster and Peter Hammond 2010


CHAPTER 21 DIFFERENTIAL EQUATIONS 63

21.3
2. (a) Direct integration yields
 x = 41 t 4  21 t 2 + C. (b) x = tet et 21 t 2 + C (c) ex (dx/dt) = t + 1, or
ex dx = (t +1) dt, so that ex dx = (t +1) dt and ex = 21 t 2 +t +C. The solution is x = ln( 21 t 2 +t +C).
4. In both cases N depends on both N and t. (In the rst gure, for instance, N (t1 ) = N (t2 ), but N (t1 )  =
N(t2 ).)
6. (a) x = Ct a (b) x = Ct b eat (c) x = Cbt b /(1 aCt b )
8. If z = x/t, then x = zt, so x = zt + z, and the equation x = g(x/t) becomes the separable equation
x3 + t 3 1x 1  x 2
t z = g(z) z. The special equation is x = = + . The suggested substitution leads
3tx 2 3t 3 t 
dz 1 dt
to t z = 13 z2 23 z. Separating the variables and integrating yields 2
= . The integral
 z 2z 3 t
z2 dz
on the LHS is . It can be found by introducing the new variable u = 1 2z3 , implying that
1 2z3 
du = 6z2 dz. The nal answer is x = 3 21 t 3 + Ct .

21.4
x t
2. (a) Because g( ) = 1/ , f (s) = s, x0 = 1, and t0 = 2, formula [21.3] yields d = s ds,

1
x t 2
or 1 21 2 = 2 21 s 2 , or 21 x 2 21 = 21 t 2 1. Thus, the required solution is x = t 2 1. (Actually,
it is probably easier to nd the general solution x 2 t 2 = C rst, and then determine the appropriate
constant.) x t x t
(b) Formula [21.3] gives 0 ( + 1)2 d = 0 e2s ds, so 0 1/( + 1) = 21 0 e2s , implying that
1 e2t
1 1/(x + 1) = 21 (1 e2t ). Hence, x = .
1 + e2t
4. Using the given identity, the separable differential equation [1] becomes
  
1 y 1 dx
+ dy =
y 1 y  x

Integration yields ln y (1/) ln |1 y  | = ln x + C1 . Multiplying both sides by  yields


ln y  ln |1 y  | = ln x  + C1 , or

y y
ln = ln eC1  x  , hence = Cx  with C = eC1 
|1 y  | 1 y 

Putting = 1/C and solving for y yields [2].

21.5
2. x = Cet/2 + 21 . The equilibrium state x = 1/2 is stable. See Fig. 21.5.2.
 
4. dx/dt = b ax, so dx/(b ax) = dt + A for some constant A. This implies that (1/a) ln |b
ax| = t + A, so ln |b ax| = at aA, and |b ax| = eataA = eat eaA . It follows that ()
b ax = eataA = eaA eat = Ceat for C = eaA . The conclusion follows by solving ()
for x.

Knut Sydster and Peter Hammond 2010


64 CHAPTER 21 DIFFERENTIAL EQUATIONS

1 x = 21 et/2 + 1
2 (C = 21 )

t
x = et/2 + 1
2 (C = 1)

Figure 21.5.2

6. From x = X/N , by logarithmic differentiation, x/x = X/X N /N . Moreover, [2] implies that
X/X = a N /N, so x/x = (a 1)N /N = (a 1)[ (1/x)]. It follows that the differential equation
for x is x = (a 1)x (a 1). The solution of this equation is x(t) = [x(0) /]e(a1)t + /.
Then [2] and x = X/N together imply that N (t) = [x(t)/A]1/(a1) , X(t) = A[N (t)]a . For 0 < a < 1,
x(t) /, N(t) (/A)1/(a1) , and X(t) A(/A)a/(a1) as t .

21.6
2. Formula [21.12] yields
 t t  t
2 s 2
x= s(1 + s 2 )e s 2 d
ds = s(1 + s 2 )et ds
0 0
2
By substituting u = t 2 , then integrating by parts, we eventually derive the solution x(t) = et 1 21 t 2 .
It follows that limt x(t) does not exist because x(t) .
4. Multiply the given equation by x n to obtain x n x = Q(t)x 1n + R(t). From z = x 1n we nd
z = (1 n)x n x, so the new equation is 1n
1
z = Q(t)z + R(t), which is a linear differential equation
in the unknown function z.
6. Usingthe technique suggested by Problem 4 yields the solution 
1/(1b)
K = Ce(1b)t + Ana0 (1 b)e(av+)t /(av + + (1 b))

21.7
2. (a) x = (x 1)(x + 1)2 . Here x = 1 is unstable, whereas x = 1 is neither stable nor unstable. (It
is stable on the right, but unstable on the left.) See Fig. 21.7.2(a). (b) No equilibrium states. See Fig.
21.7.2(b). (c) The only equilibrium state x = 0 is unstable. See Fig. 21.7.2(c).

x x x
2 2 2
1 1 1

2 2 x 1 1 x 3 2 1 x
1 1 1
2 2 2

Figure 21.7.2(a) Figure 21.7.2(b) Figure 21.7.2(c)

Knut Sydster and Peter Hammond 2010


CHAPTER 21 DIFFERENTIAL EQUATIONS 65

4. (a) x(t) = (1 + Aet )/(1 Aet ), where A = (x0 1)/(x0 + 1). Another solution is x(t) = 1 for all t.
For x0 < 1, x(t) 1 as t . For x0 > 1, which occurs when 0 < A < 1, one has x(t) as
t ( ln A) , and x(t) as t ( ln A)+ . For x0 = 1, x(t) 1. See Fig. 21.7.4(a) for some
integral curves. (b) x = 1 is stable; x = 1 is unstable. See Fig. 21.7.4(b).

x x

2
x1

1
t
x 1
2 1 1 2 x

Figure 21.7.4(a) Figure 21.7.4(b)

21.8
2. (a) It is easy to verify by direct differentiation that u1 = et and u2 = tet both satisfy x 2x + x = 0. If
tet = ket for all t, then t = k for all t, which is absurd. The general solution is therefore x(t) = Aet +Btet ,
where A and B are arbitrary constants. (b) A particular solution of the equation is obviously u (t) = 3,
so the general solution of the nonhomogeneous equation is x(t) = Aet + Btet + 3.
4. By direct differentiation, u1 = eat and u2 = eat/(1) are easily seen to be solutions. They are not
proportional, so the general solution is x(t) = Aeat + Beat/(1) , where A and B are arbitrary constants.

21.9
2. (a) x = C1 et + C2 et 21 sin t. Not stable. (b) x = C1 et + C2 et 1 t
2 te . Not stable.
(c) x = Ae5t + Bte5t + 75
2
t + 125
3
. Not stable.
1 15 1
4. x = Ae2t + Be7t/4 + t + 2 + (2 sin t + 3 cos t)
14 14 65
6. (a) If x = tf (1/t), then x = f (1/t) (1/t)f  (1/t) and x = (1/t 3 )f  (1/t). But then x + t n2 x =
(1/t 3 )f  (1/t) + t n2 tf (1/t) = (1/t 3 )[f  (1/t) + t n+2 f (1/t)] = 0, because when x = f (t) satises
x + t n2 x = 0, then f  (u) + un2 f (u) = 0 at u = 1/t in particular, and so f  (1/t) + t n+2 f (1/t) = 0.
(b) Let n = 2. Then x + t n2 x = x + x = 0, with general solution x = A sin t + B cos t. From (a), it
follows that x +t n2 x = x +t 4 x = 0, or t 4 x +x = 0, has the solution x = t[A sin(1/t)+B cos(1/t)].

8. For = (a ) > 0 the solution is p(t) = C1 ert + C2 ert k/, where r = ; for = 0 the
solution is p(t) = C1 t + C2 + 21 kt 2 ; for < 0 the solution is p(t) = C1 cos st + C2 sin st k/, where

s = . In no case is the solution ever stable.
10. If x = uert , then x = ert (u + ru) and x = ert [u + 2r u + r 2 u]. Hence, x + a x + bx = ert [u + (2r +
a)u + (r 2 + ar + b)] = ert u because r = a/2 and r satises the characteristic equation. We conclude
that x = uert satises the given equation provided u = 0. Then u = At + B, and the conclusion follows.

Knut Sydster and Peter Hammond 2010


66 APPENDIX B SUMS, PRODUCTS, AND INDUCTION

Appendix B Sums, Products, and Induction


B.1

2. (a) 2 0 + 2 1 + 2 2 + 2 3 + 2 4 = 2(1 + 2 + 3 + 2) = 2(3 + 2 + 3)
(b) (x + 0)2 + (x + 2)2 + (x + 4)2 + (x + 6)2 = 4(x 2 + 6x + 14)
(c) a1i b2 + a2i b3 + a3i b4 + + ani bn+1 (d) f (x0 )x0 + f (x1 )x1 + f (x2 )x2 + + f (xm )xm
23+35+47 6 + 15 + 28 49
4. 100 = 100 = 100 144.12
13+25+37 3 + 10 + 21 34
6. (a) The total number of people moving from region i. (b) The total number of people moving to region j .

B.2
2. (a + b)6 = a 6 + 6a 5 b + 15a 4 b2 + 20a 3 b3 + 15a 2 b4 + 6ab5 + b5 . (The coefcients are those in the sixth
row of Pascals triangle in the text.)
  
8 876 8 8
4. (a) = = 56. Also, = = 56;
 3 1 2
3 8 3 5 
8 8 8765 8+1 9 9876
+ = 56 + = 126 and = = = 126.
3 3+1 1234 3+1  4 1234
m m(m 1) (m k + 1) m! m
(b) = = = and
 k  k! (m k)!k! mk
m m m! m! m!(k + 1 + m k) (m + 1)!
+ = + = = =
k k + 1 (m k)!k! (m k 1)!(k + 1)! (m k)!(k + 1)! (m k)!(k + 1)!
m+1
k+1
"
n1
6. (a + id) = a + (a + d) + + [a + (n 2)d] + [a + (n 1)d]
i=0
= [a + (n 1)d] + [a + (n 2)d] + + (a + d) + a = 21 [2a + (n 1)d]n = na + n(n1)d
2 .
An alternative proof is this:
"
n1 "
n1 "
n1
1 n(n 1)d
(a + id) = a+d i = na + [1 + (n 1)](n 1) = na + .
2 2
i=0 i=0 i=0

B.3
i
2. j =1 aij is the sum of all the i numbers in the ith row, so in the rst double sum we sum all the sums in

the m rows. m i=j aij is the sum of all the m j + 1 numbers in the j th column, so in the second double
sum we sum all the sums in the m columns.

B.4
%
n
2k 2k % 4k 2
n
4 12 4 22 4n2
2. (a) = =
2k 1 2k + 1 4k 2 1 4 12 1 4 2 2 1 4n2 1
k=1 k=1
%n
ai a1 a2 an "
n1
 % n
 "
n1
 
(b) = (c) a s bi = ai+1 ai+2 an bi =
b b1 b2 bn
i=1 i i=1 s=i+1 i=1
(a2 a3 an )b1 + (a3 a4 an )b2 + + an bn1

Knut Sydster and Peter Hammond 2010


APPENDIX C TRIGONOMETRIC FUNCTIONS 67

B.5
2. We prove only [B.7]; the proof of [B.6] is very similar, but slightly easier. For n = 1 the LHS and the
RHS of [B.7] are both equal to 1. Suppose [B.7] is true for n = k, so that
"k & '
k(k + 1) 2
i = 1 + 2 + 3 + + k =
3 3 3 3 3
2
i=1

Then

"
k+1 & '2
k(k + 1)
i = 1 + 2 + 3 + + k + (k + 1) =
3 3 3 3 3 3
+ (k + 1)3 = (k + 1)2 ( 41 k 2 + k + 1)
2
i=1

(k + 1)2 (k 2 + 4k + 4)  (k + 1)(k + 2) 2
But this last expression is equal to = , which proves that [B.7]
4 2
is true for n = k + 1. By induction, we have proved [B.7].
4. The claim is true for n = 1. Suppose k 3 + (k + 1)3 + (k + 2)3 is divisible by 9. Then (k + 1)3 + (k +
2)3 + (k + 3)3 = (k + 1)3 + (k + 2)3 + k 3 + 9k 2 + 27k + 27 = k 3 + (k + 1)3 + (k + 2)3 + 9(k 2 + 3k + 3)
is divisible by 9, because the rst three terms are divisible by 9 by the induction hypothesis, whereas the
last term is also obviously divisible by 9.
6. s1 = 41, s2 = 43, s3 = 47, s4 = 53, s5 = 61 are all prime. For n = 41, we have n2 n + 41 = (41)2 ,
which is not prime!

Appendix C Trigonometric Functions


C.1
2. Look at Fig. C.1 on page 866 of the text. If x is changed to x, the point Px will have coordinates (u, v),
so sin(x) = sin x, and cos(x) = cos x. Then by denition [C.2], tan(x) = sin(x)/ cos(x) =
sin x/ cos x = tan x.
4. cos(y /2) = sin y follows directly from [C.8]. Then, from the hints in the question, as well as [C.8]
and the result of Problem 2, it follows that

sin(x + y) = cos[x + (y /2)] = cos x cos[(y /2)] + sin x sin[(y /2)]


= sin x cos y + cos x sin y

Also, using the result of Problem 2 again, this last equation implies that

sin(x y) = sin x cos(y) + cos x sin(y) = sin x cos y cos x sin y

6. (a) 1/2. (Either drawa gure similar C.3 on page868, or use the formula for sin(x y) in
to Figure
Problem 4.) (b) 3/2 (c) 2/2 (d) 2/2 (e) 3/3 (f) sin /12 = sin(/3 /4) =
sin /3 cos /4 cos /3 sin /4 = 41 ( 6 2).
8. Note that x + y = A and x y = B imply x = 21 (A + B) and y = 21 (A B). The claimed formula
then follows easily from the hint.

Knut Sydster and Peter Hammond 2010


68 APPENDIX C TRIGONOMETRIC FUNCTIONS

10. (a) See Fig. C.1.10(a). Period , amplitude 1. (b) See Fig. C.1.10(b). Period 4 , amplitude 3.
(c) See Fig. C.1.10(c). Period 2/3, amplitude 2.

y y y
3 4
1 2
1
2 4
2
x x
1
1 2
3 2 x

Figure C.1.10(a) Figure C.1.10(b) Figure C.1.10(c)

12. (a) y = 2 sin 41 x (b) y = 2 + cos x (b) y = 2ex/ cos x


14. The given equality is (cos x cos y)2 + (sin x sin y)2 = 1 + 1 2 cos(x y). By expanding and using
[C.7], we easily nd formula [C.8].

C.2
sin x cos x cos x sin x( sin x) cos2 x + sin2 x
2. y = tan x = gives y  = = = 1 + tan2 x. Since
cos x cos2 x cos2 x
cos2 x + sin2 x = 1, we get y  = 1/ cos2 x as an alternative answer.
4. (a) a sin ax (b) a sin bt + abt cos bt (c) a cos(at + b) sin[sin(at + b)] cos{cos[sin(at + b)]}
6. f  (x) = 3(sin x x 1)2 (cos x 1). In the open interval I = (0, 3/2), f  (x) = 0 only at cos x = 1,
when x = /2. (It is easy to see that sin x < x + 1 for all x 0 because the function dened by
g(x) = x + 1 sin x for all x 0 satises g(0) = 1 and g  (x) = 1 cos x 0 for all x 0.) Thus,
the only possible extreme points are 0, /2, and 3/2. Comparing the function values at these points,
we nd that f (x) has its maximum 1 at x = 0, and its minimum (2 + 3/2)3 at x = 3/2. (The
extreme-value theorem ensures that extreme points do exist.)
cos y y cos x
8. Implicit differentiation yields 1 cos y + x( sin y)y  y  sin x y cos x = 0, so y  = .
sin x + x sin y
At (, /2), y  = 1/2, so the equation for the tangent is y = x/2.
 /2 /2
10. (a)  cos x + C (b) 0 cos x dx= 0 sin x = sin(/2) 0 = 1 (c)  Integrating by parts yields
I = sin x dx = sin x( cos x) cos x( cos x) dx = sin x cos x + cos x dx = sin x cos x +
2 2

 I = sin x cos x +x I +C. Solving for I gives I = 2 (sin x cos x x)+C1 .


2 1
(1sin
 x) dx. Hence,
(d) 0 x cos x dx = 0 x sin x 0 sin x dx = 0 + 0 cos x = cos cos 0 = 2.
12. (a) /4 (b) /2 (c) /6 (d) /3. (You can read all these values off from Table C.1.)
1 ex + ex
14. Make use of [C.20] and the chain rule to derive y  = 
x 2
. Then simplify.
ex e 2
1+
C.3 2

2. See Fig. C.3.2.

Knut Sydster and Peter Hammond 2010


69

4. (a) 2 3(cos /3 + i sin /3) (b) cos + i sin (c) 4(cos 4/3 + i sin 4/3)
(d) 2(cos 7/4 + i sin 7/4)
Imaginary axis
4i
w = 1 + 3i
3i
2i
i z+w =3+i

Real axis
1 1 2 3 4
i
2i z = 2 2i

Figure C.3.2

Knut Sydster and Peter Hammond 2010


70

TEST I (Elementary Algebra)

A certain familiarity with elementary algebra is an essential prerequisite for reading the textbook (and for
understanding most modern economics texts). This test is designed for students and instructors to discover
whether the students have the proper background. (In a number of countries, many beginning economics
students background in elementary algebra appears to have become much weaker during the last few years.
In fact, lecturers using this test (or similar ones) have been shocked by the results, and have had to readjust
their courses.)
At the head of each problem, immediately after the number, the relevant sections of the introductory
chapters in the book are given in parentheses, followed in square brackets by the number of points for a
correct answer to each separate part of the problem. In a 2030 minute test, any student who scores less than
50 (out of 100) has serious problems with elementary algebra. Such students denitely need to review the
relevant section of Chapters 1 and 2, or consult other elementary material.
The correct answers are given on a separate page following the test.

1 (1.2) [Points: (a) 2; (b) 2; (c) 3; (d) 3] Calculate/simplify:


  
7 3 72 2 2 2 219 217
(a) (b) (5.5 3.5) 3 (c) (d)
74 5 5 5 219 + 217

2 (1.21.4) [Points: (a) 2; (b) 2; (c) 2; (d) 4]



(a) If 2x 2 y = 5, then 4x 4 y 2 = ?
(b) 11
% of 3500 is? (c) 132 122 = ?
3+ 2
(d) Rationalize the denominator of (i.e. nd a new fraction that is equal but has no square
3 2
root in the denominator).

3 (1.3) [Points: (a) 2; (b) 2; (c) 2] Expand:


(a) (x + 2y)2 (b) (2x 3y)2 (c) (a + b)(a b)

4 (1.3) [Points: (a) 2; (b) 3; (c) 3; (d) 4] Expand and simplify:


(a) 5a (3a + 2b) 2(a 3b) (b) (x + 2)2 + (x 2)2 2(x + 2)(x 2)
(c) (1 x)2 (1 + x)2 (d) (2 a)3

5 (1.2) [Points: 4] If the GNP of a certain country in 2000 was 8 billion dollars, write down an expression
for the GNP 6 years later if it increases by 5% each year.

6 (1.3) [Points: (a) 3; (b) 3; (c) 4] Factorize:


(a) 5a 2 b + 15ab2 (b) 9 z2 (c) p 3 q 4p 2 q 2 + 4pq 3

Knut Sydster and Peter Hammond 2010


71

7 (1.4) [Points: (a) 2; (b) 2; (c) 2] Expand and simplify to a single fraction:
1 1

1 1 6a a 3a 2 3
(a) (b) + (c)
2 3 5 10 20 1 1

4 6

8 (1.5) [Points: (a) 2; (b) 2; (c) 2; (d) 2] Calculate/simplify:



(a) 251/2 (b) (x 1/2 y 1/4 )4 (c) 27a 6 (d) p 1/5 (p 4/5 p 1/5 )
3

9 (2.1) [Points: (a) 2; (b) 2; (c) 2] Solve the following equations for the unknown x:
3 1 3
(a) x = 6 (b) = (c) 3 x = 2
5 x1 2x + 3

10 (1.6) [Points: (a) 2; (b) 3; (c) 3] Solve the following inequalities:


x1
(a) 3x + 2 < 5 (b) 0 (c) x 3 < x
x+3

11 (2.3) [Points: (a) 3; (b) 3; (c) 3] Solve the following equations:


(a) 3x 9x 2 = 0 (b) x 2 2x 15 = 0 (c) 2P 2 = 2 3P

12 (2.4) [Points: (a) 3; (b) 4; (c) 4] Solve the following systems of equations:
3 3
+ =3
2x y = 5 1.5p 0.5q = 14 p q
(a) (b) (c)
x + 2y = 5 2.5p + 1.5q = 28 3 1
=7
p q

Knut Sydster and Peter Hammond 2010


72

TEST II (Elementary Mathematics)

Students who now enter university or college courses in economics tend to have a wide range of math-
ematical backgrounds and aptitudes. At the low end, they may have no more than a shaky command of
elementary algebra. Or, at the high end, they may already have a ready facility with calculus, though
often it is some years since economics students took their last formal mathematics course. Experience
suggests therefore that right from the start of the course, it is very important that the instructor, as well
as each individual student, should get some impression of what the student knows well, what is vaguely
familiar, and what seems to be more or less forgotten or perhaps never learned at all.
The present test is meant to test the students actual knowledge of some elementary mathematical
topics of interest to economists. The level is nevertheless more advanced than for Test I and the topics
covered are discussed in the earlier chapters of the main text. The maximum total score is 100.

1 [Points: (a) 2; (b) 2; (c) 2; (d) 2]

(a) 25 + 25 = 2x  x = (b) 315 + 315 + 315 = 3y  y =


 226 223 z
(c) 132 52 = (d) =  z =
226 + 223 9
2 [Points: (a) 1+1; (b) 2+2]
(a) Find the slopes of the following straight lines:
(i) y = 23 x + 4 (ii) 6x 3y = 5
(b) Find the equation of the straight line that:
(i) passes through (2, 3) and has slope 2.
(ii) passes through both (a, 0) and (0, b).

3 [Points: 5] Fill in the following table and sketch the graph of y = x 2 + 2x + 4.

x 2 1 0 1 2 3 4

y = x 2 + 2x + 4

4 [Points: 4+4] Determine the maximum/minimum points for:


(a) y = x 2 4x + 8 (b) y = 2x 2 + 16x 14

5 [Points: 4+4] Perform the following polynomial divisions:


(a) (2x 3 3x + 10) (x + 2) (b) (x 4 + x) (x 2 1)

6 [Points: 5] f (x) = 43 x 3 15 x 5 . For what values of x is f  (x) = 0?

7 [Points: 3+3+3+3] Sketch the graph of a function f in each of the following cases:
(a) f  (x) > 0 and f  (x) > 0, (b) f  (x) > 0 and f  (x) < 0
(c) f  (x) < 0 and f  (x) > 0, (d) f  (x) < 0 and f  (x) < 0

Knut Sydster and Peter Hammond 2010


73

8 [Points: 3+3]
(a) The cost in dollars of extracting T tons of a mineral ore is given by C = f (T ). Give an economic
interpretation of the statement that f  (1000) = 50.
(b) A consumer wants to buy a certain item at the lowest possible price. Let P (t) denote the lowest price
found after searching the market for t hours. What are the likely signs of P  (t) and P  (t)?

9 [Points: 2+2+2+2] Write down the general rules for differentiating the following:
(a) y = f (x) + g(x)  y  = (b) y = f (x)g(x)  y  =
(c) y = f (x)/g(x)  y  = (d) y = f (g(x))  y  =

10 [Points: 2+2+2+2+2+2+2+2] Differentiate the following functions:


x
(a) y = x 2 (b) y = x 5 /5 (c) y = (d) y = (x 2 + 5)6
x+1
(e) y = ex (f) y = ln x (g) y = 2x (h) y = x x

11 [Points: 2+2+2+2+2] Which of the following statements are correct?


(a) The rule which converts a temperature measured in degrees Fahrenheit into the same temperature
measured in degrees Celsius is an invertible function.
(b) A concave function always has a maximum.
(c) A differentiable function can only have an interior maximum at a stationary point for the function.
(d) If f  (a) = 0, then a is either a local maximum point or a local minimum point.
(e) The conditions f  (a) = 0 and f  (a) < 0 are necessary and sufcient for a to be a local maximum
point for f .

12 [Points: 2+2+2+2] In each of the following cases, decide whether the given formula is correct or not:

1
(a) x 2 dx = x 3 + C
3
  
(b) [f (x) + g(x)] dx = f (x) dx + g(x) dx
  
(c) f (x)g(x) dx = f (x) dx g(x) dx
 b
(d) x dx = b a
a

Knut Sydster and Peter Hammond 2010


74

Answers to TEST I
7 3 72 73+2 75
1. (a) 4
= 4 = 4 = 754 = 71 = 7 (b) (5.5 3.5)3 = 23 = 8
 7  7  7
2 2 2 8 219 217 217 (22 1) 3
(c) = (= 0.064) (d) 19 = =
5 5 5 125 2 + 217 217 (22 + 1) 5
2. (a) 4x 4 y 2 = (2x 2 y)2 = 25 (b) 11 % of 3500 is 3500 11/100 = 3500 0.11 = 385

(c) 132 122 = (13 + 12)(13 12) = 25 = 5 (or 132 122 = 169 144, etc.)
( 132 122 = 132 122 = 13 12 = 1 is a SERIOUS mistake.)

3+ 2 ( 3 + 2)( 3 + 2) 3+2 3 2+2
(d) = = =5+2 6
3 2 ( 3 2)( 3 + 2) 32
3. (a) (x + 2y)2 = x 2 + 4xy + 4y 2 (b) (2x 3y)2 = 4x 2 12xy + 9y 2
(c) (a + b)(a b) = a 2 b2
4. (a) 5a (3a + 2b) 2(a 3b) = 5a 3a 2b 2a + 6b = 4b
(b) (x + 2)2 + (x 2)2 2(x + 2)(x 2) = [(x + 2) (x 2)]2 = 42 = 16
(c) (1 x)2 (1 + x)2 = [(1 x)(1 + x)]2 = (1 x 2 )2 = 1 2x 2 + x 4
(d) (2 a)3 = (2 a)2 (2 a) = (4 4a + a 2 )(2 a) = 8 4a 8a + 4a 2 + 2a 2 a 3 =
a 3 + 6a 2 12a + 8
5. 8(1.05)6 billion dollars.
6. (a) 5a 2 b + 15ab2 = 5ab(a + 3b) (b) 9 z2 = (3 z)(3 + z)
(c) p 3 q 4p 2 q 2 + 4pq 3 = pq(p 2 4pq + 4q 2 ) = pq(p 2q)2 . (One point for the rst equality.)
1 1 3 2 3 2 1
7. (a) = = = (b) 20 is the common denominator, so
2 3 23 32 6 6 6
6a a 3a 4 6a 2a 3a 24a 2a + 3a 25a 5a
+ = + = = =
5 10 20 20 20 20 20 20 4
1
1 6
4 2
(c) 21 13 = 12 12
= 12 = 2 (or 21 13 = 2( 41 16 ), so the ratio is 2).
4 6
3
12 2
12
1
12

8. (a) 25 = 5 (b) (x y 1/4 )4 = x (1/2)4 y (1/4)4 = x 2 y 1 (c) 27a 6 = 3 27 a 6 = 3a 2
1/2 1/2 3 3

(d) p 1/5 (p 4/5 p 1/5 ) = p1/5 p 4/5 p 1/5 p 1/5 = p1/5+4/5 p 1/51/5 = p 1
9. (a) 3x = 30, so x = 10
(b) 2x + 3 = 3(x 1), so x = 6. (Neither denominator is 0 when x = 6.)

(c) If 3 x = 2, then 3 x = 4, so x = 1. This is indeed a solution, as is easily checked.
10. (a) 3x + 2 < 5 or 3x < 3, so that x > 1. (Recall that an inequality is reversed if multiplied
by a negative number.)
(b) 3 < x 1. (Use a sign diagram. Note that the fraction is undened if x = 3.)
(c) x < 1 or 0 < x < 1. (x 3 < x, or x 3 x < 0, and so x(x 2 1) < 0, or x(x 1)(x + 1) < 0.
Then use a sign diagram.)
11. (a) 3x(1 3x) = 0, so x = 0 or x = 1/3 (b) x = 3, x = 5 (c) P = 2 or P = 1/2
12. (a) x = 3, y = 1 (b) p = 10, q = 2 (c) Put x = 1/p, y = 1/q. Then 3x + 3y = 3 and
3x y = 7. The solution to this system is x = 2 and y = 1. But then p = 1/2 and q = 1.

Knut Sydster and Peter Hammond 2010


75

Answers to TEST II

1. (a) 25 + 25 = 2 25 = 26 , so x = 6.
(b) 315 + 315 + 315 = 3 315 = 314 , so y = 14.

(c) 132 52 = 169 25 = 144 = 12.
226 223 223 (23 1) 23 1 7
(d) 26 = = = , so z = 7.
2 + 223 223 (23 + 1) 23 + 1 9
2. (a) (i) 3/2 (ii) 2 (b) (i) y = 2x 1 (ii) y = (b/a)x + b
3.
x 2 1 0 1 2 3 4

y = x 2 + 2x + 4 4 1 4 5 4 1 4

The graph of the function is shown in the gure below.


y
5
y = x 2 + 2x + 4
4
3
2
1
x
4 3 2 1 1 2 3 4
1
2
3
4

4. (a) Minimum 4 for x = 2. (Follows from x 2 4x + 8 = x 2 4x + 22 + 8 22 = (x 2)2 + 4,


or by using calculus.)
(b) Maximum 18 for x = 4. (Follows from 2x 2 + 16x 14 = 2(x 2 8x + 7) =
2(x 2 8x + 42 + 7 42 ) = 2[(x 4)2 9] = 2(x 4)2 + 18, or by using calculus.)
2x 3 3x + 10
5. (a) = 2x 2 4x + 5
x+2
x4 + x x+1 1
(b) 2 = x2 + 1 + 2 = x2 + 1 +
x 1 x 1 x1
6. f  (x) = 4x 2 x 4 = x 2 (4 x 2 ) = x 2 (2 x)(2 + x) = 0 for x = 0 and for x = 2.
7. y y y y

y = f (x) y = f (x)

y = f (x) y = f (x)

x x x x

(a) (b) (c) (d)

Knut Sydster and Peter Hammond 2010


76

8. (a) The marginal cost is 50 when output is 1000 tons. (Or: The cost of extracting one ton more than
1000 tons is approximately 50 dollars.)
(b) P  (t) 0, because more search never leads to a higher best price, and generally leads to a lower
one. However, P  (t) is likely to be positive, because the longer you search the smaller gain in price
you will obtain. (P (t) is decreasing and convex.)
9. (a) y = f (x) + g(x) y  = f  (x) + g  (x)
(b) y = f (x)g(x) y  = f  (x)g(x) + f (x)g  (x)
f  (x)g(x) f (x)g  (x)
(c) y = f (x)/g(x) y  =
(g(x))2
 
(d) y = f (g(x)) y = f (g(x))g (x) 

x 1
10. (a) y = x 2 y  = 2x (b) y = 1 5
5x y = x4 (c) y = y =
x+1 (x + 1)2
(d) y = (x 2 + 5)6 y  = 6(x 2 + 5)5 2x = 12x(x 2 + 5)5 (e) y = ex y  = ex
(f) y = ln x y  = 1/x (g) y = 2x y  = 2x ln 2 (h) y = x x y  = x x (ln x + 1)
11. (a) Correct. (b) Wrong. (Consider y = ex .) (c) Correct. (d) Wrong. (a could be an inec-
tion point.) (e) Wrong. (The conditions are sufcient, but not necessary. For example, f (x) =
x 4 has a maximum at x = 0 and yet f  (0) = f  (0) = 0.)
d
  
12. (a) Correct.
 (b) Correct.
 (c) Wrong, because dx f (x) dx g(x) dx
= f (x) g(x) dx + g(x) f (x) dx  = f (x)g(x), except for very special functions f and g.
b b
(d) Wrong, because a x dx = a 21 x 2 = 21 (b2 a 2 ), which equals b a only when a + b = 2 or
a = b.

Knut Sydster and Peter Hammond 2010

Вам также может понравиться